In vielen Artikeln und Pressemitteilungen über Astronomie liest man Sätze wie „Das Licht dieser Galaxie hat 12 Milliarden Jahre bis zu uns gebraucht.“ Folgt daraus, dass die Galaxie auch 12 Milliarden Lichtjahre entfernt ist? Nicht wirklich. In einem Universum, dass sich ausdehnt und in dem die Lichtgeschwindigkeit nicht unendlich groß ist, sind die Dinge ein wenig komplizierter.

Dass sich Himmelsobjekte bewegen, merken wir an der Rotverschiebung. Damit bezeichnet man ganz allgemein den Effekt, dass sich die Wellenlänge einer elektromagnetischen Welle zwischen der Aussendung und dem Empfangszeitpunkt verlängert hat. Rotverschiebung kann auf verschiedene Arten zustande kommen. Am bekanntesten ist der Dopplereffekt. Wir kennen ihn von vorbeifahrenden Einsatzfahrzeugen. Wenn sich ein Polizeiauto mit Sirene auf uns zu bewegt, dann hören wir die Schallwellen mit einer höheren Frequenz. Da sich das Auto bewegt, während es Schallwellen aussendet, verkürzt sich der Abstand zwischen zwei Wellenbergen. Wenn das Auto aus unserer Sicht davon fährt, ist es umgekehrt. Der Abstand erhöht sich, die Frequenz wird niedriger und der Ton tiefer. Das funktioniert auch bei Licht. In der Astronomie kann man so zum Beispiel feststellen, ob sich ein Stern auf uns zu bewegt oder von uns weg. Bei einem Stern, der sich uns nähert werden die Wellenlängen ein wenig zum kurzwelligen blauen Bereich des Spektrums verschoben; entfernt er sich, dann verschiebt sich alles zum roten Bereich hin. Durch die Messung der Spektrallinien können wir feststellen, wie stark diese Verschiebung ist. Diese Linien finden sich überall im Licht das von den Sternen ausgestrahlt ist. Sie werden von den verschiedenen chemischen Elementen erzeugt, die das Licht unterwegs trifft und jedes absorbiert einen charakteristischen Teil der Strahlung. Aus Labormessungen und theoretischen Untersuchungen wissen wir ganz genau, wo sich diese Linien im Spektrum befinden müssen. Haben sich die Linien verschoben, dann bewegt sich die Lichtquelle. Mit dieser Methode sucht man zum Beispiel nach extrasolaren Planeten. Deren Bewegung bringt den Stern den sie umkreisen ein klein wenig zum Wackeln und so wackelt er mal ein bisschen auf uns zu, mal ein bisschen von uns weg. Die gemessene Rotverschiebung entlarvt die Existenz des Planeten.

i-c18e7c5e37ed171eedc4714ad8fb2446-Hudf09z10nl-thumb-500x500.png
UDFj-39546284, eine der am weitesten entfernten bekannten Galaxien (Bild:NASA, ESA, Garth Illingworth (University of California, Santa Cruz) and Rychard Bouwens (University of California, Santa Cruz and Leiden University) and the HUDF09 Team)

Im Universum bewegen sich die Himmelskörper aber nicht nur von selbst. Sie bewegen sich auch scheinbar, aufgrund der Expansion des Universum. Ich habe deswegen „scheinbar“ geschrieben, weil es sich bei der Expansion des Universums nicht um eine Bewegung der Galaxien durch den Raum handelt. Das, was Edwin Hubble in den 1920er Jahren entdeckt hat, war etwas ganz anderes. Er fand heraus, dass sich der Raum selbst ausdehnt. Die Galaxien entfernen sich alle voneinander, weil sich der Raum zwischen den Galaxien beständig ausdehnt (Die Galaxien selbst, und mit ihnen die Sterne und Planetensysteme und alles was sich auf ihnen befindet dehnen sich nicht aus. Auf diesen kleineren Skalen wirkt die Gravitationskraft der Expansion entgegen und hält die Objekte zusammen. Darum wird unser Sonnensystem zum Beispiel nicht größer, auch wenn sich das Universum ausdehnt). Wir können auch hier messen, wie sich die Spektrallinien der fernen Galaxien verschieben. Diese kosmologische Rotverschiebung hat aber nichts mit dem Doppler-Effekt zu tun. Wie schon gesagt: Es ist nur eine scheinbare Bewegung; in Wahrheit dehnt sich nur der Raum aus und die Galaxien bewegen sich nicht (bzw. ist ihre Eigenbewegung auf diesen Skalen vernachlässigbar klein).

Man gibt die Rotverschiebung in der Astronomie mit dem Parameter z an:

i-add9654f6e1ca42cbc2e98919f88b5a7-rotverschiebung.png

λ0 ist die Wellenlänge bei der sich die Spektrallinie eigentlich befinden sollte, λbeoabachtet ist die Wellenlänge, bei der man sie tatsächlich gemessen hat. Hat eine Galaxie zum Beispiel eine Rotverschiebung von z=2, dann ist die gemessene Wellenlänge dreimal so groß wie erwartet. Eine Galaxie mit einer Rotverschiebung von z=5 zeigt Spektrallinien bei Wellenlängen die sechsmal größer sind, als man es erwarten würde. Eine Galaxie mit z=5 bewegt sich also schneller von uns weg als eine mit z=2 und ist damit auch weiter entfernt. Aber wie weit denn nun genau? Wie berechnet man aus der Rotverschiebung eine Entfernung?

i-b7d798025af6f2ed2c1ee3d96c156ec7-expansion.gif

Das ist knifflig. Eben weil sich unser Universum ausdehnt, kann man einer Rotverschiebung nicht einfach eindeutig eine Entfernung zu ordnen. Es geht nicht darum, dass man die Entfernung nicht kennt, oder das es schwierig ist, sie zu berechnen. Es ist eine Definitionsfrage und hängt davon ab, wie man „Entfernung“ definiert. Im Alltag ist das recht eindeutig. Entfernung ist – im simpelsten Fall – das, was ich mit dem Metermaß zwischen zwei Punkten messen kann. Soll ich die Entfernung zwischen zwei parkenden Autos bestimmen, ist das recht einfach. Wenn man mich aber fragt, wie weit zwei fahrende Autos voneinander entfernt sind, dann wird es schon schwieriger. Die Entfernung ändert sich ja ständig. Im Universum ist es noch ein wenig komplizierter. Eine Galaxie sendet zu einem bestimmten Zeitpunkt einen Lichtstrahl aus. Wenn dieses Licht die Erde erreicht, dann ist viel Zeit vergangen. In dieser Zeit hat sich das Universum weiter ausgedehnt (siehe die Animation oben). Wir sehen das Licht, das aus einer Zeit stammt, die in der Vergangenheit liegt und von einem Ort ausgesandt wurde, an dem sich die Galaxie heute schon längst nicht mehr befindet. Wie genau definiert man in diesem Fall eine „Entfernung“? Den Astronomen sind einige Möglichkeiten eingefallen.

  • Da ist zum einen die Leuchtkraftentfernung. Da sich das Universum ausdehnt, sehen wir die Galaxien viel schwächer, als sie eigentlich leuchten weil die Lichtwellen so stark gedehnt werden. Die Leuchtkraftentfernung gibt an, wie weit entfernt die Galaxien zu sein scheinen, wenn wir nach ihrer Helligkeit gehen.
  • Die Winkeldurchmesserentfernung bezieht sich auf das Erscheinungsbild der Galaxien. Weit entfernte Galaxien haben ihr Licht abgeschickt, als das Universum noch jünger und kleiner war. Da es damals noch nicht so ausgedehnt war, waren die Galaxien uns noch näher. Ferne Galaxien erscheinen uns also größer als sie es eigentlich sind. Die Winkeldurchmesserentfernung ist also ein Maß dafür, wie weit entfernt die Galaxien waren, als die Lichtwellen abgestrahlt wurden.
  • Die mitbewegte Entfernung ist eine Entfernungsskala, die sich mit dem Universum ausdehnt. Sie sagt uns, wie weit eine Galaxie heute entfernt ist.
  • Die Laufzeitentfernung basiert auf der Zeit, die das Licht gebraucht hat, um von einer Galaxie bis zu uns zu kommen. Der Satz aus der Einleitung („Das Licht dieser Galaxie hat 12 Milliarden Jahre bis zu uns gebraucht.“) macht also nur eine Aussage über die Laufzeitentfernung – 12 Milliarden Lichtjahre – aber es handelt sich nicht um „die“ Entfernung. Die gibt es nicht.

Noch ein Beispiel: Das Universum ist 13,7 Milliarden Jahre alt. Das Licht der fernsten Objekte wird also etwa 13 Milliarden Jahre bis zu uns gebraucht haben. Die Laufzeitentfernung von uns bis zu diesen fernsten Objekten beträgt also ebenfalls etwa 13 Milliarden Lichtjahre. Misst man den Abstand zu ihnen aber mit der mitbewegten Entfernung, dann sind sie etwa 45 Milliarden Lichtjahre entfernt.

Wenn es nur um kleine Entfernungen geht, dann liefern alle vier Entfernungsskalen die gleichen Werte. Erst bei kosmologischen Distanzen gibt es Unterschiede:

i-b02a342282a06b0d0e5e4f9d9fde7cba-redshift-thumb-500x421.gif

Auf der x-Achse ist die gemessene Rotverschiebung aufgetragen, auf der y-Achse die Entfernung, die sich daraus für die verschiedenen Definitionen ergibt. DL ist die Leuchtkraftdistanz, DC die mitbewegte Distanz, DT die Laufzeitentfernung und DA die Winkeldurchmesserentfernung. Ein Objekt mit einer Rotverschiebung von z=10 hätte zum Beispiel eine Winkeldurchmesserentfernung von 3 Milliarden Lichtjahre. Die Laufzeitentfernung wäre 13 Milliarden Lichtjahre. Das Licht stammt also aus der Zeit kurz nach dem Urknall. Die mitbewegte Entfernung beträgt allerdings 32 Milliarden Lichtjahre. Und die Leuchtkraftentfernung ist so groß, dass sie nicht mehr auf das Diagramm passt. Eine Galaxie mit z=10 erscheint uns ein paar hundert Milliarden Lichtjahre entfernt und deswegen ist es auch nicht verwunderlich, wenn sie so schwer zu beobachten sind! Die Galaxie mit der bisher größten gemessenen Rotverschiebung heißt UDFy-38135539. Man hat ein z von 8,6 gemessen. Das Licht von UDFy-38135539 war 13,1 Milliarden Jahre zu uns unterwegs. Die mitbewegte Entfernung beträgt knapp 30 Milliarden Lichtjahre. Von ihrer Größe her erscheint uns die Galaxie als wäre sie nur 3 Milliarden Lichtjahre entfernt, ihre Helligkeit entspricht allerdings einer 290 Milliarden Lichtjahre entfernte Galaxien (hier kann die Rotverschiebung automatisch umrechnen lassen). Bei der Galaxie UDFj-39546284 hat man sogar eine Rotverschiebung von z=10 gemessen. Diese Messung wurde bis jetzt allerdings noch nicht bestätigt. Es handelt sich aber auf jeden Fall um eines der am weitesten entfernten Objekte, die wir kennen. Egal mit welcher Definition von Entfernung man rechnet 😉

166 Gedanken zu „Die Rotverschiebung und die vielen Entfernungen der Kosmologie“
  1. „Bei einem Stern, der sich uns nähert werden die Wellenlängen ein wenig zum langwelligen roten Bereich des Spektrums verschoben; entfernt er sich, dann verschiebt sich alles zum blauen Bereich hin. “

    Da hat sich doch der Fehlerteufel eingeschlichen. Blauverschiebung – energiereicher – falls der Stern sich auf uns zu bewegt und Rotverschiebung – energieärmer – falls der Stern sich von uns weg bewegt.

  2. Sehr interessanter Artikel!
    Aber folgendes gehört doch umgedreht, oder?

    „Bei einem Stern, der sich uns nähert werden die Wellenlängen ein wenig zum langwelligen roten Bereich des Spektrums verschoben; entfernt er sich, dann verschiebt sich alles zum blauen Bereich hin.“

  3. Mal ne dumme Frage:
    Wie schließt man den auf das Alter des Universums ? Doch über die beobachteten Geschwindigkeiten die ja mit der Entfernung zunehmen. Was ist aber wenn die meisten Galaxien schon außerhalb der Reichweite des Lichts sind ? Dann könnte es doch viel älter sein.

  4. Hochinteressant – und faszinierend!
    Ich bin bestenfalls Astronomie-Interessiert – aber keinsesfalls in der Lage, diese Zahlen in Gänze zu begreifen. Aber eines macht mich doch nachdenklich:
    Wenn das Licht einer Galaxie 13 Milliarden jahre zu uns gebraucht hat, der Helligkeit nach aber 290 Milliarden Jahre weit weg ist, dann passt das nicht zur Aussage, dass das Universum 13,7 Milliarden Jahre alt ist. Besagte Galaxie hätte sich ja dann mit Überlichtgeschwindigkeit von uns fortbewegt!

  5. @oldsiggi: „Wenn das Licht einer Galaxie 13 Milliarden jahre zu uns gebraucht hat, der Helligkeit nach aber 290 Milliarden Jahre weit weg ist, dann passt das nicht zur Aussage, dass das Universum 13,7 Milliarden Jahre alt ist. Besagte Galaxie hätte sich ja dann mit Überlichtgeschwindigkeit von uns fortbewegt! „

    Nein, eben nicht. Darum geht es ja gerade. Es gibt nicht DIE Entfernung. Es gibt verschiedene Arten das Wort „Entfernung“ zu definieren. Die Galaxie sieht aus wie eine, die 290 Milliarden LJ weit weg ist. Das heißt nicht, dass dies ihre tatsächliche Entfernung ist. Es gibt keine „tatsächliche Entfernung“.

  6. Er fand heraus, dass sich der Raum selbst ausdehnt. Die Galaxien entfernen sich alle voneinander, weil sich der Raum zwischen den Galaxien beständig ausdehnt (Die Galaxien selbst, und mit ihnen die Sterne und Planetensysteme und alles was sich auf ihnen befindet dehnen sich nicht aus. Auf diesen kleineren Skalen wirkt die Gravitationskraft der Expansion entgegen und hält die Objekte zusammen. Darum wird unser Sonnensystem zum Beispiel nicht größer, auch wenn sich das Universum ausdehnt).

    Nur zum Verständnis: Raum ist doch ein äußerst abstrakter Begriff und physikalisch nur beschrieben durch das Verhalten der Objekte in ihm, nicht wahr? Insofern müsste die Aussage, dass die Galaxien sich alle gleichmäßig voneinander fort bewegen, doch eigentlich gleichbedeutend sein mit der Aussage, dass der Raum zwischen de Galaxien sich ausdehnt, oder?

    Und die zweite Frage: Wenn die Rotverschiebung nicht durch den Doppler-Effekt verursacht wird, wodurch dann?

  7. Mir ist der Sinn der mitbewegten Entfernung nicht ganz klar. Du sagst, sie gibt an, wie weit das Objekt *heute* entfernt ist, aber es gibt ja eigentlich keine universelle Zeit und damit auch kein heute, das hier und auf dem entfernten Objekt gleich ist.

    Aber selbst wenn man einfach einmal eine globale Zeitbasis annehmen würde: Wenn man die mitbewegte Entfernung betrachtet, dann sind die Objekte soweit von uns entfernt, dass ihr gegenwärtiger Zustand von uns nicht beobachtbar und damit nicht Teil unserer Realität ist. Welchen physikalischen Sinn ergibt dann die mitbewegte Entfernung?

  8. Verstehe ich richtig: Das Licht brauchte von der weitesten Galaxie 13.7 Milliarden Lichtjahre zu uns. Derzeit (!?), also nach diesen 13.7 Milliarden Jahren, befindet sich die Galaxie aber schon 45 Milliarden Lichtjahre weit weg. Das ergibt eine Geschwindigkeit zwischen „uns“ Galaxien von 45/13.7 = 3.28 Lichtjahre pro Jahr. Wenn ich mich nicht irre, sind das ca. 985.000 km pro Sekunde. Wenn sich der Raum in unserem Sonnensystem, in jedem Atom und ja auch in meinem Magen so schnell ausdehnen will, dann muss mein Magen aber ganz schöne Kräfte entfalten, dass er sich so überhaupt nicht aufbläht, oder?

  9. @Peter
    Das macht mich auch ein bißchen stutzig. Die Theorie sagt, dass sich zwar der Raum ausdehnt, Masse selbst aber NICHT ! Zweidimensional gesehen quasi so wie wenn du Münzen auf einer Gummifolie aufklebst und die Folie dann in die Länge ziehst.

    Das ist einer der Punkte warum mir die ganze Urknalltheorie ein bißchen suspekt ist. Florian schreibt, dass die Gravitation die Masse quasi zusammenhält. Auf Quantenebene ist die Gravitation aber vernachlässigbar, warum dehnt sich also das Atom nicht aus. Kann nicht nur an der Gravitation liegen.

  10. @schak:

    Wie schließt man den auf das Alter des Universums ? Doch über die beobachteten Geschwindigkeiten die ja mit der Entfernung zunehmen.

    Nein, das ist so bestenfalls halb richtig. In Wirklichkeit nimmt man viele verschiedene Arten von Messungen, von denen diese „Geschwindigkeits-„Messungen nur eine ist, und schaut dann, welches rechnerische Modell am besten zu all diesen Messungen passt. Und dieses rechnerische Modell spuckt dann eben unter anderem auch einen Wert für das Alter des Universums aus.

  11. @pirx: „Insofern müsste die Aussage, dass die Galaxien sich alle gleichmäßig voneinander fort bewegen, doch eigentlich gleichbedeutend sein mit der Aussage, dass der Raum zwischen de Galaxien sich ausdehnt, oder?“

    Ja.

    „Wenn die Rotverschiebung nicht durch den Doppler-Effekt verursacht wird, wodurch dann? „

    Doppler-Effekt heisst: Die Quelle befindet sich in Bewegung. Das tut eine Galaxie aber nicht. SIe bewegt sich nicht aus eigenem Antrieb. Nur der Raum zwischen ihr und uns dehnt sich aus. Das hat das gleiche Ergebnis wie eine Rotverschiebung durch Doppler-Effekt. Aber die Galaxie bewegt sich nicht.

  12. @schak: Ich habe wohl KEINE (fundierte) Ahnung. Aber wenn der sich ausdehnende Raum ganze Galaxien mitnimmt, dann will er wohl auch Maikäfer von der Erde und Elektronengewölke vom Atomkern mitnehmen und wird das wohl auch versuchen. Aber irgendwie sind die Kräfte zwischen Maikäfer und im Atom größer. Ohne Raumausdehnung wären sie allerdings dann noch größer – und ein Atom (samt Maikäfer) müsste kollabieren. Die Raumausdehnung hält uns also zusammen – oder sind diese ganzen Überlegungen kompletter Unsinn?

  13. Zweidimensional gesehen quasi so wie wenn du Münzen auf einer Gummifolie aufklebst und die Folie dann in die Länge ziehst.

    Der Vergleich hinkt.
    Eher sind die beiden Münzen mit einer Schnur verbunden und wenn du jetzt den Gummi unter ihnen dehnst, behalten sie ihren Abstand bei.

    Florian schreibt, dass die Gravitation die Masse quasi zusammenhält.

    Auf der Ebene von Sonnen, Planten, Monden, Systemen, Galaxien.
    Auf atomarer Ebene und darunter ist die Gravitation unter ferner liefen. Atome werden durch elektromagnetische Kräfte zusammengehalten. In Atomkernen dominiert dann die starke und die schwache Kernkraft.

  14. und nochmals @schak: Natürlich werden Atome nicht nur von der Gravitation zusammen gehalten, sondern auch von elektrischen Kräften usw – und die sind eindeutig nicht vernachlässigbar!

    Das ist einer der Punkte warum mir die ganze Urknalltheorie ein bißchen suspekt ist.

    Was sind die anderen Punkte?

  15. @schak: „Das macht mich auch ein bißchen stutzig. Die Theorie sagt, dass sich zwar der Raum ausdehnt, Masse selbst aber NICHT ! „

    Es kommt auf die Kräfte an. Die Expansion ist die eine Kraft, die die Dinge ausdehnen will. Gravitation und Elektromagnetismus wirken in die andere Richtung. In deinem Körper, in der Erde, im Sonnensystem etc sind die andere Kräfte stärker als die Expansion und halten alles zusammen. Auf großen Skalen wirkt die Expansion stärker und dehnt den Raum aus.

    „Auf Quantenebene ist die Gravitation aber vernachlässigbar, warum dehnt sich also das Atom nicht aus. Kann nicht nur an der Gravitation liegen. „

    Da gibt es starke und schwache Kernkraft.

  16. @Peter Rebernik: Atome, Moleküle, Planeten, Sonnensysteme und selbst Galaxien sind schlicht so klein, dass auf sie die Raumausdehung kaum eine Wirkung hat – die Ausdehnung merkt man erst bei richtig großen Skalen so richtig. Die riesigen Geschwindigkeiten, die du da ausrechnest, kommen einfach daher, dass du Objekte in extrem großen Entfernungen betrachtest – bei kleineren Entfernungen hat man auch viel kleinere Geschwindigkeiten…

    Viel anschaulicher, als solche Geschwindigkeiten auszurechnen, finde ich folgende Zahl: pro Milliarde Jahre dehnt sich das Universum um gerade mal 7% aus – so schnell ist die Raumausdehnung also wirklich nicht…

  17. @schlappohr:

    Mir ist der Sinn der mitbewegten Entfernung nicht ganz klar. Du sagst, sie gibt an, wie weit das Objekt *heute* entfernt ist, aber es gibt ja eigentlich keine universelle Zeit und damit auch kein heute, das hier und auf dem entfernten Objekt gleich ist.

    Das „heute“ bezieht sich natürlich auf unser eigenes Bezugssystem. Und in allen Bezugssystemen, die sich nicht bewegen (sprich: nur von der Ausdehnung des Universums „mitgetragen“ werden – das gilt in guter Näherung auch für uns), misst man übrigens dieselbe Zeit; insofern gibt es schon so etwas wie eine „universelle Zeit“.

    Die mitbewegte Entfernung ist meiner Ansicht nach die anschaulichste, das, was am ehesten dem entspricht, was man landläufig als „Entfernung“ bezeichnet. Stell‘ dir vor, du könntest die Ausdehnung des Universums so lange anhalten, wie du brauchst, um direkt nach zu messen (mit einem Meterstab oder ähnlichem… viel Spass), wie weit die Galaxie weg ist – dann bekommst du genau die mitbewegte Entfernung heraus!

    Wenn man die mitbewegte Entfernung betrachtet, dann sind die Objekte soweit von uns entfernt, dass ihr gegenwärtiger Zustand von uns nicht beobachtbar und damit nicht Teil unserer Realität ist. Welchen physikalischen Sinn ergibt dann die mitbewegte Entfernung?

    Dass die Objekte so weit von uns entfernt sind, dass ihr gegenwärtiger Zustand für uns nicht beobachtbar ist, liegt doch nicht an der Verwendung der mitbewegten Entfernung zur Entfernungsmessung, sondern sie sind schlicht so weit weg!

  18. SIe bewegt sich nicht aus eigenem Antrieb. Nur der Raum zwischen ihr und uns dehnt sich aus. Das hat das gleiche Ergebnis wie eine Rotverschiebung durch Doppler-Effekt. Aber die Galaxie bewegt sich nicht.

    Nicht, dass ich mich stur stellen will, aber das klingt als wäre das ein rein mathematischer Formalismus. Wir nehmen beide Galaxien als ruhend an, also müssen wir den Raum dazwischen als variabel annehmen, damit die Entfernungsänderung (genauer gesagt die Änderung einiger Entfernungstypen) zwischen ihnen erklärbar ist.

    Wenn ich Dein erstes „ja“ richtig deute, könnte man genausogut annehmen, der Raum wäre unveränderlich und die Galaxien würden sich auf eine bestimmte Weise relativ zueinander bewegen. Hätte das unterm Strich das gleiche Ergebnis oder gibt es da einen Unterschied, der sich mir gerade nicht erschließt?

    Denn sonst könnte man ja auch sagen, dass ich und der Krankenwagen still stehen und der Raum zwischen uns schrumpft und sich wieder ausdehnt. (Wenn man mal den Rest des Universums vernachlässigt.) Und dann könnte ich das tiefer werdende Tatütata plötzlich auch nicht mehr durch den Doppler-Effekt erklären.

    (Entschuldige die hartnäckige Fragerei, aber ich würde es tatsächlich gern verstehen. Interessanter Artikel, nebenbei bemerkt.)

  19. @Kallewirsch
    Ich denke du hast den Vergleich mißverstanden. Die Münzen repräsentieren Planeten, die Gummihaut den Raum. Wenn du den Raum dehnst entfernen sich die Münzen, bleiben selbst aber gleich.

    @FF
    Stimmt, ich war ein bißchen zu sehr auf die Gravitation fixiert weil du diese explizit erwähnt hast.

    @Bjoern
    Warum mir die Urknalltheorie suspekt ist ? Sie braucht immer sehr viel Klimmzüge und zusätzliche Erklärungen um stimmig zu sein. Eben der oben beschriebene Fall der Ausdehnung von Raum only oder auch die notwendige Inflation am Anfang, oder die Ungleichverteilung von Materie und Antimaterie oder fehlende Masse in Form dunkler Materie oder dunkle Energie oder … Da ist noch viel offen, deshalb klingelt mein Skeptikerbullshitdetektor, oder wie Florian öfter schreibt: Große Hypothesen müssen mit starken Argumenten untermauert werden. Hintergrundrauschen und Rotverschiebung erscheinen mir da etwas dünn. Versteh mich nicht falsch, die Theorie ist die Beste die wir haben aber wenns so viele Widersprüche gibt kanns nicht alles sein.

  20. @schak:

    Sie braucht immer sehr viel Klimmzüge und zusätzliche Erklärungen um stimmig zu sein. Eben der oben beschriebene Fall der Ausdehnung von Raum only …

    Das ist weder ein Klimmzug noch eine zusätzliche Erklärung, sondern schlicht eine direkte Folgerung aus grundlegender Physik. Ich weiss nicht, warum du da ein Problem siehst…?

    oder auch die notwendige Inflation am Anfang

    Ob die nötig ist, steht noch gar nicht fest – es gibt auch alternative Erklärungen.

    oder die Ungleichverteilung von Materie und Antimaterie

    Die ist kein Problem der Urknalltheorie, sondern sogar zu erwarten, wenn man ein wenig Teilchenphysik kennt – schon seit 50 Jahren ist bekannt, dass es CP-verletzende Zerfälle von Elementarteilchen gibt, und die führen unter den geeigneten Bedingungen automatisch zu einer Ungleichverteilung von Materie und Antimaterie!

    oder fehlende Masse in Form dunkler Materie oder dunkle Energie

    Und auch diese beiden sind keine Probleme der Urknalltheorie, sondern sogar zu erwarten, wenn man sich ein wenig in der Teilchenphysik auskennt!

    Große Hypothesen müssen mit starken Argumenten untermauert werden. Hintergrundrauschen und Rotverschiebung erscheinen mir da etwas dünn.

    Äh, es gibt noch weit mehr Belege für die Theorie (nicht „Hypothese“!) als nur diese beiden… schau‘ mal da rein (falls nötig, kann ich dir auch eine deutsche Version beschaffen):
    https://www.talkorigins.org/faqs/astronomy/bigbang.html

    aber wenns so viele Widersprüche gibt

    Nichts von dem, was du aufgezählt hast, ist ein „Widerspruch“! Was soll sich denn da widersprechen?!

  21. Hallo Florian,

    mal ne andere Frage:
    Wenn man in eine Richtung von der Erde aus schaut und eine Galaxie sieht, die 13 Lichtjahre von uns entfernt ist und dann in die genau entgegengesetzte Richtung schaut und auch eine Galaxie sieht, die 13 Lichtjahre von uns entfernt ist, sind diese beiden Galaxien dann 26 Lichtjahre voneinander entfernt?

    Wie ist das aber dann mit dem Alter des Universums von ca. 13 Milliarden Jahren vereinbar.

  22. Das Universum ist 13,7 Milliarden Jahre alt. Das Licht der fernsten Objekte wird also etwa 13 Milliarden Jahre bis zu uns gebraucht haben.

    Gibt es noch viel fernere Objekte, deren Licht uns nie erreicht?

  23. Jepp.
    Das passiert zum Beispiel dann, wenn sie so weit weg sind, daß sich der Raum zwischen uns und dort schneller ausdehnt als mit Lichtgeschwindigkeit. Das führt dann zu der paradox erscheinenden Situation, daß ein sich auf uns zu bewegender Lichtstrahl sich effektiv immer weiter entfernt.
    Was übrigens dann auch noch gleich den von Pirx vorhin angefragten Unterschied zwischen den sich auseinanderbewegenden Galaxien und der Ausdehnung des Raumes selber erklärt: da der Raum erstmal so keine Masse hat, kann er sich mit beliebigen Geschwindigkeiten ausbreiten. Galaxien schaffen das nicht. Die sog. „Inflation“ des Universums soll ja auch, wenn ich die Kosmologen da richtig verstanden habe, innerhalb einer Nanosekunde das ganze Ding auf mindestens ein Viertel der heute sichtbaren Ausdehnung aufgepustet haben.

  24. @Bullet & @FF:

    Was übrigens dann auch noch gleich den von Pirx vorhin angefragten Unterschied zwischen den sich auseinanderbewegenden Galaxien und der Ausdehnung des Raumes selber erklärt: da der Raum erstmal so keine Masse hat, kann er sich mit beliebigen Geschwindigkeiten ausbreiten. Galaxien schaffen das nicht.

    Okay, das ist ein Punkt, den ich tatsächlich nie so recht an dieser Expansionsgeschichte verstanden habe: Wie verhindert man bei einem sehr großen, schnell expandierenden Universum, dass sich weit voneinander entfernte Objekte mit mehr als Lichtgeschwindigkeit relativ zueinander bewegen. Anscheinend liegt die Antwort in der Unterscheidung zwischen Relativgeschwindigkeit zwischen sich bewegenden und der Raumdehnung zwischen ruhenden Objekten. Nur hilft mir die Masselosigkeit des Raumes noch nicht so recht dabei, den Unterschied zu verstehen. Denn auch wenn der sich dazwischen ausdehnende Raum masselos ist, haben die Galaxien immer noch eine Masse. Und wenn der Raum zwischen den Galaxien sich ausdehnt, dann resultiert daraus ja eine Relativbewegung der Galaxien. Weshalb kann man im Fall von Galaxien diese Relativbewegung als Ruhezustand bei sich ausdehnendem Raum interpretieren und beim Krankenwagen nicht? Sind das zwei unterschiedliche physikalische Phänomene, die sich eindeutig voneinander unterscheiden lassen?

  25. @Bullet
    Dann könnte das Universum aber auch unendlich sein und die ganze Urknalltheorie dadurch entstehen weil wir nur einen begrenzten Teil ’sehen‘ können. Vielleicht leben wir auch in einem schwarzen Loch und die 13,7 Milliarden Jahre sind der Schwarzschildradius.

    Ok, ich drifte ins Esoterische ab 🙂

  26. @Pirx:

    Und wenn der Raum zwischen den Galaxien sich ausdehnt, dann resultiert daraus ja eine Relativbewegung der Galaxien.

    Äh … eigentlich nicht. Das Beispiel mit dem aufgepusteten Luftballon kennst du ja bestimmt. Das böse daran ist: diese Relativbewegung ist sichtbar für jeden, der nicht auf dem Ballon steht. Aber die Punkte, die auf den Ballon gemalt wurden, sind (ballon-)ortsfest. Diese Relativbewegung der Galaxien wäre für einen Betrachter außerhalb unseres Universums möglicherweise tatsächlich sicht- und meßbar.
    Für uns als Beteiligte aber nicht.

    @schak:
    Die Auseinanderdrifterei ist aber keine Illusion. Ein unendliches Universum kann nicht auseinanderdriften. Wohin denn? 😉 Übrigens ist die Bezeichnung „Schwarzschildradius“ in diesem Fall zwar falsch – obwohl ich weiß, was du meinst – aber es gibt einen Terminus für die kosmologische Sichtbarkeitsgrenze. Ich müßte nur selbst kurz wikipieren, um das Wort zu finden…

  27. @Bullet:

    Naja, für jemanden, der auf dem Ballon steht, ist die Beobachtung sicher schwieriger, weil alle seine Bezugssysteme sich mit ausdehnen. Aber weshalb sollte sie prinzipiell unmöglich werden? Zumindest eine Rotverscheibung oder meinetwegen eine Tieftonverschiebung müsste er ja prinzipiell messen können. Wir können die Ausdehnung des Universums ja auch über die Rotverschiebung erkennen.

    Und von außen betrachtet führen die Punkte auf dem Ballon ja eine Relativbewegung zueinander aus. Dass der Ballonbewohner den Begrif Doppler-Effekt an dieser Stelle nicht verwendet und die Ortsveränderung aufgrund der Raumdehnung nicht mehr Bewegung nennt, scheint mir eine reine Konvention zu sein.

    (Wir hatten vor einiger Zeit schonmal eine ähnliche Diskussion: Dehnung [des Kontinuums] ist die Ableitung der Verschiebung [der Punkte im Kontinuum] nach dem Ort. Und ich behaupte kackfrech, dass eine Verschiebung einer Bewegung entspricht.)

  28. @Bjoern: Wenn ich mich nicht irre: Die Relativgeschwindigkeiten, die zwei Dinger, die z,B. einen Abstand von 90Grad auf einem sich konstant aufblähenden Ballon haben und diesen Abstand behalten, hängen nicht von ihrem Winkelabstand, sondern nur von der Radialgeschwindigkeit (Änderungsgeschwindigkeit des Radius) ab – ist also am ganzen Ballon gleich. Das Beispiel mit dem Ballon kann also beim Weltraum nicht funktionieren. Und natürlich können sich die Galaxien (oder beliebig Raumkörper) mit Überlichtgeschwindigkeit voneinander entfernen (haben sie ja schon), aber alle Wirkungen davon können wir nur mit Lichtgeschwindigkeit wahrnehmen. Trotzdem würde ich gerne wissen, wie schnell sich der Raum hier zwischen Heidelberg und Wien ausdehnt – pro Sekunde oder Jahr.

  29. @pirx und Überlichtgeschwindigkeit: So wie ich das verstanden habe bewegen sich Galaxien durchaus mit Überlichtgeschwindigkeit voneinander weg. Das sind die, welche jenseits des Sichtbarkeitshorizonts von ca. 45Mill. Lichtjahren sind. Man schätzt den wahren Radius des Universums glaube ich auf ungefähr das Doppelte dieser 45Mill. Lichtjahre. Das ist denke ich aber keine „Einstein-Verletzung“ da wir ja keine Information von diesen Galaxien bekommen können und die 300000km/s nur für Informationsübertragung gilt. Das Ballon-Beispiel von bullet verstehe ich auch nicht und sehe es so wie du. Aber ich bin da kein Experte.
    Aber wenn wir schon bei bullet sind ;-): Laut wiki war die Inflationszeit wesentlich kürzer als eine Nanosekunde, nämlich in der Größenordnung 10^-30s. Und das Universum war danach auch nur ca. 1m groß. Diese Größenordnungen habe ich auch aus anderen Quellen in Erinnerung.

  30. @Bjoern

    „Und in allen Bezugssystemen, die sich nicht bewegen […] misst man übrigens dieselbe Zeit“

    Nicht, wenn Du die Gravitation berücksichtigst.

    „Dass die Objekte so weit von uns entfernt sind, dass ihr gegenwärtiger Zustand für uns nicht beobachtbar ist, liegt doch nicht an der Verwendung der mitbewegten Entfernung zur Entfernungsmessung, sondern sie sind schlicht so weit weg!“

    Mit dieser Aussage legst Du implizit die mitbewegte Entfernung als „die richtige“ fest, aber genau „die richtige“ existiert nicht.

    Ich kann Deinen Standpunkt schon nachvollziehen: Um die Distanz weit entfernter Objekte zu bestimmen, muss man die Expansion einbeziehen und landet dann bei der mitbewegten Entfernung.

    Aber ein Objekt wird erst dann für uns existent, wenn wir es sehen (also allgemein seine Existenz nachweisen) können. Wenn uns das Licht eines Objektes zum ersten mal erreicht, beginnt es eine Rolle zu spielen. Deswegen meine ich, dass alle Metriken, die eine größere Entfernung als die Laufzeitdistanz liefern, keinen physikalischen Erkenntnisgewinn bringen.

  31. @Peter Rebernik

    Wenn ich mich nicht irre: Die Relativgeschwindigkeiten, die zwei Dinger, die z,B. einen Abstand von 90Grad auf einem sich konstant aufblähenden Ballon haben und diesen Abstand behalten, hängen nicht von ihrem Winkelabstand, sondern nur von der Radialgeschwindigkeit (Änderungsgeschwindigkeit des Radius) ab – ist also am ganzen Ballon gleich.

    Doch, Du irrst Dich, Du musst die Entfernung als Bogenlänge entlang der Ballonoberfläche messen, und die ist Radius * Winkel im Bogenmaß. Und natürlich nimmt ein größerer Bogen pro Zeiteinheit auch um absolut mehr zu als ein kleinerer Bogen, wenn r gleichmäßig wächst.

    @stillerleser

    Man schätzt den wahren Radius des Universums glaube ich auf ungefähr das Doppelte dieser 45Mill. Lichtjahre.

    Wo hast Du denn das her? Da das Weltall flach ist, schätzt man, dass es erheblich größer als der sichtbare Bereich bis zum kosmologischen Horizont ist, ich hab‘ da mal was vom 10^27-fachen gelesen, aber die ehrliche Antwort ist, man weiß es nicht. Wenn die Form eine Art Kugel oder Torus wäre, dann wäre jedenfalls dessen Krümmung im sichtbaren Teil des Universums so klein, dass der Bereich dahinter sehr viel größer als nur das doppelte sein müsste. Wobei auch die Möglichkeiten bestehen, dass das Universum komplett flach und unendlich ausgedehnt ist und es auch schon beim Urknall unendlich groß war.

  32. @schak

    @Kallewirsch
    Ich denke du hast den Vergleich mißverstanden. Die Münzen repräsentieren Planeten, die Gummihaut den Raum. Wenn du den Raum dehnst entfernen sich die Münzen, bleiben selbst aber gleich.

    Na ja. Die Frage war ja, warum werden dann Atome oder das Sonnensystem nicht größer? Eben weil der Vergleich hinkt. Elektronen und Protonen sitzen ja nicht nur im Raum wie auf einer Gummihaut die man dehnt, sondern sie sind ja auch noch durch die elektromagnetische Kräfte aneinander gekoppelt – eben einem Faden, der die beiden Münzen verbindet und verhindert, dass sie die Dehnung der Gummihaut mitmachen.

  33. @schak: „Sie braucht immer sehr viel Klimmzüge und zusätzliche Erklärungen um stimmig zu sein.“

    Auch wenn das manche immer glauben. Aber die Urknall-Kosmologie ist nichts, was man sich mal eben so ausgedacht hat, sondern etwas, was aus den Beobachtungen folgt.
    Auch die Inflation folgt direkt aus der Theorie und wurde „entdeckt“ und nicht „erfunden“. Ich empfehle zu diesem Thema das Buch von Alan Guth: https://www.amazon.de/gp/product/3426266180/ref=as_li_ss_tl?ie=UTF8&tag=astrodisimple-21&linkCode=as2&camp=1638&creative=19454&creativeASIN=3426266180

    https://www.scienceblogs.de/astrodicticum-simplex/2011/02/den-urknall-gab-es-wirklich-teil-1-wie-die-elemente-entstehen.php
    https://www.scienceblogs.de/astrodicticum-simplex/2011/02/den-urknall-gab-es-wirklich-teil-2-das-licht-aus-der-vergangenheit.php

  34. @Kallewirsch

    Was ist denn der Faden? Die Schwerkraft ist ja gerade kein Faden, der sich nicht mehr dehnt, wenn man ihn spannt. Wenn eine kleine Kraft das Weltall ausdehnt, dann ist das eine Kraft, die sich zur viel größeren, entgegengerichteten Gravitation addiert, aber sie müsste dann dennoch einen Einfluss haben und z.B. die Planetenbahnen vergrößern (bei den Atomen ist das anders, da sind ja nur gequantelte Orbitale erlaubt, die lassen sich nicht einfach mal um einen reellen Faktor vergrößern).

    Florian hatte in einem anderen Thread mal ein Paper verlinkt, dass der Effekt im Sonnensystem vorhanden sei, aber auch über Jahrmilliarden so klein, dass man ihn kaum nachweisen könne.

  35. @Alderamin: Vielen Dank; sehr guter Artikel. Laut Deinem Artikel beträgt die (als konstant angenommene) Raumausdehnung bei unserem nächsten Stern (4.5 Lj) ca. 10 cm pro Sekunde, also nur ca. 3000 km pro Jahr. Da das Licht ca. 4.5 Jahre braucht, wäre er jetzt (Gravitation und relative Geschwindigkeit vernachlässigt) 13.500 km weiter weg, also um einen Erddurchmesser. Der Alpha Centauri kommt allerdings mit 16 km/s auf uns zu, was ihn im Jahr um mehr als 500 Millionen km zu uns näher bringt. Vielen Dank nochmals.

  36. Die Schwerkraft ist ja gerade kein Faden, der sich nicht mehr dehnt, wenn man ihn spannt.

    Ja das hab ich mich auch schon gefragt.

    Was ich mich auch gefragt habe:
    Unser Meter ist ja jetzt auch indirekt mit irgendeiner Wellenlänge verknüpft (Cäsium irgendwas). Wenn sich jetzt das Universum ausdehnt, werden dann unsere Meterstäbe im Laufe der Zeit kürzer? Oder länger?

    Irgendwie ist das alles verwirrend. 🙂

  37. Hallo Florian und hallo an alle Anderen,

    Zur Info ich bin das was manche Professoren den Laien nennen auf dessen Niveau ihr euch jetzt heran geben müsst 🙂

    Ich verfolge dein Blog nun schon eine Weile und finde dass selbst ich vieles von dem kapiere was du schreibst. Und das wo ih in Physik in der Schule nur eine 3 hatte ^^

    Bei diesem Artikel stellt sich mir aber die Frage ob das Universum sich an den verschiednen Orten unterschiedlich stark ausweitet?

    Ih stelle mit das so vor dass unsere Galaxien Boote auf dem Ozean (Universu
    ) sind. Ihre Masse bleibt gleich (Fischfang, Wind und andere Effekte mal ausgenommen) und sie treiben auf dem Meer. Je nachdem wie stark die Strömung ist bewegt sich ein Boot schneller als das andere. Ist das so oder muss ich mit das anders vorstellen?

  38. @Peter Rebernik

    Ich glaube, der Dank gebührt jemand anderem, ich hatte ja keinen Artikel verlinkt, nur den von Florian erwähnt (wenn Du den meinst und den Link gefunden hast, kannst Du ihn ja nochmal posten).

    Ansonsten würde ich naiv gerechnet annehmen, die Hubble-Konstante ist in unserer Umgebung rund 70 km/s/Mpc, wobei 1 Mpc = 3,26 Millionen Lichtjahre sind, das macht also pro Lichtjahr 70000 m/s/3,26e+6 LJ = 0,0214 m/s/LJ = 2,14 cm/s/LJ.

    Die Entfernung von der Erde zur Sonne ist 1 AE, und 63000 AE sind ungefähr 1 LJ, d.h. da macht die Hubble-Konstante etwa 3,4e-7 m/s/AE aus. Nun warte aber mal 1 Milliarde Jahre oder 3,16e+16 s, dann wären es schon 10 Millionen km, und das wäre ein beträchtlicher Teil eines AE.

    Ich hab das Paper damals nicht gelesen, keine richtige Zeit gehabt, sollte das aber dringend mal nachholen, weil die zitierten Zahlen wesentlich kleiner waren, und die naive Rechnung nicht stimmen kann. Es ist nur halt so, dass auch ein kleiner Effekt von der Schwerkraft nicht „ausgeschaltet“ werden kann, das ist z.B. beim Yarkowski-Effekt ja ganz ähnlich. Wundert mich ein bisschen.

  39. @alderamin: Ich habe gerade nochmal im astrotreff nachgeschaut: Ja tatsächlich, die 2X44Mill. LJ werden als Mindestgröße des Universums angesehen und die tatsächliche Größe ist unbekannt.

  40. @Verena Weiß: „Ist das so oder muss ich mit das anders vorstellen? „

    Ne, die Expansion ist gleichförmig. Jede Galaxie im Universum entfernt sich von jeder anderen Galaxie und zwar um so schneller, je größer ihr Abstand ist. So wie im klassischen Beispiel vom Kuchenteig. Stell dir eine Schüssel mit Hefeteig vor, in dem sich Rosinen befinden. Der Teig ist das Universum, die Rosinen sind die Galaxien. Wenn der Teig aufgeht, dann entfernt sich jede Rosine von jeder anderen und das gleichmäßig.

  41. @Peter Rebernik:

    Wenn ich mich nicht irre: Die Relativgeschwindigkeiten, die zwei Dinger, die z,B. einen Abstand von 90Grad auf einem sich konstant aufblähenden Ballon haben und diesen Abstand behalten, hängen nicht von ihrem Winkelabstand, sondern nur von der Radialgeschwindigkeit (Änderungsgeschwindigkeit des Radius) ab – ist also am ganzen Ballon gleich.

    Ja, stimmt.

    Das Beispiel mit dem Ballon kann also beim Weltraum nicht funktionieren.

    Äh, wieso???

    Trotzdem würde ich gerne wissen, wie schnell sich der Raum hier zwischen Heidelberg und Wien ausdehnt – pro Sekunde oder Jahr.

    Ohne Kräfte dazwischen: um 7% pro Milliarde Jahre, wie oben schon erwähnt. Da die Erdkugel aber durch Kräfte zusammen gehalten wird, ändert sich der Abstand schlicht überhaupt nicht.

  42. @schlappohr:

    Nicht, wenn Du die Gravitation berücksichtigst.

    Auch die ist in den meisten Fällen vernachlässigbar, macht maximal ein paar Prozent aus.

    Mit dieser Aussage legst Du implizit die mitbewegte Entfernung als „die richtige“ fest,…

    Öh, nein, tue ich nicht.

    Für jede Galaxie kann man Koordinaten angeben (in unserem Bezugssystem – in anderen natürlich auch). Aus diesen Koordinaten kann man einerseits folgern, dass eine Galaxie so weit entfernt ist, dass ihr gegenwärtiger Zustand für uns nie beobachtbar wird; andererseits kann man daraus aber auch jedes beliebige der von Florian angegebenen Entfernungsmaße berechnen. Das grundlegende sind die Koordinaten, nicht die Entfernung!

    Um die Distanz weit entfernter Objekte zu bestimmen, muss man die Expansion einbeziehen und landet dann bei der mitbewegten Entfernung.

    Öh, inwiefern hat die mitbewegte Entfernung etwas damit zu tun, „die Expansion einzubeziehen“? Was verstehst du darunter, die Expansion *nicht* mit einzubeziehen?

    Deswegen meine ich, dass alle Metriken, die eine größere Entfernung als die Laufzeitdistanz liefern, keinen physikalischen Erkenntnisgewinn bringen.

    Mit Verlaub, aber das ergibt schlicht keinen Sinn. Nochmals: das grundlegende sind die Koordinaten, nicht die „Metriken“. Und eine Metrik ist doch auch nicht darauf ausgelegt, „physikalischen Erkenntnisgewinn“ zu bringen – sie ist schlicht ein Maß, nicht mehr und nicht weniger!

  43. @Alderamin:

    …dann wären es schon 10 Millionen km, und das wäre ein beträchtlicher Teil eines AE.

    Ja, nämlich ziemlich genau die 7%, die ich weiter oben schon erwähnt hatte…

  44. @Bjoern

    „Auch die [Gravitation] ist in den meisten Fällen vernachlässigbar, macht maximal ein paar Prozent aus. “

    Ja, aber diese paar Prozent vermasseln Dir die Existenz einer globalen (d.h. Universum-weiten) Zeitbasis. Es geht ja nicht um eine möglichst gute Abschätzung, sondern um die *physikalische Existenz* einer solchen Zeitbasis.

    „Für jede Galaxie kann man Koordinaten angeben“

    Dazu brauchst Du ein kosmisches Koordinatensystem, und das existiert ebensowenig wie eine Zeitbasis. Verschiedene Beobachter mit verschiedenen Bewegungszuständen an verschiedenen Orten werden immer verschiedene Messungen machen, ohne dass einer davon global gesehen Recht oder Unrecht hätte. Relativitätstheorie.

    „Das grundlegende sind die Koordinaten, nicht die Entfernung!“

    Wenn das so wäre, dann wäre Florians Artikel hinfällig. Ein Koordinatensystem ermöglicht eine eindeutige Entfernungsmessung, indem man feste Intervalle auf den Koordinatenachsen abzählt (im kartesischen Fall, ansonsten Winkel oder was auch immer). Dann kann ein Objekt nicht nicht gleichzeitig 13 GLy *und* 30GLy entfernt sein. Entweder eines davon ist falsch, oder es gibt mehrere, unterschiedliche, aber gleichermaßen richtige Koordinatensysteme.

    “ Was verstehst du darunter, die Expansion *nicht* mit einzubeziehen? “

    Ich verstehe darunter, die Rotverschiebung zu messen, daraus auf die Entfernung des Objekts zum Zeitpunkt der Aussendung des Lichtes zu schließen und die Distanz zu ignorieren, die das Objekts seitdem aufgrund der Expansion des Universums hinzugewonnen hat.
    Zugegeben ist meine Einschränkung nicht ganz richtig, da die Rotverschiebung bereits die Expansion impliziert.

    „Und eine Metrik ist doch auch nicht darauf ausgelegt, „physikalischen Erkenntnisgewinn“ zu bringen – sie ist schlicht ein Maß, nicht mehr und nicht weniger!“

    Wozu eine Metrik definieren und damit Messungen durchführen, wenn daraus kein Nutzen gezogen werden kann? Astronomen haben ein Koordinatensystem im Sonnensystem definiert, um z.B. die Planetenbahnen
    zu vermessen. Das hat dann z.B. zur Entdeckung des Neptun (oder Uranus?) geführt. Das nenne ich durchaus einen Erkenntnisgewinn.

    Wenn ich nun eine Entfernungsangabe für ein Objekt habe, das so weit entfernt ist, dass es innerhalb der nächsten 30 Gyr *hier* keine Wirkungen verursachen kann, dann bringt mir diese Entfernungsangabe keine Erkenntnisse über die aktuelle Struktur des Universums. Vielleicht hat das Objekt durch Protonenzerfall einfach aufgehört zu existieren, vielleicht ist es zu einem Schwarzen Loch oder Quasar geworden oder etwas anderes, das noch niemand gesehen hat. Was auch immer, es hat keine Bedeutung. Es haben nur Dinge Bedeutung, über die wir Informationen gewinnen können, und die müssen innerhalb unseres Vergangenheitslichtkegels liegen, und damit inerhalb der der Laufzeitdistanz.

  45. @Bjoern

    Ja, nämlich ziemlich genau die 7%, die ich weiter oben schon erwähnt hatte…

    Schön, dann hab‘ ich (naiv) ja richtig gerechnet. Leider aber nicht wirklich, ich hab das oben erwähnte Paper wiedergefunden. Dort wird mit Formeln, die nachzuvollziehen ich leider zu doof bin, vorgerechnet, dass

    – Die Gravitationsbeschleunigung der Hubble-Expansion über 1AE etwa 3,17e-47 m/s^2 ist und damit 44 Größenordnungen kleiner als die der Sonne auf die Erde,
    – sich die Erdbahn über die Lebensdauer des Sonnensystems dadurch nur um ein 10e24tel vergrößert, was wesentlich weniger ist als z.B. durch den Massenverlust der Sonne durch den Sonnenwind.

    Wie gesagt, leider verstehe ich die Formeln nicht, aber ich vertraue da mal den Fachleuten, die übrigens anscheinend die ersten sind, die das erst 1998 mal durchgerechnet haben. Da war die Dunkle Energie gerade frisch entdeckt. Also offenbar keine blöde Frage, warum das Sonnensystem nicht auch 7% pro Jahrmilliarde wächst.

  46. Florian Freistetter schrieb (03.01.12 · 11:00 Uhr):
    > DL ist die Leuchtkraftdistanz, DC die mitbewegte Distanz, DT die Laufzeitentfernung und DA die Winkeldurchmesserentfernung. […]
    > Wenn es nur um kleine Entfernungen geht, dann liefern alle vier Entfernungsskalen die gleichen Werte. Erst bei kosmologischen Distanzen gibt es Unterschiede

    Paare von Beteiligten, die untereinander „z == 0“ messen, erhalten auch den gleichen Wert „DC == DT“ (entsprechend den Definitionen dieser Messgrößen), egal wie groß oder klein oder „komologisch“ dieser Wert ist.

    Andererseits sind für Paare von Beteiligten, die untereinander „z =/= 0“ messen, dazu die Größen „DC“ oder „DT“ gar nicht kompatibel;
    vielmehr eignen sich ihr „Stoßparameter“ und ihre „Dauern seit Erreichen (bzw. bis zum Erreichen) der Stoßparameter-Entfernung voneinander“ zur Charakterisierung und Unterscheidung der geometrischen bzw. kinematischen Beziehungen verschiedener solcher Paare.

  47. @schlappohr:

    Ja, aber diese paar Prozent vermasseln Dir die Existenz einer globalen (d.h. Universum-weiten) Zeitbasis. Es geht ja nicht um eine möglichst gute Abschätzung, sondern um die *physikalische Existenz* einer solchen Zeitbasis.

    Ich habe keine Ahnung, was „physikalische Existenz einer Zeitbasis“ überhaupt heissen soll.

    Fakt ist: in alle mitbewegten Koordinatensystemen, in denen gravitative Einflüsse praktisch vernachlässigbar sind (und das gilt sicher zumindest in den voids), ist seit dem Urknall genau dieselbe Zeit vergangen. In Bezugssystem Erde ist es zwar nicht exakt dieselbe Zeit, aber der Unterschied ist vernachlässigbar gering. (selbst „ein paar Prozent“ war sicher noch deutlich zu hoch geschätzt)

    Dazu brauchst Du ein kosmisches Koordinatensystem, und das existiert ebensowenig wie eine Zeitbasis.

    Äh, schon mal was von der Robertson-Walker-Metrik gehört…? Was liegt der denn zu Grunde, wenn nicht ein „kosmisches Koordinatensystem…?

    Verschiedene Beobachter mit verschiedenen Bewegungszuständen an verschiedenen Orten werden immer verschiedene Messungen machen, ohne dass einer davon global gesehen Recht oder Unrecht hätte. Relativitätstheorie.

    Das bestreitet doch auch keiner! Das einzige, was ich sage, ist, dass eben mitbewegte Koordinatensystem so etwas wie eine „universelle Zeit“ liefern können… das hat doch nix mit „Recht oder Unrecht“ zu tun.

    Wenn das so wäre, dann wäre Florians Artikel hinfällig. Ein Koordinatensystem ermöglicht eine eindeutige Entfernungsmessung, indem man feste Intervalle auf den Koordinatenachsen abzählt (im kartesischen Fall, ansonsten Winkel oder was auch immer).

    So, dann rat‘ mal, was die „mitbewegte Entfernung“ ist…

    Dann kann ein Objekt nicht nicht gleichzeitig 13 GLy *und* 30GLy entfernt sein.

    Kein Mensch behauptet, dass ein Objekt gleichzeitig 13 und 30 GLy entfernt ist! Hast du denn Florian Artikel überhaupt nicht verstanden?!?

    “ Was verstehst du darunter, die Expansion *nicht* mit einzubeziehen? “ Ich verstehe darunter, die Rotverschiebung zu messen, daraus auf die Entfernung des Objekts zum Zeitpunkt der Aussendung des Lichtes zu schließen und die Distanz zu ignorieren, die das Objekts seitdem aufgrund der Expansion des Universums hinzugewonnen hat.

    O.k., das wäre noch ein zusätzliches, von Florian nicht erwähntes Entfernungsmaß – das aber reichlich wenig Sinn ergibt. (übrigens: wenn du bestreitest, dass „heutige Entfernung eines Objekts“ sinnvoll definiert ist, dann ist „Entfernung des Objekts zum Zeitpunkt der Aussendung des Lichts“ genausowenig definiert…)

    „Und eine Metrik ist doch auch nicht darauf ausgelegt, „physikalischen Erkenntnisgewinn“ zu bringen – sie ist schlicht ein Maß, nicht mehr und nicht weniger!“ Wozu eine Metrik definieren und damit Messungen durchführen, wenn daraus kein Nutzen gezogen werden kann?

    Äh, „Nutzen“ und „physikalischer Erkenntnisgewinn“ ist doch nicht dasselbe?!?

    Wenn ich nun eine Entfernungsangabe für ein Objekt habe, das so weit entfernt ist, dass es innerhalb der nächsten 30 Gyr *hier* keine Wirkungen verursachen kann, dann bringt mir diese Entfernungsangabe keine Erkenntnisse über die aktuelle Struktur des Universums.

    In der Tat nicht. Und??? Deswegen kann man doch diese Entfernung trotzdem angeben?!? Ich verstehe dein Problem echt nicht!

    Was auch immer, es hat keine Bedeutung. Es haben nur Dinge Bedeutung, über die wir Informationen gewinnen können, …

    Wow. Sehr „interessante“ Weltsicht…

  48. So wie ich das verstanden habe bewegen sich Galaxien durchaus mit Überlichtgeschwindigkeit voneinander weg. Das sind die, welche jenseits des Sichtbarkeitshorizonts von ca. 45Mill. Lichtjahren sind. Man schätzt den wahren Radius des Universums glaube ich auf ungefähr das Doppelte dieser 45Mill. Lichtjahre. Das ist denke ich aber keine „Einstein-Verletzung“ da wir ja keine Information von diesen Galaxien bekommen können und die 300000km/s nur für Informationsübertragung gilt.

    Ich gebe zu ich bin auch nur ein Laie in Physikalischen Dingen, aber meiner Meinung nach gibt es eine andere Begründung dafür, dass es keine „Einsteinverletzung“ ist. Diese begründung wäre folgende: die ganzen Aussagen wie höchstens mit Lichtgeschwindigkeit, oder auch die relativistische Massenzunahme beziehen sich auf Dinge die sich bewegen, also eine von 0 verschiedene Geschwindigkeit haben. Geschwindigkeit im physikalischen Sinne ist, wenn ich mich richtig erinnere definiert, als die Ableitung des Ortes nach der Zeit.Wenn jetzt also Galaxien durch die Raumausdehnung immer weiter entfernt sind, dann bewegen sie sich nicht, weil für jede dieser Galaxien gilt, dass sie am selben Ort bleibt, also Geschwindigkeit Impuls etc., die aus der Raumausdehnung resultieren 0 sind. Und wenn sich 2 Objekte mit einer Geschwindigkeit von 0 auseinanderbewegen, dann hat auch Einstein da nichts dagegen.

  49. @MartinB:

    Danke für den Link. Dein Artikel macht einiges klarer. Trotzdem tue ich mich noch schwer mit dem Unterschied zwischen Bewegung und Expansion. Dumme Frage dazu: Ändert sich die Lichtgeschwindigkeit mit der Expansion?

  50. @Oberclown:
    Exakt. Die Ableitung des Ortes nach der Zeit ist für diesen Prozess gleich 0, da keine der Galaxien eine Ortsveränderung erfährt. Es kommt nur einfach „mehr Ort“ hinzu.

    @Bjoern:

    Was auch immer, es hat keine Bedeutung. Es haben nur Dinge Bedeutung, über die wir Informationen gewinnen können, …Wow. Sehr „interessante“ Weltsicht…

    Warum? Diese Weltsicht ist doch völlig stimmig. Was hinter dem Horizont des Vergangenheitslichkegels liegt kann (durch die Bindung an die Lichtgschwindigkeit) nie mit uns interagieren (Informationsaustausch). Es ist also für unsere Realität volkommen irrelevant.

    Gruß Hawk

  51. @Bjoern
    Könnte man sich das so vorstellen:
    Der Raum dehnt sich zwar auch in atomaren und Planetendistanzen aus, aber die 4 Grundkräfte halten die Materie auf gleicher Distanz ?
    Das würde ja dann umgekehrt bedeuten, dass sich alle Planeten langsam in Richtung Sonne bewegen um die Expansion der Universums auszugleichen ?

  52. @schak: „Der Raum dehnt sich zwar auch in atomaren und Planetendistanzen aus, aber die 4 Grundkräfte halten die Materie auf gleicher Distanz ?“

    Stells dir eher wie ein Tauziehen vor. Die Expansion will die Atome deines Körpers auseinander ziehen. Die elektromagnetischen/Kernkräfte ziehen dagegen. Sie sind stärker, also bleibt dein Körper so wie er ist.

  53. @schak

    Nein. Die Kraft der Expansion ist da, sie vergrößert die Bahnen der Planeten, aber nur um einen winzigen Dreckeffekt. Und für kleinere Objekte spielt sie noch weniger eine Rolle.

  54. @Bjoern

    „Ich habe keine Ahnung, was „physikalische Existenz einer Zeitbasis“ überhaupt heissen soll.“

    Ich meine damit, dass keine Möglichkeit besteht, eine universum-weite Zeit anzugeben, eine kosmische Uhr, die überall zur gleichzeitig und gleich schnell tickt. Auch wenn die Gravitation nur „ein paar Prozent“ ausmacht, verbietet sie
    eben die Existenz dieser Uhr. Sicher kannst Du die Gravitation näherungsweise
    vernachlässigen und bekommst Ergebnisse, die halbwegs stimmen, aber eben keine exakte theoretische Beschreibung zulassen. Die Gravitation ist real, und
    die globale Zeit existiert deswegen nicht.

    „Äh, schon mal was von der Robertson-Walker-Metrik gehört…?“

    Nein, aber ich habe es gerade auf Wiki nachgelesen, ohne den Hintergrund
    genau zu verstehen. Aber auch das ist eine erste Näherung, eine Vereinfachnung, die z.B. absolute Homogenität voraussetzt. Du musst explizit Störungen einrechnen, um zu einem realistischen Modell zu kommen. Man kann gut damit rechnen, aber ein globales Koordinatensystem definiert diese Metrik auch nur näherungsweise.

    „Kein Mensch behauptet, dass ein Objekt gleichzeitig 13 und 30 GLy entfernt ist! Hast du denn Florian Artikel überhaupt nicht verstanden?!?“

    Um sicher zu gehen, habe ich den Artikel nochmal gelesen. Dort heißt es:

    „Das Licht von UDFy-38135539 war 13,1 Milliarden Jahre zu uns unterwegs.
    Die mitbewegte Entfernung beträgt knapp 30 Milliarden Lichtjahre. Von ihrer Größe her erscheint uns die Galaxie als wäre sie nur 3 Milliarden Lichtjahre entfernt, ihre Helligkeit entspricht allerdings einer 290 Milliarden Lichtjahre entfernte Galaxien […]“

    Welche dieser Entfernungen ist denn „die richtige“? Die mitbewegte? Und warum? In einem Kommentar oben schreibt Florian:

    „Es gibt nicht DIE Entfernung. Es gibt verschiedene Arten das Wort „Entfernung“ zu definieren. Die Galaxie sieht aus wie eine, die 290 Milliarden LJ weit weg ist. Das heißt nicht, dass dies ihre tatsächliche Entfernung ist.
    Es gibt keine „tatsächliche Entfernung“. “

    Dieser Satz beschreibt den Kern des Problems eigentlich sehr gut.

    „O.k., das wäre noch ein zusätzliches, von Florian nicht erwähntes Entfernungsmaß“

    Ich dachte, das wäre genau die Laufzeitdistanz. Meine Messung basiert auf Licht, dass vor langer Zeit ausgesendet wurde (etwas anderes steht mir nicht zur Verfügung für meine Messung). Was das Objekt seitdem getan hat,
    kann ich nicht wissen und ignoriere es deshalb.

    „(übrigens: wenn du bestreitest, dass „heutige Entfernung eines Objekts“ sinnvoll definiert ist, dann ist „Entfernung des Objekts zum Zeitpunkt der Aussendung des Lichts“ genausowenig definiert…)“

    So ist es.

    „Äh, „Nutzen“ und „physikalischer Erkenntnisgewinn“ ist doch nicht asselbe?!? “

    Für Kosmologen sind diese beiden Begriffe identisch. Man definiert ja
    keine Matriken aus langer Weile. Ich kann sicher die Entfernung einer Galaxie
    bestimmen, wenn es mir Spaß macht. Aber ein Astronom oder Kosmologe will schließlich herausfinden, wie diese Welt funktioniert.

    „In der Tat nicht. Und??? Deswegen kann man doch diese Entfernung trotzdem angeben?!? Ich verstehe dein Problem echt nicht!“

    Florian hat vier Entfernungsmaße vorgestellt, die um Zehnerpotenzen unerschiedliche Distanzen liefern und trotzdem alle richtig sind. Das ist soweit klar, wenn man akzeptiert, das auf kosmischen Skalen der klassische Begriff der Entfernung schlichtweg nicht existiert, man muss diesen hier eben geeignet definieren, und dazu gibt es verschiedene Ansätze. Sind wir uns soweit einig?

    Da keiner dieser Ansätze „richtig“ ist, habe ich mich gefragt, welcher davon
    vielleicht „sinnvoll“ ist. Als sinnvoll bezeichne ich Entfernungsmaße, die einem
    {Astronom, Kosmologen, Physiker} erlauben, eine Erkenntnis über das Universum zu erlangen, die er ohne dieses Entfernungsmaß nicht erlangen könnte.

    Nun meine ich, dass Erkenntnisse nur gewonnen werden können aus Objekten, die im sichtabren Teil des Universums liegen, deren Licht, Neutrinos, Gravitation oder was auch immer hier in unseren Instrumenten landen kann.

    Wenn ich nun ein Entfernungsmaß habe, das mir ein Objekt außerhalb des Beobachtungshorizonts platziert, dann bringt mir diese Information überhaupt keinen Gewinn, da das besagte Objekt keinerlei Information auf den hiesigen Teil des Universums überträgt. Wohl oder übel muss ich also meine Überlegungen ohne das besagte Objekt anstellen.

    Mein Problem besteht darin, den Sinn in einem Entfernungsmaß zu finden, das eben keine verwertbaren Informationen liefert.

  55. Soweit ich es bisher verstehe wirken Kräfte wie Gravitation und die Kräfte in den Elementarteilchen der Expansion (genauer: einer Vergrößerung in Folge der Expansion) entgegen. Ich habe auch verstanden, dass die Ausdehnung des Raumes etwas anderes als eine homogene Relativbewegung ist. So weit so gut.

    Nun denke ich mir folgendes: Die Proportionen in Elementarteilchen oder auch Planetensystemen ergeben sich aus einem Kräftegleichgewicht. Es gibt bestimmte Abstände, in denen sich die Kräfte (Gravitation-Zentrifugalkraft oder die entsprechenden Kräfte im Atom) ausgleichen. Aber warum verändern sich diese Gleichgewichtsabstände nicht, wenn sich der Raum als solcher ausdehnt? Warum müssen diese Abstände nicht entsprechend größer bzw. die Rotationsgeschwindigkeiten entsprechend höher werden?

  56. @Alderamin

    Danke für den Hinweis auf das Paper. Kannte ich noch nicht.
    (Das ist aber doch nicht so schwer?)

    Das waren aber nicht die Ersten die sich damit beschäftigt haben, zu diesem Thema gibt es ziemlich viele und auch ziemlich frühe Veröffentlichungen.
    In diesem ziemlich aktuellen Paper https://arxiv.org/abs/0810.2712 wird ein Überblick über Betrachtungen zu diesem Thema gegeben. Bei den Literaturangaben sind dann auch fast alle vorherigen Veröffentlichungen gesammelt.
    Das Fazit dieses Papers ist aber leider folgendes:

    However, the general problem of gaining a qualitative and quantitative understanding of how the cosmological dynamics influences local systems remains challenging, with only partial clues being so far provided by exact solutions to the field equations of General Relativity.

    Das Problem ist, dass man allgemein im Rahmen der ART rechnen muss. Das ist bisher nur für sehr seltsame Speziallfmetriken gelungen, wobei diese Ergebnisse kaum auf unsere Metrik übertragen werden können.
    Bei dem von dir verlinkten Paper machen es sich die Autoren sehr einfach, deswegen können sie Zahlenwerte angeben.
    Sie betrachten einfach das Netwonsche Zwei-Körper-Problem. Die Expansion beschreiben sie (wie Florian im Post direkt über diesem) als Kraft, die die Körper auseinander ziehen will.
    Diese Kraft wirkt der zusammenhaltenden Gravitationskraft entgegen und der Radius wird etwas größer als in einem Zwei-Körper-Problem ohne Expansion des Universums.
    Anschließend rechnen sie dann mehr oder weniger mit klassischer Mechanik weiter.
    Das Problem ist allerdings, dass auf Körper, die sich mit dem Hubble-Fluss bewegen, der ART zufolge dadurch keine Kräfte wirken.

    Um die wirkende Kraft zu ermitteln verwenden die Autoren R ‚ (t) = H(t) * R(t), also die lineare Hubble-Beziehung. (H ist der Hubble-Parameter, R der Abstand usw.)
    Durch Ableiten bekommt man die Beschleunigung R ‚ ‚ (t) = H ‚ (t) * R(t) + H(t) * R ‚ (t).
    Wenn man jetzt die Definition des Hubble-Parameters als H = a ‚ (t) / a(t) einsetzt, bekommt man als Kraft, die die Körper auseinander zieht, F = m * R ‚ ‚ (t) = m * R(t)* a ‚ ‚ (t)/a(t). [a(t) ist der Skalenfaktor]
    Diese Kraft wirkt der Gravitationskraft entgegen.
    Das setzen sie jetzt einfach als äußere Störung ins newtonsche Zwei-Körper-Problem ein.
    Anschließend wird weiteren vereinfacht und schließlich kommt eine ziemlich simple Formel raus.
    Allerdings ist die ART nichtlinear, so eine Aufspaltung in einen äußeren Störungsterm ist also keinesfalls erlaubt.

    Wobei durch diese Ansätze und während des Vereinfachungs-Prozesses derart viele Fehler gemacht werden, dass das errechnete Ergebnis meiner Meinung nach nicht mehr viel wert ist.
    Das kann um ziemlich viele Größenordnungen falsch sein.

    Wenn man beim Newtonschen Ansatz anders vorgeht, bekommt man übrigens auch andere Ergebnisse. Aus einem anderen Paper:

    As an example, the deviation in the radius for an hypothetical spacecraft orbiting around the Sun at 100 AU would be just of the order of 1 mm. Since it grows with the fourth power of the distance, the deviation at 1000 AU would be of the order of 10 meters.

    Dieses Ergebnis ist meiner Meinung nach aus ähnlichen Gründen nicht besonders brauchbar.
    Allgemein ist man sich eigentlich nur einig, dass die Expansion auf kleinen Skalen keine messbaren Auswirkungen hat.

  57. @Niels

    (Das ist aber doch nicht so schwer?)

    Mit Deinen Erläuterungen (vielen Dank!) nicht, aber die Texte über Kosmologie, die ich so lese, vermeiden Formeln wie die Pest, ich bin da nicht so bewandert.

    Das Problem ist, dass man allgemein im Rahmen der ART rechnen muss. Das ist bisher nur für sehr seltsame Speziallfmetriken gelungen, wobei diese Ergebnisse kaum auf unsere Metrik übertragen werden können.

    Wer hätte gedacht, dass das Problem so komplex ist?
    Dabei heißt es doch, die Dunkle Energie sei einfach eine kosmologische Konstante, die als konstanter Term in der Einsteinschen Feldgleichung drinnen steht.

    Das Problem ist allerdings, dass auf Körper, die sich mit dem Hubble-Fluss bewegen, der ART zufolge dadurch keine Kräfte wirken.

    Aber ist denn der Prozess von gänzlich anderer Art, als der bei der kosmischen Inflation? Da bewirkt doch ein negativer Druck eine negative Gravitation, d.h. eine Beschleunigung. Ich hätte vermutet, das ist bei der Dunklen Energie das gleiche, nur eben mit sehr viel kleinerem Wert.

    Allgemein ist man sich eigentlich nur einig, dass die Expansion auf kleinen Skalen keine messbaren Auswirkungen hat.

    Damit sind wir so klug wie zuvor 🙁 Bei weit entfernten Objekten schafft es die Dunkle Energie, deren Abstand gegen die wechselseitige Gravitation um 7% pro Jahrmilliarde zu vergrößern, bei nahen Objekten und größerer Gravitation ist kein Effekt messbar, das ist irgendwie nicht ganz stimmig. Wo ist der Übergang zwischen beiden Fällen und warum gibt es ihn?

  58. @Alderamin:
    die Konstante sagt schon etwas aus, und du hast sie selber zitiert:

    rund 70 km/s/Mpc,

    Das heißt, daß alles, was weniger als etwa 2 Mpc groß ist, von derlei Nebensächlichkeiten wie der Hubble-Expansion intern verschont wird. (Denn 2 Mpc bedeutet, daß die Ränder eines solchen Konglomerates sich durch jene Expansion mit etwa 140 km/s voneinander entfernen – eine ziemlich hohe Geschwindigkeit für einen einzelnen Stern, aber weit von den verschiedenen kosmischen Geschwindigkeiten entfernt, die ein „Objekt“ wie eine Galaxie zerstören würden.
    Zum Vergleich: um die Milchstraße zu verlassen, brauchst du lt. Wikiwiki 320 km/s.)
    Und nun find mal ein Objekt dieser Ausdehnung. Galaxienhaufen vielleicht – aber sind das noch „Objekte“?
    Ach ja, und 2 Mpc sind etwa 6,5 Millionen Lichtjahre. Ein einzelnes popeliges Lichtjahr Differenz in der Entfernung wäre tief im Meßrauschen versteckt. Wer „Mpc“ verwendet, darf unter kly nicht mit Meßgenauigkeit ankommen.

  59. @Bullet

    Das ist nur der Hubble-Parameter, den kann ich in beliebigen Einheiten angeben. Tatsächlich steht in dem von mir verlinkten Paper, dass die Hubble-Expansion unterhalb von nicht weniger als 200 MPc keine Rolle mehr spielt. Was aber (laut dem Paper) nicht bedeutet, dass gar keine Expansion stattfindet. Die Autoren verneinen die Aussage, dass es eine Untergrenze gibt, unterhalb der einfach keine Hubble-Expansion mehr da sei.

    Ich hab‘ auch nochmal im Wiki-Artikel zur AR nachgeschaut, und da steht, dass die kosmologische Konstante (genau wie die Inflation) als „perfektes Fluid mit negativem Druck“ dargestellt werden kann. Dann wäre sie eine abstoßende Gravitation und mithin eine Kraft, die der Gravitationsanziehung zwischen Massen entgegenwirkt. Die kann auf kleinem Raum eigentlich nicht einfach verschwinden., bestenfalls unbedeutend werden. Aber wie gesagt, der Yarkovski-Effekt ist auch klein und hat trotzdem über lange Zeiten eine messbare Auswirkung. Das ganze bleibt irgendwie mysteriös.

    Ich nehme an, die Jungs in dem Paper haben sich vielleicht, wie Niels sagt, um ein paar Größenordnungen verrechnet, aber da ist so viel Platz bis zur Messbarkeit, dass der Effekt trotzdem bis zur Größe von Galaxienhaufen vernachlässigbar ist bzw. im Rauschen der Eigenbewegungen der Galaxien verschwindet.

  60. @Alderamin
    Du verwechselt Expansion mit dunkler Energie.
    Die dunkle Energie ist nicht die Ursache der Expansion des Universums!
    Wenn unser Universum überhaupt keine dunkle Energie enthalten würde, würde es trotzdem expandieren. Nur eben nicht beschleunigt.
    Für das aktuelle Thema ist die dunkle Energie also erst einmal ziemlich unwichtig.
    In dem von dir verlinktem Paper wurden sogar Vereinfachungen für ein materie-dominiertes Universum gemacht, die dunkle Energie wurde dort also nicht berücksichtigt.
    Wie gesagt, viele Vereinfachungen.

    Wer hätte gedacht, dass das Problem so komplex ist?
    Dabei heißt es doch, die Dunkle Energie sei einfach eine kosmologische Konstante, die als konstanter Term in der Einsteinschen Feldgleichung drinnen steht.

    Das Problem wird ja auch dadurch komplex, dass man eine passende Lösung der Einsteinschen Feldgleichungen finden muss. Nicht durch die kosmologische Konstante.

    Aber ist denn der Prozess von gänzlich anderer Art, als der bei der kosmischen Inflation? Da bewirkt doch ein negativer Druck eine negative Gravitation, d.h. eine Beschleunigung. Ich hätte vermutet, das ist bei der Dunklen Energie das gleiche, nur eben mit sehr viel kleinerem Wert.

    Inflatonfeld und dunkle Energie führen jeweils zu Gleichungen mit negativem Druck.
    Insofern ähneln sie sich. Das ist aber auch kein Wunder: beide wurden eingeführt, um das selbe zu erklären, nämlich eine schnellere Expansion des Universums als jeweils ohne ihr Vorhandensein.
    Das hast du schon richtig verstanden.

    (Ob Inflation und dunkle Energie aber irgendwie aufeinander beruhen haben ist meines Wissens nicht geklärt.)

    Beides hat aber wie schon geschrieben erst einmal nichts mit unserem Problem zu tun.
    Der negative Druck wirkt nicht auf Objekte im Raum. Inflation und negative Energie beschleunigen schließlich nicht die Objekte, sondern die Ausdehnung des Raums.
    Die Expansion des Universums führt nach der ART nie zu Kräften auf die Objekte im Raum, egal ob die Expansion beschleunigt, gebremst oder sonstwie verläuft.

    Damit sind wir so klug wie zuvor 🙁 Bei weit entfernten Objekten schafft es die Dunkle Energie, deren Abstand gegen die wechselseitige Gravitation um 7% pro Jahrmilliarde zu vergrößern, bei nahen Objekten und größerer Gravitation ist kein Effekt messbar, das ist irgendwie nicht ganz stimmig.

    Die 7% kommen daher, dass sich momentan der Skalenfaktor um 7% in einer Milliarde Jahre vergrößert.
    Das ist genau das, was man Expansion des Universums nennt.
    Man könnte auch sagen, die Universum wächst um 7%. (bzw. das Volumen nimmt um 23% zu.)
    Früher hat sich das Universum prozentual aber deutlich schneller vergrößert und in Zukunft wird es sich prozentual langsamer vergrößern.

    Der Abstand zwischen zwei Objekten wächst aber nur dann genau wie der Skalenfaktor, wenn zwischen den Objekten überhaupt keine Kräft wirken.

    Wobei es natürlich keinen wundersame Grenze gibt, ab der die Expansion plötzlich keine Rolle mehr spielt, sondern einen fließenden Übergang.
    Bzw. nach dem von mir oben verlinkten Paper in der newtonschen Näherung:
    it grows with the fourth power of the distance
    Aber wie im anderen Zitat steht:
    gaining a qualitative and quantitative understanding of how the cosmological dynamics influences local systems remains challenging

    Tatsächlich steht in dem von mir verlinkten Paper, dass die Hubble-Expansion unterhalb von nicht weniger als 200 MPc keine Rolle mehr spielt. Was aber (laut dem Paper) nicht bedeutet, dass gar keine Expansion stattfindet. Die Autoren verneinen die Aussage, dass es eine Untergrenze gibt, unterhalb der einfach keine Hubble-Expansion mehr da sei.

    Das ist keine Idee der Autoren, dass ist die Meinung aller Kosmologen.
    Nach dem Urknallmodell expandiert jedes infinitesimale Raumvolumen um den selben Wert.
    Dadurch ensteht genau die Hubble-Expansion in dieser linearen Form.
    Das durch dein Zimmer begrenzte Volumen expandiert ganz genau so wie das selbe Volumen in 10 Milliarden Lichtjahren Entfernung. Das ist die Kernidee das Urknallmodells.

    Ich nehme an, die Jungs in dem Paper haben sich vielleicht, wie Niels sagt, um ein paar Größenordnungen verrechnet, aber da ist so viel Platz bis zur Messbarkeit, dass der Effekt trotzdem bis zur Größe von Galaxienhaufen vernachlässigbar ist bzw. im Rauschen der Eigenbewegungen der Galaxien verschwindet.

    Na ja, dass dieser Effekt auf kleinen Skalen vernachlässigbar ist, ist ja schon lange klar.
    Das Problem bei ihrer Rechnung ist eigentlich, dass man nicht einmal abschätzen kann, um wie viele Größenordnungen die Rechnung falsch ist.
    Bzw. die Beobachtungen liefern natürlich eine Grenze, aber dann bräuchte man auch die Rechnung nicht.

  61. @schlappohr:

    Ich meine damit, dass keine Möglichkeit besteht, eine universum-weite Zeit anzugeben, eine kosmische Uhr, die überall zur gleichzeitig und gleich schnell tickt. Auch wenn die Gravitation nur „ein paar Prozent“ ausmacht, verbietet sie eben die Existenz dieser Uhr.

    Damit hast du zwar recht, ich sehe aber nicht so ganz die Relevanz dieser Aussage…?

    Nein, aber ich habe es gerade auf Wiki nachgelesen, ohne den Hintergrund
    genau zu verstehen. Aber auch das ist eine erste Näherung, eine Vereinfachnung, die z.B. absolute Homogenität voraussetzt. Du musst explizit Störungen einrechnen, um zu einem realistischen Modell zu kommen. Man kann gut damit rechnen, aber ein globales Koordinatensystem definiert diese Metrik auch nur näherungsweise.

    Für solche großen Skalen, wie sie in der Kosmologie üblich sind (also genau für solche Skalen, wo die verschiedenen Entfernungsmasse sich deutlich unterscheiden!) ist die Näherung aber extrem gut!

    Welche dieser Entfernungen ist denn „die richtige“? Die mitbewegte?

    Wie du selbst schon sagtest – es gibt keine „richtige“ oder „falsche“ Entfernung. Man kann höchstens sagen, dass es Entfernungsmasse gibt, die besser oder schlechter mit dem überein stimmen, was man landläufig als „Entfernung“ bezeichnen würde. Und da stimmt meiner Ansicht nach die mitbewegte deutlich am besten mit überein.

    Ich dachte, das [die Entfernung, welche die Galaxie zur Zeit der Lichtaussendung hatte] wäre genau die Laufzeitdistanz.

    Nö, ist es nicht. Die Laufzeitdistanz ist größer – weil sich das Universum während der Laufzeit ja ausdehnt!

    Für Kosmologen sind diese beiden Begriffe [Nutzen und physikalischer Erkenntnisgewinn] identisch.

    Äh, wie kommst du auf die Idee?!

    Aber ein Astronom oder Kosmologe will schließlich herausfinden, wie diese Welt funktioniert.

    Ja. Und?!? Deshalb muss doch trotzdem nicht gleich jede Zahl, die er angibt bzw. berechnen könnte, gleich „physikalischen Erkenntnisgewinn“ bringen?!?

    Da keiner dieser Ansätze „richtig“ ist, habe ich mich gefragt, welcher davon vielleicht „sinnvoll“ ist. Als sinnvoll bezeichne ich Entfernungsmaße, die einem
    {Astronom, Kosmologen, Physiker} erlauben, eine Erkenntnis über das Universum zu erlangen, die er ohne dieses Entfernungsmaß nicht erlangen könnte.

    Ich würde sagen, das leisten alle diese Maßstäbe gleich gut bzw. gleich schlecht.

    Nun meine ich, dass Erkenntnisse nur gewonnen werden können aus Objekten, die im sichtabren Teil des Universums liegen, deren Licht, Neutrinos, Gravitation oder was auch immer hier in unseren Instrumenten landen kann.

    Wenn du meinst, dass die Objekte jetzt sichtbar sind *oder* in Zukunft irgendwann mal sichtbar werden, wie sie früher mal waren, dann hast du natürlich recht. Aber du scheinst zu meinen, dass man nur dann Erkenntnisse gewinnen kann, wenn man Objekte beobachten kann, deren jetzigen Zustand man irgendwann mal wird sehen können – und das ist schlicht Blödsinn.

    Wenn ich nun ein Entfernungsmaß habe, das mir ein Objekt außerhalb des Beobachtungshorizonts platziert, …

    Nochmals: ob ein Objekt außerhalb des Beobachtungshorizonts ist oder nicht, liegt doch nicht am verwendeten Entfernungsmaß! Entweder ist das Objekt da oder nicht – da ändert sich doch nix dadurch dran, wie ich die Entfernung nun genau messe!

  62. @schlappohr: Nochmals genauer: Was du anscheinend übersiehst, ist folgendes: klar können wir von Objekten, die nun jenseits unseres Beobachtungshorizonts liegen, nichts über ihren jetzigen Zustand erfahren. Aber wir können natürlich selbstverständlich trotzdem sehen, wie sie *früher* einmal waren! Ist das deiner Ansicht nach allen Ernstes *kein* Erkenntnisgewinn?!

  63. Niels·
    04.01.12 · 19:34 Uhr

    @Alderamin
    Du verwechselt Expansion mit dunkler Energie.
    Die dunkle Energie ist nicht die Ursache der Expansion des Universums!
    Wenn unser Universum überhaupt keine dunkle Energie enthalten würde, würde es trotzdem expandieren. Nur eben nicht beschleunigt.

    Ich weiß, bevor die dunkle Energie entdeckt wurde, galt ja noch, dass das Weltall seine Expansion wegen der wechselseitigen Gravitation verlangsamen müsste, und dann wäre es geschlossen gewesen, wenn sich die Expansion irgendwann umgekehrt hätte, offen, wenn die Expansion gegen eine endliche Geschwindigkeit konvergiert wäre oder flach im Grenzfall dazwischen. Das wäre dann wie eine Art Schwung gewesen, den der Urknall der Raumzeit mitgegeben hätte, und die Materie im Raum hätte dessen Expansion aufhalten können – so ungefähr hatte ich’s in den 80ern verstanden. Da wäre es dann kein Problem, zu erklären, warum das Sonnensystem nicht expandiert: Es wäre ja keinerlei Kraft mehr da, die den Raum weiterhin antreibt, ausser der Trägheit der Massen und ihrer wechselseitigen Gravitation.

    Inflatonfeld und dunkle Energie führen jeweils zu Gleichungen mit negativem Druck.
    Insofern ähneln sie sich. Das ist aber auch kein Wunder: beide wurden eingeführt, um das selbe zu erklären, nämlich eine schnellere Expansion des Universums als jeweils ohne ihr Vorhandensein.
    Das hast du schon richtig verstanden.

    Wie kann dann aber die Expansion, wie Du sagtest, kräftefrei sein (und ihre Behandlung als abstoßende Kraft im Paper nicht korrekt), wenn sie doch eine negative Gravitation ist?

    Die Expansion des Universums führt nach der ART nie zu Kräften auf die Objekte im Raum, egal ob die Expansion beschleunigt, gebremst oder sonstwie verläuft.

    Hmm, die negative Gravitation wirkt durch den negativen Druck ja in jeder Richtung, d.h. auf eine betrachtete Masse wirkt keine Nettokraft, aber von der Masse aus betrachtet scheinen Kräfte auf alle entfernten Objekte in jeder Richtung zu wirken (man misst ja eine Beschleunigung entfernter Galaxien, also scheint zumindest eine Kraft zu wirken). Das hieße aber, von der Erde aus schiene eine (sehr kleine) Kraft auf die Sonne zu wirken und von der Sonne aus gesehen eine entsprechende auf die Erde, aber beide Orte würden keine auf sie wirkende Nettokraft spüren. Kann man das so sehen?

    Der Abstand zwischen zwei Objekten wächst aber nur dann genau wie der Skalenfaktor, wenn zwischen den Objekten überhaupt keine Kräft wirken.

    Klar, aber wenn Gravitationskräfte wirken, dann müssten diese sich überlagern mit der oben von mir beschriebenen Scheinkraft, die auf entfernte Objekte wirkt – wenn ich das dort richtig verstanden habe. Dann wäre der Effekt, wie im Paper beschrieben, eine leichte Vergrößerung der Erdbahn, weil sie wie eine Verkleinerung der Sonnenmasse wirken würde. Sag‘ jetzt ja, sonst gebe ich’s auf….

    Der ganze Rest Deines Textes: Vollkommen d’accord.

  64. @Bjoern

    Ich glaube, wir haben uns ein wenig verzettelt, deswegen antworte ich nur auf Dein letztes Posting, das bringt uns zurück zum eigentlichen Problem:

    „Aber wir können natürlich selbstverständlich trotzdem sehen, wie sie *früher* einmal waren! Ist das deiner Ansicht nach allen Ernstes *kein* Erkenntnisgewinn?!“

    Selbstverständlich! Aber wir können unsere Erkenntnisse eben *nur* aus den Informationen über den früheren Zustand des Objektes beziehen. Welche Erkenntnisse bringt uns die Information, dass das Objekt von der Expansion mittlerweile auf eine Entfernung von 30 GLy verschoben wurde?

    Wir können damit nichteinmal Rückschlüsse auf die Größe des Universums ziehen, da es (soweit ich weiß) vielleicht Galaxien gibt, die aufgrund der primordialen Expansion niemals innerhalb unseres Beobachtungshorizonts gelegen haben und jetzt eine beliebige Entfernung haben können.

    Wir wissen nur, dass wir in 30 Mrd Jahren + weitere Expansion den jetzigen und dann völlig veralteten Zustand des Objektes beurteilen können, falls wir noch existieren. Welche Erkenntnis bringt uns das *jetzt* ?

    _Genau_ das ist das Problem.

  65. @Alderamin

    Hmm, die negative Gravitation wirkt durch den negativen Druck ja in jeder Richtung, d.h. auf eine betrachtete Masse wirkt keine Nettokraft, aber von der Masse aus betrachtet scheinen Kräfte auf alle entfernten Objekte in jeder Richtung zu wirken (man misst ja eine Beschleunigung entfernter Galaxien, also scheint zumindest eine Kraft zu wirken). Das hieße aber, von der Erde aus schiene eine (sehr kleine) Kraft auf die Sonne zu wirken und von der Sonne aus gesehen eine entsprechende auf die Erde, aber beide Orte würden keine auf sie wirkende Nettokraft spüren. Kann man das so sehen?

    Na gut, man kann die Expansion wohl schon irgendwie als Scheinkraft ansehen.

    Ich habs oben ja geschrieben, der Newtonsche Ansatz funktioniert genau so:
    Nach dem oben beschriebenen Ansatz bekommt man eine
    „kosmische Kraft“, die die Körper auseinander zieht, F = m * R ‚ ‚ (t) = m * R(t)* a ‚ ‚ (t)/a(t).

    Dann wäre „kosmische Beschleunigung“ durch die Expansion
    R ‚ ‚ (t) = R(t)* a ‚ ‚ (t)/a(t).

    Damit wäre die dunkle Energie dann keineswegs unwichtig.
    Es gibt dann mehrere Fälle.
    1)
    In unserem frühen Universum und in Universen ohne dunkle Energie aber mit Materie ist die Expansion gebremst, damit ist a ‚ ‚ (t) negativ, die Kraft bzw. die „kosmologische Beschleunigung“ R ‚ ‚ (t) wirkt also in die selbe Richtung wie die Gravitation, der Radius eines um die Sonne kreisenden Planeten ist kleiner als ohne Expansion.
    2)
    Beim Universumsalter von etwa 7 Milliarden Jahren hatten dunkle Energie und Materie ungefähr den gleichen Einfluss auf a ‚ ‚ (t), damit war a ‚ ‚ (t) = 0 und die damit auch die „kosmologische Beschleunigung“ Null.
    Ein Planet ist genau nach dem Zwei-Körper-Modell um die Sonne gekreist bzw. es gab keinen Einfluss auf die Gravitation.
    3)
    In unserem momentanen Universum ist die Expansion durch die dunkle Energie beschleunigt, a ‚ ‚ (t) ist positiv, die die „kosmologische Beschleunigung“ wirkt der Gravitation entgegen, der Radius eines um die Sonne kreisenden Planeten ist größer als ohne Expansion.
    Für die Zukunft läuft die „kosmische Beschleunigung“ gegen einen Grenzwert.

    a ‚ ‚ (t)/a(t) kann man dann natürlich berechnen und man kann in R ‚ ‚ (t) = R(t)* a ‚ ‚ (t)/a(t) verschiedene Radien bzw. Abstände R(t) einsetzen und das Ganze mit der Gravitationsbeschleunigung vergleichen.
    Wenn du lustig bist, kannst du das ja mal für verschiedene Abstände und Massen machen.
    Momentan ist a ‚ ‚ (t)/a(t) ungefähr 3*10^(-36)* 1/(Sekunde^2)

    Allerdings ist dieses Problem im Rahmen der ART noch nicht gelöst und man hat deswegen keine Ahnung, ob der einfache Zusammenhang R ‚ ‚ (t) = R(t)* a ‚ ‚ (t)/a(t) irgend etwas mit der Realität zu tun hat, ja sogar noch nicht einmal, ob in der richtigen Formel überhaupt a ‚ ‚ (t) vorkommt.

  66. @schlappohr

    Welche Erkenntnisse bringt uns die Information, dass das Objekt von der Expansion mittlerweile auf eine Entfernung von 30 GLy verschoben wurde?

    Welche Erkenntnisse bringen uns die Leuchtkraftentfernung, die Winkeldurchmesserentfernung und die Laufzeitentfernung?

    Wir können damit nichteinmal Rückschlüsse auf die Größe des Universums ziehen

    Doch. Die Laufzeitentfernung eines Atoms der Hintergrundstrahlung ergibt ziemlich genau den Radius des beobachtbaren Universums.
    Und aus der Größe des beobachtbaren Universums können wir Abschätzungen über die Größe das ganzen Universums treffen.

  67. @schlappohr:

    Welche Erkenntnisse bringt uns die Information, dass das Objekt von der Expansion mittlerweile auf eine Entfernung von 30 GLy verschoben wurde?

    Eben diese Erkenntnis, dass dieses Objekt nun da ist. Findest du es nicht interessant zu wissen, wo das Objekt, das man gerade beobachtet, sich eigentlich überhaupt befindet?!

  68. @Bjoern
    Stimmt, für das Verständnis unseres Universums uns seiner Expansion find ich das auch ziemlich wichtig.
    Es gibt auch genug Menschen, die der Meinung sind, Objekte könnten sich seit dem Urknall höchstens 13,7 entfernt haben. Man sollte die Laufzeitentfernung also sehr viel häufiger erwähnen.

  69. Niels schrieb (04.01.12 · 23:42 Uhr):
    > Die Laufzeitentfernung eines Atoms der Hintergrundstrahlung ergibt ziemlich genau […]

    Gegeben:
    zwei Beteiligte („Alice“ und „Bob“), die gegenüber einander den (gleichen) Wert
    „z = 1089.1“ gemessen haben; d.h. den „z“-Wert, durch den „Hintergrundstrahlung“ charakterisiert wird.

    Gesucht:
    Der Wert der „Laufzeitentfernung“ von Alice und Bob voneinander (während des Versuches, aufgrund dessen der angegebenen „z“-Wert zu ermitteln war).

    (Hinweis:
    Nicht bloß auf die besonderen Kurven schauen, die oben im Diagramm „Bild: Richard Powell, CC-BY-SA 2.5“ eingetragen sind — die stellen nur irgendein bestimmtes kosmologisches Modell dar; sondern auf die Messgrößen „Laufzeitentfernung (DT)“ bzw. „Redshift (z)“ an sich, die die gesamte Diagrammfläche aufspannen sollen.)

  70. Beide Male muss es natürlich mitbewegte Entfernung statt Laufzeitentfernung heißen.

    Warum hast du jetzt eigentlich 18 Zeilen gebraucht, um auf diesen Flüchtigkeitsfehler hinzuweisen?

    Übrigens ist zum erstem die Laufzeitentfernung keine Messgröße.
    Zum zweiten sucht man natürlich nicht die „Laufzeitentfernung“ von Alice und Bob voneinander während des Versuches, aufgrund dessen der angegebenen „z“-Wert zu ermitteln war. Das wäre ein absolut anderer Wert. Außerdem braucht man für Laufzeitentfernung selbstverständlich irgendein bestimmtes kosmologisches Model.
    Zum dritten benötigt man selbstverständlich keine zwei Beteiligte, die etwas gegenüber einander messen. Das wäre im Beispielsfall auch ziemlich fatal, da Alice und Bob niemals mehr miteinander kommunizieren können.

  71. Hallo Niels,

    Warum hast du jetzt eigentlich 18 Zeilen gebraucht, um auf diesen Flüchtigkeitsfehler hinzuweisen?

    Willkommen im Club. Wappler hat mal 13 Zeilen für ein zustimmendes „ja“ gebraucht. Er is ein Schwätzer, und eigentlich ist es Zeitverschwendung, auf ihn zu reagieren.

  72. Niels schrieb (05.01.12 · 04:14 Uhr):
    > Beide Male muss es natürlich mitbewegte Entfernung statt Laufzeitentfernung heißen. […] Flüchtigkeitsfehler

    Das kann ich nach wie vor nicht beurteilen; und das war nicht Gegenstand meiner obigen Frage (05.01.12 · 00:49 Uhr).

    Falls du nun (in Korrektur zu deiner Aussage von 04.01.12 · 23:42 Uhr) die folgende Formulierung eben eher für diskutabel hälst:

    > Die mitbewegte Entfernung eines Atoms der Hintergrundstrahlung ergibt ziemlich genau […]

    … dann steht meine Frage entsprechend:

    Gegeben:
    zwei Beteiligte („Alice“ und „Bob“), die gegenüber einander den (gleichen) Wert
    „z = 1089.1“ gemessen haben; d.h. den „z“-Wert, durch den „Hintergrundstrahlung“ charakterisiert wird.

    Gesucht:
    Der Wert der „mitbewegten Entfernung“ von Alice und Bob voneinander (während des Versuches, aufgrund dessen der angegebenen „z“-Wert zu ermitteln war).

    (Hinweis:
    Nicht bloß auf die besonderen Kurven schauen, die oben im Diagramm „Bild: Richard Powell, CC-BY-SA 2.5“ eingetragen sind — die stellen nur irgendein bestimmtes kosmologisches Modell dar; sondern auf die Messgrößen „mitbewegte Entfernung (DC)“ bzw. „Redshift (z)“ an sich, die [bzw. genauer: deren Wertebereiche] die gesamte Diagrammfläche aufspannen sollen.)

    > Laufzeitentfernung [ist] keine Messgröße

    Merkwürdig nur, dass der Ausdruck „ly“ im obigen Diagramm dennoch offenbar als „Maßeinheit“ gebraucht wird …

    > Außerdem braucht man für Laufzeitentfernung selbstverständlich irgendein bestimmtes kosmologisches Model .

    Aha. Na dann ist „Laufzeitentfernung“ offenbar keine Messgröße, sondern eben nur (irgend-)ein Modellparameter.

    Wenden wir uns also besser der „mitbewegten Entfernung“ zu — wie deren Werte (zumindest im gedanken-experimentellen Prinzip) zu messen sind; und insbesondere für Beteiligte (Paare), die durch einen bestimmten Wert der „Redshift (z)“ (zueinander) charakterisiert sind.

    > Zum dritten benötigt man selbstverständlich keine zwei Beteiligte, die etwas gegenüber einander messen.

    Die „Formel für z“ oben im Artikel benutzt doch offensichtlich zwei verschiedene Indices: „beobachtet“ und „0„.
    Beziehen diese zwei verschiedenen Indices nicht auf (mindestens) zwei verschiedene Beteiligte? …

    > Das wäre im Beispielsfall auch ziemlich fatal, da Alice und Bob niemals mehr miteinander kommunizieren können.

    Versteh ich nicht — ist das (wieder mal nur irgend-)eine Modellerwartung?
    Oder ist es grundsätzlich ausgeschlossen, dass ein „Atom der Hintergrundstrahlung„, dessen Anzeigen Bob (oder du oder ich) dauern beobachteten (um einen „z-Wert“ abzuschätzen), unmittelbar darauf eintrifft?
    Wäre doch beispielsweise fatal, wenn es zu Besuch käme und alle zusammen feststellen würden, dass Bob (oder du oder ich) Fehler bei der Abschätzung von Zahlen begangen hätten, die mit dem Index „0“ zu tun haben …

    Das heißt natürlich nicht, dass zwei gegebene Beteiligte zwangsläufig (d.h. allein schon wegen der zugrundeliegenden Definition der Messgröße) gegenseitig gleiche „z-Werte“ erhalten würden. Der Pound-Rebka-Versuch zeigt ja (u.a.) stattdessen, dass sich bestimmte Paare von Beteiligten i.A. entsprechend in „den Höheren“ und „den Tieferen“ unterscheiden lassen.

    Aber für Einträge im obigen Diagramm kommen doch sicherlich nur solche ausgewählten Paare in Betracht, die gegenseitig und einvernehmlich (wie sich spätestens beim eventuell anschließenden Besuch zu erweisen hätte) den gleichen Wert ihrer „Redshift (z)“ (zueinander) abgeschätzt hätten; also den einen Wert „Redshift (z)„, der sie als Paar gemeinsam charakterisierte (zumindest vor einem eventuell anschließenden Besuch).

  73. So ganz logisch erscheint mir das weiterhin nicht. Vor 13 Mrd Jahren befanden sich alle Objekte viel näher zusammen. Ausgesendetes Licht hatte nur die damalige Distanz zu überwinden. es hätte also viel früher dort gewesen sein müssen, wo unser Ort sich damals befunden hat. Die Expansionsgeschwindigkeit ist ja zudem geringer gewesen.
    Die Rotverschiebung belegt, daß sich das Licht im Verhältnis zu uns jetzt mit annähernd Lichtgeschwindigkeit nähert. Wir können also nicht mit dem Universum in derartiger Geschwindigkeit expandieren, daß wir vor dem Lichtsignal „herfliegen“. Dann wäre die Rotverschiebung viel größer. Wir könnten es gar nicht als Licht wahrnehmen. Bewegt sich in dem Beispiel vom Polizeiauto mit Schallgeschwindigkeit weg, so hört man gar nichts mehr, weil man sich neben den Wellen bewegt und solche nicht mehr eintreffen und keine töne mehr produzieren können.
    Die angenommene Signallaufzeit des Lichts ist so nicht erklärt. Es stammt vielleicht von viel späteren Zeitpunkten?

  74. @Mano: Ich versteh nicht genau, wo dein Problem liegt. Kannst du es nochmal erklären?

    „Die Rotverschiebung belegt, daß sich das Licht im Verhältnis zu uns jetzt mit annähernd Lichtgeschwindigkeit nähert. „

    Das Licht bewegt sich IMMER mit Lichtgeschwindkeit. Egal, was sich wie bewegt.

  75. „Das Licht bewegt sich IMMER mit Lichtgeschwindkeit. Egal, was sich wie bewegt.“
    Klar. Die errechnete Distanz ergibt sich aus Lichtgeschwindigkeit und Laufzeit. Diese Distanz hat es bei Aussendung des Lichtsignals aber nicht gegeben. Wie weit waren die Orte vor 13 Mrd Jahren entfernt? Mit Lichtgeschwindigkeit hätte unser Ort auch bei Expansion viel früher erreicht werden müssen.
    Nur wenn wir uns entsprechend schnell entfernen, kann uns das Signal erst jetzt erreichen. Dann wäre die Differenz zur Lichtgeschwindigkeit aber geringer und das Licht würde mit einer geringeren Frequenz eintreffen. Genauso wie es zur angenommenen Rotverschiebung bei Aussendung kommt.
    – Irgendwas verstehe ich offensichtlich also nicht.

  76. @mano:

    Wir können also nicht mit dem Universum in derartiger Geschwindigkeit expandieren, daß wir vor dem Lichtsignal „herfliegen“.Doch. Alles, was weiter entfernt ist als etwa 45 Mrd. Lichtjahre, enfernt sich durch die Expansion so schnell von uns, daß sich alles in unsere Richtung ausgesendete Licht weniger schnell nähert als sich das Universum ausdehnt. Wie bereits beschrieben: solches Licht fliegt in unsere Richtung, entfernt sich aber von uns. Diese Bereiche des Universums sind für uns für immer unsichtbar.

  77. @Niels:

    „Welche Erkenntnisse bringen uns die Leuchtkraftentfernung, die Winkeldurchmesserentfernung und die Laufzeitentfernung?“

    Letztere bringt uns die Information, dass ein Objekt nahe genug war oder ist, um durch Strahlung und Gravitation die Geschichte des sichtbaren Universums in irgeneiner Weise mitzugestalten. Sie hat uns z.B. erlaubt, die filamentartige Struktur des Universums zu entdecken, die Voids und die Great Wall. Diese Strukturen ergaben sich aus der Laufzeitdistanz.

    „Die Laufzeitentfernung eines Atoms der Hintergrundstrahlung ergibt ziemlich genau den Radius des beobachtbaren Universums.“

    Was bitte ist ein Atom der Hintergrundstrahlung? Ich kann Dir nicht ganz folgen.

    „Und aus der Größe des beobachtbaren Universums können wir Abschätzungen über die Größe das ganzen Universums treffen. “

    Richtig. Wenn Du mir jetzt noch erklärst, wie Dir die mitbewegte Entfernung bei dieser Erkenntnis geholfen hat, bin ich zufrieden. Ich habe nicht gesagt, das wir die Größe nicht abschätzen können, aber eben nur aus Objekten, deren LZ-Distanz wir messen können.

    @Bjoern

    „Findest du es nicht interessant zu wissen, wo das Objekt, das man gerade beobachtet, sich eigentlich überhaupt befindet?!“

    Interessant ist das ganze ohne Frage. Aber wo das Objekt eigentlich ist, sagt uns die mitbewegte Entfernung genauso wenig wie jedes andere Maß. Zeig mir bitte mal ein einziges Paper, in dem die mitbewegte Distanz *benötigt* wurde, um zu einem relevanten Ergebnis zu kommen.

  78. @Mano: „Irgendwas verstehe ich offensichtlich also nicht. „

    Es geht im Artikel ja gerade darum, dass es keine eindeutige Entfernung gibt, die man definieren kann.

  79. @Bullet: Das Licht hat aber trotzdem Lichtgeschwindigkeit. Wenn es bei uns nur mit Rotverschiebung eintrifft, können wir nicht nennenswert vor ihm „herfliegen“. Sonst sähen wir es nicht als Lichtsignal.
    Bei Absendung war das alles nicht 45 Mrd Lichtjahre entfernt.
    @Florian: Doch, sie beträgt angeblich 13 Mrd Jahre, die Zeit nämlich, die das Licht unterwegs war. Zweifelhaft ist, daß das Licht bei konstanter Lichtgeschwindigkeit für die bei Aussendung bestehende Entfernung plus damaliger Expansionsgeschwindigkeit so lange gebraucht haben könnte, bis es hier eintrifft.
    Wenn das Universum 13,7 Mrd Jahre alt ist, dann kann es mit 0,7 Mrd Jahren viel weniger als 1/13 an Ausdehnung gehabt haben, zumal die Expansionsgeschwindigkeit ja erst später einsetzte und dann zunahm. Die Ausdehnung ist ein dynamischer Vorgang, die Lichtgeschwindigkeit ist statisch. Unsere jetzige Region war damals der Lichtquelle umso näher und das Licht hätte sie viel früher erreichen müssen.
    Daß es keine eindeutige Entfernung gibt meint nur, daß die Ausdehnung fortschreitet und der Entstehensort nicht festgelegt werden kann außer in Lichtjahren.
    Dennoch gibt es ihn.

  80. Bullet schrieb (05.01.12 · 14:39 Uhr):

    > Alles, was weiter entfernt ist als etwa 45 Mrd. Lichtjahre, enfernt sich […]
    > Diese Bereiche des Universums sind für uns für immer unsichtbar.

    Dann meinst du offenbar stattdessen „alles, was“ keiner, der dich je beobachtet hat oder beobachten wird, jemals beobachtet hat oder beobachten wird; und keiner, den du je beobachtet hast oder beobachten wirst, jemals beobachtet hat oder beobachten wird;

    und gegenüber „all dem“ folglich weder du noch irgendwer, mit dem du zu tun hättest, je irgendeinen bestimmten „z-Wert“ feststellen könnte, und ob man „sich [(voneinander)] entfernte„, oder nicht;

    ganz zu schweigen von irgendeinem „Entfernungs„-Wert (sei es „DL„, „DC„, „DT“ oder „DA„), und ob „45 Gly“ ein größerer oder kleinerer Wert sei.

  81. @Mano

    Martin Bäker hat in seinem Artikel zur Expansion des Universums den Fall eines Photons gezeigt, dass sich anfangs sogar mit der Hubble-Expansion von uns entfernt und erst allmählich die Kurve bekommt, um uns schließlich doch noch zu erreichen.

    Das Licht ist zwar lokal immer gleich schnell, aber überlagert mit der Expansionsgeschwindigkeit des Weltalls kann es von einem mit der Hubble-Expansion bewegten entfernten Ziel auf dem Weg zu uns sogar rückwärts laufen und dann scheinbar immer schneller werden.

    Die nach oben laufenden Linien sind übrigens Linien gleicher mitbewegter Entfernung.

  82. @Frank Wappler
    Wie mitbewegte Entfernung und die Laufzeitentfernung berechnet werden, findest du heraus, wenn du drei Sekunden lang googelst.
    https://lmgtfy.com/?q=mitbewegte+entfernung
    Den ersten Treffer anklicken.

    Ist damit die Frage beantwortet?

    @schlappohr

    Letztere bringt uns die Information, dass ein Objekt nahe genug war oder ist, um durch Strahlung und Gravitation die Geschichte des sichtbaren Universums in irgeneiner Weise mitzugestalten.

    Hm? Schon mal vom Horizontproblem und der Inflation gehört?
    Die Laufzeitentfernung hilft uns bei dieser Frage nicht weiter.
    Und selbst ohne Inflation könnte man aus Laufzeitentfernung und mitbewegter Entfernung genau das selbe schließen.

    Sie hat uns z.B. erlaubt, die filamentartige Struktur des Universums zu entdecken, die Voids und die Great Wall. Diese Strukturen ergaben sich aus der Laufzeitdistanz.

    Warum soll uns das ausgerechnet durch die Laufzeitentfernung erlaubt haben?

    „Und aus der Größe des beobachtbaren Universums können wir Abschätzungen über die Größe das ganzen Universums treffen. “
    Richtig. Wenn Du mir jetzt noch erklärst, wie Dir die mitbewegte Entfernung bei dieser Erkenntnis geholfen hat, bin ich zufrieden.

    Der Radius des beobachtbaren Universums ist genau die mitbewegte Entfernung eines gedachten Objektes, dessen kurz nach dem Urknall abgestrahltes Licht uns gerade im Moment erreicht.
    Die Laufzeitentfernung ist 13,7 Gly, die mitbewegte Entfernung ist 47 Gly.

    Aber wo das Objekt eigentlich ist, sagt uns die mitbewegte Entfernung genauso wenig wie jedes andere Maß

    Doch, die mitbewegte Entfernung sagt uns genau das.

    Zeig mir bitte mal ein einziges Paper, in dem die mitbewegte Distanz *benötigt* wurde, um zu einem relevanten Ergebnis zu kommen.

    In kosmologischen Papern verwendet man in der Regel sowohl „proper distance“ als auch „comoving distance“. Prinzipiell ist es aber egal, welche dieser Größen verwendet wird.
    Eine Dinge sieht man bei Benutzung der einen Skala besser, andere bei Verwendung der Anderen.
    Du kannst ja mal arXiv mit diesen Begriffen füttern.

    @Bullet

    Alles, was weiter entfernt ist als etwa 45 Mrd. Lichtjahre, enfernt sich durch die Expansion so schnell von uns, daß sich alles in unsere Richtung ausgesendete Licht weniger schnell nähert als sich das Universum ausdehnt. Wie bereits beschrieben: solches Licht fliegt in unsere Richtung, entfernt sich aber von uns. Diese Bereiche des Universums sind für uns für immer unsichtbar.

    Das ist schon richtig, nur extrem ungenau.
    Die Frage, ob wir das heute abgestrahlte Licht eines Objekte irgendwann werden sehen können, beantwortet allerdings der kosmologische Ereignishorizont, nicht der Radius des beobachtbaren Universums.
    Deswegen fängt dieser Bereich schon ab diesem Horizont mit einem Abstand von 16 Gly an.

  83. @schlappohr:

    [Die Laufzeitentfernung] bringt uns die Information, dass ein Objekt nahe genug war oder ist, um durch Strahlung und Gravitation die Geschichte des sichtbaren Universums in irgeneiner Weise mitzugestalten.

    Äh, diese Information kann man doch aus allen anderen Entfernungsmassen auch ablesen? Bzw. letztlich gilt doch für alles, was wir im Moment beobachten können, dass es „die Geschichte des sichtbaren Universums in irgeneiner Weise“ mitgestaltet hat!

    Sie hat uns z.B. erlaubt, die filamentartige Struktur des Universums zu entdecken, die Voids und die Great Wall. Diese Strukturen ergaben sich aus der Laufzeitdistanz.

    Bitte was?!? Wo hast du denn das her? Meines Wissens ergab sich diese Strukturen schlicht aus Messungen der Rotverschiebung! Siehe z. B. diese Grafik:
    https://www.talkorigins.org/faqs/astronomy/lcrs.gif

    Was bitte ist ein Atom der Hintergrundstrahlung?

    Ein Atom, das einen Teil der Strahlung ausgesendet hat, die wir nun als Hintergrundstrahlung sehen. (Etwas ungünstig ausgedrückt, aber durchaus verständlich, finde ich.)

    Wenn Du mir jetzt noch erklärst, wie Dir die mitbewegte Entfernung bei dieser Erkenntnis geholfen hat, bin ich zufrieden.

    Niels hat sich schlicht (mal wieder 😉 ) verschrieben; was er meinte, war:

    Die mitbewegte Entfernung eines Atoms der Hintergrundstrahlung ergibt ziemlich genau den Radius des beobachtbaren Universums.

    (inzwischen hat er’s ja schon selbst korrigiert)

    Ich habe nicht gesagt, das wir die Größe nicht abschätzen können, aber eben nur aus Objekten, deren LZ-Distanz wir messen können.

    Äh, aber die Objekte, deren Laufzeit-Entfernung wir bestimmen (nicht „messen“) können, sind doch genau dieselben Objekte wie die, deren mitbewegte Entfernung wir bestimmen können! (und auch alle anderen Entfernungsmasse!) Das sind doch
    nämlich schlicht und einfach alle Objekte, die im Moment beobachtbar sind!

    Aber wo das Objekt eigentlich ist, sagt uns die mitbewegte Entfernung genauso wenig wie jedes andere Maß.

    Wie weiter oben schon erklärt, ist die mitbewegte Entfernung (meiner Ansicht nach – aber damit stehe ich nicht alleine) das, was am ehesten mit dem übereinstimmt, was man intuitiv unter „Entfernung“ versteht.

    Zeig mir bitte mal ein einziges Paper, in dem die mitbewegte Distanz *benötigt* wurde, um zu einem relevanten Ergebnis zu kommen.

    Kann ich nicht. Das gilt für all die anderen Entfernungsmaße aber genauso… was *benötigt* wird, um zu relevanten Ergebnissen zu kommen, ist einzig und alleine eine Messung der Rotverschiebung und der Helligkeit. Die Entfernungsmaße sind nur zur Veranschaulichung gedacht, *nötig* ist keines davon wirklich, um zu relevanten Ergebnissen zu kommen.

  84. @Mano: Erst mal eine sehr wichtige Korrektur, die bisher anscheinend jeder übersehen hat:

    Die Expansionsgeschwindigkeit ist ja zudem geringer gewesen.

    Das ist falsch – die Expansionsgeschwindigkeit (genauer eigentlich: die Expansionsrate) war damals größer!

    Damit dürfte sich das folgende

    Ausgesendetes Licht hatte nur die damalige Distanz zu überwinden. es hätte also viel früher dort gewesen sein müssen, wo unser Ort sich damals befunden hat.

    eigentlich erledigt haben…?

    Die Rotverschiebung belegt, daß sich das Licht im Verhältnis zu uns jetzt mit annähernd Lichtgeschwindigkeit nähert.

    Häh? Die Rotverschiebung belegt nichts dergleichen.

    Wir können also nicht mit dem Universum in derartiger Geschwindigkeit expandieren, daß wir vor dem Lichtsignal „herfliegen“. Dann wäre die Rotverschiebung viel größer.

    Bitte was? Dann würden wir das Lichtsignal schlicht nie zu gesicht bekommen, es gäbe also keinen Sinn, von dessen Rotschiebung für uns zu reden…

    Die errechnete Distanz ergibt sich aus Lichtgeschwindigkeit und Laufzeit.

    Wenn du die Laufzeit-Entfernung meinst, dann ja.

    Diese Distanz hat es bei Aussendung des Lichtsignals aber nicht gegeben. Wie weit waren die Orte vor 13 Mrd Jahren entfernt?

    Du willst wissen, wie weit ein Ort, der heute eine Laufzeit-Entfernung von 13 Milliarden Lichtjahren von uns hat, von uns entfernt war, als das Licht ausgesendet wurde? Moment, das kann man nachrechnen…… etwa 3,6 Milliarden Lichtjahre.

    Mit Lichtgeschwindigkeit hätte unser Ort auch bei Expansion viel früher erreicht werden müssen.

    Warum? Weil dir das plausibel vorkommt? Tut mir leid für dich, aber viel, was einem auf den ersten Blick plausibel vorkommt, ist trotzdem falsch… wie gesagt, man kann es nachrechnen.

    Nur wenn wir uns entsprechend schnell entfernen, kann uns das Signal erst jetzt erreichen.

    Tja, dann entfernen wir uns halt offensichtlich „entsprechend schnell“!?!

    Dann wäre die Differenz zur Lichtgeschwindigkeit aber geringer und das Licht würde mit einer geringeren Frequenz eintreffen.

    Bitte was?!? Das ist nun völlig unverständlich. Was meinst du mit „Differenz zur Lichtgeschwindigkeit“? Und: Die Frequenz würde niedriger als was sein?!?

    Genauso wie es zur angenommenen Rotverschiebung bei Aussendung kommt.

    Bitte was? Die Rotverschiebung passiert während der Laufzeit des Lichts, nicht bei der Aussendung.

    Das Licht hat aber trotzdem Lichtgeschwindigkeit. Wenn es bei uns nur mit Rotverschiebung eintrifft, können wir nicht nennenswert vor ihm „herfliegen“. Sonst sähen wir es nicht als Lichtsignal.

    Das ist etwas vage. Was meinst du mit „nennenswert“?

    Doch, sie [die Entfernung] beträgt angeblich 13 Mrd Jahre,

    Das ist ein mögliches Entfernungsmass. Wie Florian aber im Artikel erklärt hat, gibt es aber eben kein eindeutiges Entfernungsmass. Zu sagen, die Laufzeit-Entfernung wäre eindeutig die richtige Entfernung, ist schlicht falsch. (wenn schon, dann entspricht die „richtige“ Entfernung immer noch am ehesten der mitbewegten…)

    Zweifelhaft ist, daß das Licht bei konstanter Lichtgeschwindigkeit für die bei Aussendung bestehende Entfernung plus damaliger Expansionsgeschwindigkeit so lange gebraucht haben könnte, bis es hier eintrifft.

    Warum ist das zweifelhaft? Ich kann dir nicht ganz folgen.

    Wenn das Universum 13,7 Mrd Jahre alt ist, dann kann es mit 0,7 Mrd Jahren viel weniger als 1/13 an Ausdehnung gehabt haben,

    Äh, wieso?!? Die Ausdehnung ist doch nicht proportional zur vergangenen Zeit! (tatsächlich war die Ausdehnung zu der Zeit übrigens etwa 1/7,2 der heutigen Ausdehnung, also mehr als 1/13 der heutigen Ausdehnung…)

    …zumal die Expansionsgeschwindigkeit ja erst später einsetzte und dann zunahm.

    Falsch, siehe oben. Wo hast du denn nur diese Idee her?!? (hm, vielleicht „beschleunigte Expansion des Universums“ falsch verstanden…?)

    Daß es keine eindeutige Entfernung gibt meint nur, daß die Ausdehnung fortschreitet und der Entstehensort nicht festgelegt werden kann außer in Lichtjahren.

    Nein, das meint es nicht! Les‘ Florians Artikel nochmal, und versuch‘ diesmal, ihn zu verstehen!

  85. Niels schrieb (05.01.12 · 17:21 Uhr):
    > Wie mitbewegte Entfernung und die Laufzeitentfernung berechnet werden [… https://de.wikipedia.org/wiki/Entfernungsmaß ]

    > Ist damit die Frage beantwortet?

    Aber nein! —
    das Anliegen meiner Frage ist doch nicht (nur) irgendeine Berechnung (insbesondere nicht nur im Rahmen irgendeines bestimmten Modells; z.B., wie in dem angegebenen Wikipedia-Artikel vorausgesetzt, nur „in einem Universum, dessen globale Entwicklung durch die Friedmann-Gleichungen beschrieben wird„).
    Es geht vielmehr sozusagen darum, wie zu entscheiden wäre, ob die eine oder andere Berechnungs-Funktion physikalisch/empirisch zutreffend und „richtig“ ist, oder nicht.

    Das oben diskutierte Diagramm hat doch zwei formal unabhängige Achsen (eine für „z-Werte“, und eine u.a. für „mitbewegte Entfernung, DC„).
    Es geht mir deshalb darum, ob und wie für ein bestimmtes Paar von „Quelle und Beobachter“ (wie die Beteiligten im angegebenen Wikipedia-Artikel genannt werden) deren „DC„-Wert zu messen wäre, ohne dass insbesondere der Wert z2 schon bekannt wäre (wobei entsprechend der obigen „z-Formel“ z1 = 0 gesetzt sein mag).

    Und auch ganz unabhängig von irgendwelchen Betrachtungen zu „Helligkeit“ und/oder „Winkeldurchmesser“ (der „Quelle„; ggf. hinsichtlich des „Beobachters„), weil ja die Zahl „λ beobachter / λ 0“ sicherlich für beliebige „Helligkeits„- und/oder „Winkeldurchmesser„-Werte gleichermaßen auswertbar ist;
    also hinsichtlich einer „Galaxie 0“ zumindest im Prinzip genauso wie für ein „Atom 0„.

    Nun mag es ja sein, dass du schlicht keine unabhängigen Messdefinitionen für die Größen auf den beiden Diagrammachsen kennst, sondern eben wirklich nur den Berechnungszusammenhang, den ich deinem Hinweis folgend gefunden habe.

    Dann kann ich meine Frage sinngemäß auch ganz ausdrücklich unabhängig von irgendwelchen „kosmologischen Modellen“ stellen, sondern eben im Rahmen der (Experimental-)Physik an sich; und zwar bezugnehmend auf Begriffe, die im oben angegebenen Wikipedia-Artikel genannt und mitunter sogar verlinkt sind:

    Gegeben:
    drei Paare von unterscheidbaren Beteiligten: A und J, B und K, sowie P und Q;
    so dass sich jedes dieser drei Paare (mindestens) einmal „traf und passierte“;
    so dass die so identifizierbaren drei Ereignisse „AJ“, „BK“ und „PQ“ zueinander „raumartig“ waren;
    und so dass von jedem Paar aus diesen Ereignissen je ein bestimmter „Linienelement, ds„-Wert zu messen ist, d.h. so dass die Verhältnisse

    ds(AJ)(BK) / ds(AJ)(PQ)“ und
    ds(BK)(PQ) / ds(AJ)(PQ)“ sowie
    ds(AJ)(BK) / ds(BK)(PQ)

    als reelle Zahlen ermittelt werden könnten.

    Gegeben sei weiterhin, dass sich die Beteiligten i.A. gegenseitig beobachteten,
    wobei B nach Wahrnehmung des Treffens mit J ausdrücklich auch wahrnahm, dass sich A und J getroffen hatten.
    (Gegeben seien auch hinreichend weitere geeignete Beteiligte, falls erforderlich.)

    Gesucht:
    Der „z-Wert“ betreffend A bei Ereignis „AJ“ als „Quelle„, und B (beim Wahrnehmen, dass sich A und J getroffen hatten) als „Beobachter„;
    und zwar ausdrücklich ohne irgendwelche weiteren (Modell-)Vorgaben.
    (Hinweis: das ist in Analogie zur oben gestellten Aufgabe; und bestimmt nicht schwieriger!)

    Auch gesucht (weil ich das selbst kaum zu lösen weiß):
    eine (gedanken-experimentelle) Methode, durch deren Anwendung die Beteiligten in Auswertung ihrer gegenseitigen Beobachtungen die genannten „Linienelement„-Verhältnisse einvernehmlich ermitteln könnten;
    insbesondere (und sicherlich besonders einfach), wie sie feststellen sollen, ob
    ds(AJ)(BK) / ds(AJ)(PQ)“ gleich 1, oder ungleich 1 war.
    (Anmerkung: Das sollte man beantworten können, wenn man Koordinaten nicht nur als „beliebig darübergestreuselt“ betrachten möchte, sondern als zumindest nachträglich „skaliert-isometrisch zugeordnet“.)

    p.s.
    „Text<sub>Subskript</sub>“ zeigt sich als: „TextSubskript

  86. Frank Wappler schrieb (06.01.12 · 01:26 Uhr):
    > drei Paare von unterscheidbaren Beteiligten: A und J, B und K, sowie P und Q; so dass sich jedes dieser drei Paare (mindestens) einmal „traf und passierte“ […]
    > wobei B nach Wahrnehmung des Treffens mit J ausdrücklich auch wahrnahm, dass sich A und J getroffen hatten.

    Sollte stattdessen sein:
    „… wobei B nach Wahrnehmung des Treffens mit K ausdrücklich auch wahrnahm, dass sich A und J getroffen hatten.“

  87. @Alderamin
    Hattest du eigentlich gesehen, dass ich dir am 04.01.12 · 23:28 Uhr geantwortet habe?

    @Frank Wappler
    Die Frage wurde beantwortet.
    Auf deine rabulistischen Spielchen lass ich mich nicht ein, nicht zuletzt, weil es dir erkennbar nicht um Erkenntnisgewinn geht.

  88. Florian, ich finde deinen Artikel und insbesondere die Diskussion der kosmologischen Distanz in Abhängigkeit von der Rotverschiebung sehr gelungen. Selten findet man das so kompakt beisammen. Leider bin ich sehr spät darauf gestoßen.

    Nun ist die Vorstellung, das Licht entfernter Galaxien verschiebt rot, weil der Raum sich ausdehnt, zwar weit verbreitet, unter Kosmologen aber nicht völlig unumstritten. Vermutlich bist du darauf nicht eingegangen um Verwirrung zu vermeiden.

    Zunächst ein Verfechter der Expansion.

    In seinem Buch „Kosmologie“, 1984, beklagt Edward R. Harrison die weitverbreitete „Verwechslung von Expansions- und Doppler-Rotverschiebung“. Den Unterschied macht er mit einem Gedankenexperiment klar, in dem Emission und Absorbtion des Photons in einem statischen Universum stattfinden (also keine Dopplerverschiebung), das aber, solange das Photon unterwegs ist, expandiert und so dessen Wellenlänge „streckt“.

    Gegen dieses Argument wenden Bunn & Hogg jedoch in „The kinematic origin of the cosmological redshift“, 2009, https://arxiv.org/PS_cache/arxiv/pdf/0808/0808.1081v2.pdf ein, daß mit dem Paralleltransport der vierer-Geschwindigkeit auf dem Pfad dieses Photons auch bei einer ungleichmäßigen Expansionsrate von Dopplerverschiebung gesprochen werden kann. Sie glauben allerdings mit diesem Ansatz eine rein kinematische Erklärung der kosmologischen Rotverschiebung zu haben. Demgegenüber erweitert Peacock, Autor des Standardwerkes „Cosmological Physics“, 1999, in seinem Artikel „A diatribe on expanding space“, 2008, https://arxiv.org/PS_cache/arxiv/pdf/0809/0809.4573v1.pdf die speziell-relativistische Dopplerformel unter Anwendung des Birkhoff-Theorems um einen gravitativen Blauverschiebungsterm und folgert: „To second order it is exactly correct to think of the cosmological redshift as a combination of doppler and gravitational redshift.“ Ähnlich argumentieren auch andere Kosmologen, wie etwa Chodorowski in „The kinematic component of the cosmological redshift“, 2011, https://arxiv.org/PS_cache/arxiv/pdf/0911/0911.3536v3.pdf .

    Was soll man nun davon halten? Offenbar gibt es kein Experiment, das eine Expansion von Raum beweist und die atronomischen Befunde sind interpretiertbar. Peacock’s Bemerkung in dem erwähnten Artikel in Verbindung mit zähen Mißverständnissen, „The worst of these is the ‚expanding space‘ fallacy“ zielt sicherlich auf die Fachwelt. Und Chodorowski’s „such statements are misleading“ (gemeint ist die Expansion) ebenfalls. Muß man daraus schließen, daß die Experten in dieser Frage uneinig sind? In populärwissenschaftlichen Zeitschriften und Büchern werden zur Expansion des Raums alternative Deutungen der kosmologischen Rotverschiebung mit Ausnahme des Artikels von Tamara Davis im Spektrum 11/10 nicht mal erwähnt. Warum eigentlich? Dem Leser wird immer wieder erklärt, weshalb Überlichtgeschwindigkeiten nicht gegen die spezielle Relativitätstheorie verstoßen, aber nicht, daß diese ein Artefakt mitbewegter Koordinaten sind und bei einer kinematisch-gravitativen Deutung (quasi-statische Koordinaten) garnicht auftreten, auch nicht während der inflationären Phase (s. Chodorowski). Quasi-statisch um anzudeuten, daß die Rotverschiebung nicht ausschließlich einer Relativbewegung (Doppler-Effekt) zuzuschreiben ist.

    Interessant ist finde ich in diesem Zusammenhang die Gleichwertigkeit eines expandierenden masselosen FRW-Universums mit dem Milne-Modell, das ein reines SRT Universum ist. Da Gravitation nicht vorhanden, sind hier Expansion des Raums und Relativbewegung ohne Einschränkung äquivalent. Die Koordinatentransformation ist relativ einfach. Näheres dazu findet sich in der Dissertation von Tamara Davis im Kapitel 4, „The empty Universe“, https://www.dark-cosmology.dk/~tamarad/papers/thesis_complete.pdf

  89. @Güntim

    Das ist eigentlich alter, längst überholter Kram. Wenn man die Rotverschiebung durch andere Effekte erklären wollte (die dann noch gar nicht nachgewiesen wären) und man also ein statisches Universum annähme (keine Expansion, keine Kompression), da fragt es sich zunächst, wie sich die Galaxien auf Position halten sollen, wo sie sich doch untereinander anziehen müssten. Deshalb hatte Einstein seine kosmologische Konstante erfunden.

    Man muss dann erklären, woher die kosmologische Hintergrundstrahlung kommt. Warum ferne Galaxien eine höhere Sternentstehungsrate haben. Warum es keine nahen Quasare gibt. Warum Galaxien ab einem gewissen z (ca. 1,5, siehe =>hier) plötzlich wieder größer zu werden scheinen, so als ob sie näher wären.

    Es gibt im Prinzip zwei endgültige Sargnägel für die Steady-State-Theorie:

    1) die Entfernungsmessung über Supernovae belegt eindeutig, wie sich der Hubble-Parameter (also die Rotverschiebung in Abhängigkeit von der Entfernung) entwickelt hat. Bei „müdem Licht“ oder dergleichen wäre überhaupt nicht zu erwarten, dass sich der Effekt über die Zeit ändert. Bei einem Cold Dark Matter Universum mit Dunkler Energie erklärt sich die Entwicklung des Hubble-Parameters hingegen ganz zwanglos.

    2) Die Zeitdilatation von Ereignissen bei hohem z. Da gibt’s zweierlei Arten von Ereignissen, die wir beobachten können: Gamma Ray Bursts und Supernovae. Die laufen um den Faktor 1+z langsamer ab. Der Mechanismus ist hierbei der folgende: zwei hintereinander ausgesendete Zeitmarken entfernen sich durch die Ausdehnung des Raums zwischen ihnen immer weiter voneinander, so dass die Empfangszeitpunkte bei einem sehr weit entfernten Empfänger zeitlich viel weiter auseinanderliegen werden, als die Sendezeitpunkte. Und das ist genau der Mechanismus der kosmologischen Rotverschiebung, die kein Dopplereffekt ist.

    Es gibt immer noch ein paar Anhänger alternativer Thesen, und es ist auch gut, dass sich Leute Gedanken machen, aber die Urknalltheorie steht genau so fest und sicher wie die Evolution, auch wenn einige Aspekte wie dunkle Energie oder dunkle Materie noch der Erklärung harren.

  90. @ Florian

    Die Steady-State-Theorie ist ein alter Hut um den es allerdings nicht geht, wovon schon das Erscheinungsdatum der erwähnten papers zeugt. Es geht um die Wahl der Koordinaten und damit um die Vorstellung Expansion vs. Relativbewegung. Dabei wird etwa die lambda-Kosmologie oder das kosmologische Prinzip ebensowenig angetastet, wie das Schwarze Loch, wenn man Schwarzschild- in Kruskal-Koordinaten umrechnet.

    Der Kosmologe Ned Wright in FAQ:
    https://www.astro.ucla.edu/~wright/cosmology_faq.html#MX
    Are galaxies really moving away from us or is space just expanding?

    This depends on how you measure things, or your choice of coordinates. In one view, the spatial positions of galaxies are changing, and this causes the redshift. In another view, the galaxies are at fixed coordinates, but the distance between fixed points increases with time, and this causes the redshift. General relativity explains how to transform from one view to the other, and the observable effects like the redshift are the same in both views. Part 3 of the tutorial shows space-time diagrams for the Universe drawn in both ways.

    Alles weitere zum Verständnis findet sich bei Peacock und den anderen Autoren.

  91. @Güntim

    Ok. Nun ist es zunächst einmal ein philosophischer Unterschied, ob die Galaxien sich von uns fortbewegen oder der Raum zwischen ihnen und uns zunimmt. Die Frage ist dann allerdings, was die Bewegung im ersten Fall beschleunigen lässt, so wie man es beobachtet. Man müsste dann eine Kraft postulieren, die auf die Galaxien abstoßend wirkt, und die mit zunehmender Entfernung wächst und den Raum gleichmäßig erfüllt. Das is ziemlich gegen die Intuition und alle physikalischen Kräfte, die man bisher kennt.

    Ich hab‘ leider weder Zeit, noch Muße, mir die 156 Seiten der Arbeit von Frau Davis reinzuziehen und bin ohnehin mehr interessierter Laie als Fachmann für Kosmologie, aber wenn Du Einzelheiten hast, kann ich gerne meinen Kommentar dazu abgeben. Florian wird sicher auch keine Zeit haben, sich die Doktorarbeit durchzulesen. Sehr kompetent zu dem Thema (vermutlich Profi) ist der Poster Niels, der allerdings schon eine Weile hier nicht mehr geschrieben hat, keine Ahnung, ob er im Moment keine Zeit hat, oder derzeit in anderen Blogs kommentiert. Vielleicht liest er das hier ja zufällig und meldet sich.

  92. @Alderamin

    Um Philosophie geht’s nicht, sondern um 2 äquivalente(?) Betrachtungsweisen, von denen die eine dominiert und die andere nahezu unbekannt ist. Die abstoßend wirkende Kraft – üblicherweise schlicht abstoßende Gravitation genannt – wurde schon längst postuliert (s. lambda-CDM Modell). Es muß keine neue Physik postuliert werden, wenn man eine Koordinatentransformation macht (Beispiel Schwarzes Loch in meinen ersten Beitrag).

    Das Kapitel 4 der Dissertation von Davis umfasst ein paar relativ leicht quer zu lesende Seiten.

    Es mag durchaus Einwände geben. Ich habe danach gesucht, ob jemand die erwähnten Artikel kritisiert, aber nichts gefunden.

  93. @Güntim

    Die abstoßend wirkende Kraft – üblicherweise schlicht abstoßende Gravitation genannt – wurde schon längst postuliert (s. lambda-CDM Modell).

    Aber was wäre die Quelle für die abstoßende Gravitation in einem nicht-expandierenden Raum? Die Masse der Galaxien selbst? Dann müsste die Abstoßung irgendwie mit der Verteilung der Masse korrelieren, und das scheint sie nicht zu tun (z.B. weniger Abstoßung am Rande eines Voids). Die Dunkle Energie hingegen entspringt dem Vakuum, sie ist überall und gleichförmig wirksam, und je mehr Vakuum zwischen zwei Objekten entsteht, desto stärker wird ihr Effekt, der Abstand wächst exponentiell. Das wäre m.E. nach ein Gegenargument für die Arbeit von Frau Davis, aber dazu müsste ich sie besser verstehen, sie geht mir dann doch zu sehr ins Eingemachte.

  94. @Alderamin

    „Aber was wäre die Quelle für die abstoßende Gravitation in einem nicht-expandierenden Raum? Die Masse der Galaxien selbst?“

    Unter abstoßender Gravitation subsummiert man spekulativ eine Vakuumenergie, die man für die Inflation, eine kosmologische Konstante und/oder die Dunkle Energie verantwortlich macht. Sie stößt auf großen Skalen die Massen voneinander ab (das unterscheidet die Modelle Expansion/Relativbewegung nicht). Supercluster wegen der hier überkritischen Materiedichte nicht. Hingegen schon in voids (Materiedichte unterkritisch).

    P.S. Wie schafft man ein Zitat ins graue Feld?

  95. Güntim·
    11.03.12 · 23:02 Uhr

    @Alderamin

    Unter abstoßender Gravitation subsummiert man spekulativ eine Vakuumenergie, die man für die Inflation, eine kosmologische Konstante und/oder die Dunkle Energie verantwortlich macht.

    Ja, im expandierenden Vakuum, aber wie soll das funktionieren, wenn der Raum sich nicht ausdehnt, sondern die Galaxien sich bewegen? Wenn man alles von der Urknalltheorie übernimmt, wo bleibt denn dann noch der Unterschied?

    P.S. Wie schafft man ein Zitat ins graue Feld?

    &ltblockquote&gtZitat&lt/blockquote&gt ergibt

    Zitat

  96. Ja, im expandierenden Vakuum, aber wie soll das funktionieren, wenn der Raum sich nicht ausdehnt, sondern die Galaxien sich bewegen?

    Abstoßende Gravitation bewirkt, daß sich die Galaxien voneinander entfernen. Dies dominiert auf großen Skalen gegenüber der anziehenden Gravitation. Den Befund – Galaxien entfernen sich voneinander – schließt man aus den Rotverschiebungsdaten. Mehr hat man hierzu nicht.

    Nun kommt die Frage nach der Interpretation des Befundes. Entfernen sich die Galaxien voneinander, weil sie sich durch den Raum bewegen (voneinander wegfallen) oder weil sie im Raum ruhen, dieser aber sich zwischen ihnen ausdehnt. Beide Vorstellungen sind, wie es auch Ned Wright in seiner FAQ schön auf den Punkt bringt, äquivalent.

    Demnach wäre es Geschmacksache, welcher Interpretation man zuneigt.

  97. Vom Beobachtungsstandpunkt ist es identisch, ob nun die Galaxien sich bewegen oder der Raum ausdehnt, aber bei der Erklärung der Dunklen Energie macht es schon einen Unterschied.

    Im Standardmodell wirkt keine Kraft auf die Galaxien, sondern der Raum zwischen den Galaxien quillt auf durch seine Vakuumenergie. Es ergibt sich von alleine eine exponentielle Ausdehnung des Raums, wenn ein Raumsegment in einer bestimmten Zeit um einen festen prozentualen Anteil wächst, unabhängig davon, wie viel Materie darin enthalten ist. Das ist die Wrkung einer kosmologischen Konstanten.

    Bei einer Bewegung der Galaxien braucht es jedoch eine Kraft, die die Beschleunigung verursacht. Das Feld, das die Kraft bewirkt, muss eine Quelle haben. Der Raum kann es nicht sein, denn Raum ist auf allen Seiten der Galaxien, etwaige Kräfte würden sich aufheben. Wenn die Materie selbst der Verursacher wäre, sich also zur Gravitation noch eine weitere abstoßende Kraft mir großer Reichweite gesellte (so wie bei Protonen zur anziehenden starken Kernkraft die elektrische Kraft hinzukommt, mit der sich zwei Protonen abstoßen, wenn sie sich nicht nahe genug für die Wirkung der starken Kernkraft kommen), dann wäre die Wirkung dieser Kraft abhängig von der Anwesenheit von Materie, also ihrer Dichte. Dann würde sie bei expandierendem Universum immer kleiner werden, weil sich bei der Expansion die Dichte verringert. Das wird jedoch nicht beobachtet (Supernova-Ia-Messungen).

    In diesem Punkt sehe ich den Unterschied zwischen dem Standardmodell und dem von Frau Davis. Zu der Zeit, als sie ihre Arbeit schrieb, war es noch nicht ausgemacht, ob die Dunkle Energie sich wie eine fünfte Grundkraft oder wie eine kosmologische Konstante verhält. Mittlerweile deutet alles auf die kosmologische Konstante hin. Ich denke, damit ist entschieden, dass sich die Galaxien nicht bewegen, sondern der Raum wächst.

  98. @Alderamin

    Mittlerweile deutet alles auf die kosmologische Konstante hin. Ich denke, damit ist entschieden, dass sich die Galaxien nicht bewegen, sondern der Raum wächst.

    Nein, das ist bereits die Interpretation, wie erläutert. Ich müßte jetzt anfangen mich zu wiederholen. Stattdessen empfehle ich, sich etwas näher mit der angegebenen Literatur zu beschäftigen und die Bedeutung von Koordinatentransformationen nachzuvollziehen. Das macht allerdings wenig Sinn, wenn man von vornherein glaubt, es besser zu wissen, als diese Autoren. Leicht lesbar ist der Artikel von T. Davis im Spektrum Heft 11/10.

  99. @Alderamin:
    „2) Die Zeitdilatation von Ereignissen bei hohem z.“
    Die tritt sowohl bei der kosmologischen Rotverschiebung, als auch beim Dopplereffekt auf.
    Kann es überhaupt Rotverschiebungen geben, bei denen sie nicht auftritt?

  100. @UMa

    Nein, an der Stelle dachte ich noch, es ginge ums Steady-State-Universum mit nicht bewegten Galaxien, bei dem die Rotverschiebung gemeinhin durch Tired Light erklärt wird. Aber Tired Light ist Quatsch, und somit tritt (hohe) Rotverschiebung immer zusammen mit Zeitdilatation auf, egal ob Doppler, kosmologisch oder gravitativ.

    Schade, dass Niels nicht mehr hier ist, der kennt sich super aus mit dem Thema.

  101. @Alderamin
    Ja, als Abonnent umsonst, sonst kostet es eben. Ich weiss nicht mal, ob die Einzelabruf haben, müssten aber. Schade. Mein Netz ist hier langsam, und so sah es so aus … Ich leb jetzt ohne weiter 😉

  102. @Güntim

    Güntim schrieb:
    Nun ist die Vorstellung, das Licht entfernter Galaxien verschiebt rot, weil der Raum sich ausdehnt, zwar weit verbreitet, unter Kosmologen aber nicht völlig unumstritten.

    Doch, das ist völlig unbestritten.
    Du hast nur drei Paper von Autoren gefunden, die der Meinung sind, eine Interpretation der Gleichungen mit Hilfe des Dopplereffekt wäre sinnvoller oder pädagogisch hilfreicher.

    Womit sie eine andere Meinung vertreten als alle Lehrbücher, die so eine Gleichsetzung ausdrücklich als schädlich für das Verständnis ablehnen.

    Güntim schrieb
    :
    Der Kosmologe Ned Wright in FAQ:

    Are galaxies really moving away from us or is space just expanding?
    This depends on how you measure things, or your choice of coordinates. In one view, the spatial positions of galaxies are changing, and this causes the redshift. In another view, the galaxies are at fixed coordinates, but the distance between fixed points increases with time, and this causes the redshift.[…]
    Part 3 of the tutorial shows space-time diagrams for the Universe drawn in both ways.
    […]
    Entfernen sich die Galaxien voneinander, weil sie sich durch den Raum bewegen (voneinander wegfallen) oder weil sie im Raum ruhen, dieser aber sich zwischen ihnen ausdehnt. Beide Vorstellungen sind, wie es auch Ned Wright in seiner FAQ schön auf den Punkt bringt, äquivalent.

    Nein, das will Ned Wright damit bestimmt nicht sagen.
    Wenn man mal in Part 3 guckt und sich dort die Diagramme anschaut, merkt man, dass es Ned Wright hier nur um den Unterschied zwischen den beiden kosmologischen Entfernungen (bzw. Koordinaten) „proper distance“ = Laufzeitentfernung und „comoving distance“ = mitbewegte Entfernung geht.
    (Erkennt man sofort, wenn man die Diagramme bei Wright mit „Figure 1.1“ auf Seite 8 der Dissertation von Tamara Davis vergleicht. [Die Arbeit ist oben von Güntim verlinkt worden.])
    In Laufzeitentfernung wird die Entfernung zwischen uns und anderen Galaxien mit der Zeit immer größer, in mitbewegter Entfernung bleibt die Entfernung per Definition immer gleich.
    Siehe Florians Artikel.

    Mehr meint Ned Wright damit nicht, vor allem, weil er auf dem Rest seiner Seite das Urknallmodell und die Expansion auf die übliche Weise vorstellt.
    Sogar bei den FAQ steht kurz unter der von dir zitierten Stelle:

    What is the Universe expanding into?
    This question is based on the ever popular misconception that the Universe is some curved object embedded in a higher dimensional space, and that the Universe is expanding into this space. Everything that we measure is within the Universe, and we see no edge or boundary or center of expansion.

    Wie soll das mit damit zusammen passen, dass deiner Meinung nach Wright meint, dass es äuqivalent ist, dass sich nur die Galaxien entfernen, aber sich der Raum nicht ausdehnt?
    Würden sich nur die Galaxien entfernen, müsste sie sich doch durch schon vorhandenen Raum bewegen, oder?

    Güntim schrieb:
    Was soll man nun davon halten? Offenbar gibt es kein Experiment, das eine Expansion von Raum beweist und die atronomischen Befunde sind interpretiertbar.

    Das ist komplett falsch. Sobald man das Urknall-Modell vertritt, glaubt man daran, dass der Raum expandiert.

    Die kosmologische Rotverschiebung ergibt sich über
    1+z = [a(jetzt)]/[a(damals)]
    [a(t) ist der Skalenfaktor, z die Rotverschiebung]
    Damit bekommt man aus der Rotverschiebung also eine Angabe, wie stark das Universum expandiert ist.

    Die zahllosen Befunde, die für das Urknall-Modell sprechen, zähle ich jetzt nicht extra auf. Das ist bei Bedarf auch ziemlich leicht zu ergoogeln.
    Momentan sprechen jedenfalls alle atronomischen Beobachtungen dafür, dass das Universum expandiert.

    Auch deine zitierten Quellen gehen vom Urknall-Modell und von der Expansion des Universums aus. Das streitet überhaupt niemand ab.

    Die hast 3 Quellen genannt:

    1) Peacock : „A diatribe on expanding space“

    Finally, some remarks about the relevance of the idea of expanding space to the nature of the redshift. For small redshifts, it is normal to interpret the redshift as a Doppler shift (z = v/c). Even though the idea of ‘expanding space’ might challenge such a view, it connects perfectly with the general idea that 1 + z measures the factor by which the universe expanded between emission and absorption of a photon.

    2) Bunn & Hogg: „The kinematic origin of the cosmological redshift“

    There is no “fact of the matter” about the interpretation of the cosmological redshift: what one concludes depends on one’s coordinate system or method of calculation. Nonetheless, it is instructive to analyze the differing interpretations of the redshift, partly to improve the understanding of cosmology, but more importantly to improve the understanding of general relativity. That analysis leads us to conclude that the most natural interpretation of the redshift is kinematit.

    3) Chodorowski: „The kinematic component of the cosmological redshift“

    A standard interpretation of the cosmological redshift in the framework of the Friedman-Lemaˆıtre-Robertson-Walker (FLRW) models is that it is an effect of the expansion of the Universe. This interpretation is obviously correct since 1 + z = a(to)/a(te), where z
    is the value of the redshift, a(t) is the scale factor of the Universe and te and to are respectively the times of emission and observation of a sent photon.

    Auch diese Kosmologen sind sich also alle einig, dass das Universum expandiert.
    Es geht ihnen nur darum, dass sie die Beschreibung der kosmologischen Rotverschiebung über den Dopplereffekt für „natürlicher“, „passender“ oder „klarer“ halten.

    Güntim schrieb:
    Die abstoßend wirkende Kraft – üblicherweise schlicht abstoßende Gravitation genannt – wurde schon längst postuliert (s. lambda-CDM Modell).

    […]
    Unter abstoßender Gravitation subsummiert man spekulativ eine Vakuumenergie, die man für die Inflation, eine kosmologische Konstante und/oder die Dunkle Energie verantwortlich macht. Sie stößt auf großen Skalen die Massen voneinander ab

    […]
    Abstoßende Gravitation bewirkt, daß sich die Galaxien voneinander entfernen. Dies dominiert auf großen Skalen gegenüber der anziehenden Gravitation. Den Befund – Galaxien entfernen sich voneinander – schließt man aus den Rotverschiebungsdaten.

    Das Universum würde auch dann expandieren, wenn die kosmologische Konstante Null wäre oder das umgekehrte Vorzeichen hätte.
    Die kosmologische Konstante/Dunkle Energie sorgt für die beschleunigte Expansion des Universums, nicht für die Expansion an sich.
    Außerdem sind die meisten Kosmologen der Meinung, dass der Begriff „abstoßende Kraft/Gravitation“ in die Irre führt. Schließlich wirken auf Objekte, die sich mit dem Hubble-Fluss bewegen, dadurch laut ART keine Kräfte.

    Die Inflation in der Frühzeit des Universums ist schließlich etwas anderes und beruht auf dem sogenannten Inflatonfeld.
    Ob das irgendwie mit der kosmologischen Konstante/Dunklen Energie zusammen hängt ist völlig unklar.
    Dafür spricht eigentlich nicht viel, außer des es eben ähnliche Effekte sind.

    Güntim schrieb:
    Interessant ist finde ich in diesem Zusammenhang die Gleichwertigkeit eines expandierenden masselosen FRW-Universums mit dem Milne-Modell, das ein reines SRT Universum ist.

    Da Gravitation nicht vorhanden, sind hier Expansion des Raums und Relativbewegung ohne Einschränkung äquivalent.

    Die Koordinatentransformation ist relativ einfach. Näheres dazu findet sich in der Dissertation von Tamara Davis im Kapitel 4, „The empty Universe“

    Komisch, Davis schreibt doch genau das Gegenteil und widerspricht sogar ausdrücklich dem von dir aufgeführtem Peacock?

    Peacock claims that using the special relativistic Doppler formula to calculate recession velocity from large cosmological redshifts, although generally incorrect, is appropriate in the case of an empty universe.
    We maintain it is not appropriate, even in the empty FRW universe. However, if we have made the change in coordinate systems as outlined in Section 4-1 we can use the special relativistic formula to calculate vM. This is not the velocity that appears in Hubble’s law.

    […]

    In summary, we have shown that Minkowski coordinates (Milne universe) and the Robertson-Walker coordinates (FRW universe) are interchangeable descriptions for the empty universe.
    However, velocities in the Milne universe are not equivalent to velocities in the FRW universe because of the different definitions of time and distance in these two models.

  103. @Alderamin

    Ich hab‘ leider weder Zeit, noch Muße, mir die 156 Seiten der Arbeit von Frau Davis reinzuziehen

    Die ersten beiden Kapitel sind allerdings extrem gut und auch ziemlich leicht zu verstehen. (Die anderen hab ich mir noch nicht angetan.)
    Dort findet man übrigens auch:

    The general relativistic interpretation of the expansion interprets cosmological redshifts as an indication of velocity since the proper distance between comoving objects increases.
    However, the velocity is due to the rate of expansion of space, not movement through space, and therefore cannot be calculated with the special relativistic Doppler shift formula.
    […]
    Despite the fact that special relativity incorrectly describes cosmological redshifts it has been used for decades to convert cosmological redshifts into velocity because the special relativistic Doppler shift formula (Eq. 2.2), shares the same low redshift approximation, v = cz, as Hubble’s Law (Fig. 2.1). It has only been in the last decade that routine observations have been deep enough that the distinction has become significant. Figure 2.1 shows a snapshot of the GR velocity-redshift relation for various models as well as the SR velocity-redshift relation and their common low redshift approximation, v = cz. Present day recession velocities exceed the speed of light in all viable cosmological models for objects with redshifts greater than z ∼ 1.5. At higher redshifts special relativistic “corrections” can be more incorrect than the simple linear approximation on (Fig. 2.4).

    Die ersten beiden Kapitel hat Davis übrigens auch in einem populärwissenschaftlichen Artikel nochmal ohne Formeln dargelegt.
    Hab ich hier bestimmt schon mal verlinkt:
    https://homepage.univie.ac.at/Michael.Berger/lit/urknall.pdf
    Da lohnt es sich, mal kurz den Kasten 4 auf Seite 45 anzuschauen: „Warum gibt es eine kosmische Rotverschiebung?“

    Ich hab also keine Ahnung, warum Güntim ausgerechnet auf diese Arbeit hingewiesen hat.

    Zu den verschiedenen Papern komm ich noch, wenn mir morgen arg langweilig ist. Bisher hab ich nur
    Bunn & Hogg : „The kinematic origin of the cosmological redshift“
    durchgelesen.

    Erinnerst du dich noch, weiter oben hatten wir es darüber, ob und wie die Expansion die Bahnen der Planeten verändert.
    In diesem Paper wird das kurz in einem kleinen Absatz gelöst:

    A student presented with the stretching-of-space description of the redshift cannot be faulted for concluding, incorrectly, that hydrogen atoms, the Solar System, and the Milky Way Galaxy must all constantly “resist the temptation” to expand along with the universe.
    Similarly, it is commonly believed that the Solar System has a very slight tendency to expand due to the Hubble expansion (although this tendency is generally thought to be negligible in practice). Again, explicit calculation shows this belief not to be correct.
    The tendency to expand due to the stretching of space is nonexistent, not merely negligible.

    Schön, dass es so einfach ist. Jetzt ist es eindeutig geklärt.
    Dass es tatsächlich Forscher gibt, die daran forschen, Arbeiten veröffentlichen und auch noch zu einem falschen Schluss kommen.
    Dabei ist es doch offensichtlich völlig trivial. Na ja, es kann halt nicht jeder derart genial sein wie diese beiden Autoren. 😉

    Diese Argumentation hier finde ich auch noch klasse:

    “A driver is pulled over for speeding. The police officer says to the driver, ‘According to the Doppler shift of the radar signal I bounced off your car, you were traveling faster than the speed limit.’
    “The driver replies, ‘In certain coordinate systems, the distance between us remained constant during the time the radar signal was propagating. In such a coordinate system, our relative velocity is zero, and the observed wavelength shift was not a Doppler shift. So you can’t give me a ticket.’ ”
    If you believe that the driver has a legitimate argument, then you have our permission to believe that cosmological redshifts are not really Doppler shifts

    Jetzt bist du natürlich auch überzeugt dass kosmologische Rotverschiebung eigentlich Dopplerverschiebung ist, oder?
    Bleibt einem ja gar nix anderes mehr übrig. 😉

  104. @Niels

    Vielen Dank für die Zeit, die Du Dir für das Thema genommen hast, und insbesondere für die erheiternden Einsichten in die Papers. Eigentlich merkwürdig, dass studierte Fachleute für diesen Themenbereich manchmal zu recht absurden Schlussfolgerungen kommen.

    Was gewinnt man denn dadurch, dass man die kosmologische Rotverschiebung zum Dopplershift erklärt? Was treibt denn dann die beschleunigte Expansion des Universums an? Wo soll die Quelle des antreibenden Kraftfelds sein?

  105. Hi Niels,

    du schreibst: „Schön, dass es so einfach ist. Jetzt ist es eindeutig geklärt.
    Dass es tatsächlich Forscher gibt, die daran forschen, Arbeiten veröffentlichen und auch noch zu einem falschen Schluss kommen.
    Dabei ist es doch offensichtlich völlig trivial. Na ja, es kann halt nicht jeder derart genial sein wie diese beiden Autoren. ;-)“

    Das lässt sich relativ leicht klären. Wenn „a“ der Skalenfaktor ist, dann ist da/dt>0 Expansion, da/dt< Kontraktion. Wenn Expansion einen Effekt auf z.B. die Physik des Sonnensystems hat, dann werden dort Störungsterme auftauchen, die da/dt als dominanten Term enthalten. Wenn dem so ist, hast du Recht und die Professoren Peacock, Bunn & Hogg sind Spinner. Sollten da aber stattdessen Terme mit d²a/dt² auftauchen, dann hätten die Herren Professoren doch mal was richtig gemacht, und deine Reaktion wäre nur ein Paradebeispiel für die von ihnen konstatierte Gefahr, mit der Vorstellung sich dehnenden Raumes auch mal zu falschen Schlüssen zu gelangen. Um Sucherei zu ersparen, hier ein Link auf ein neueres Review zum Thema "Auswirkung der Expansion auf das Sonnensystem": https://arxiv.org/abs/gr-qc/0602098
    Seite 6 unten.

  106. Sorry, der Absatz sollte heißen:

    Das lässt sich relativ leicht klären. Wenn „a“ der Skalenfaktor ist, dann ist da/dt größer 0 Expansion, da/dt kleiner 0 hingegen Kontraktion. Wenn Expansion einen Effekt auf z.B. die Physik des Sonnensystems hat, dann werden dort Störungsterme auftauchen, die da/dt als dominanten Term enthalten. Wenn dem so ist, hast du Recht und die Professoren Peacock, Bunn & Hogg sind Spinner.

  107. @Ich
    Wir hatten genau dieses Thema weiter oben schon mal.
    Siehe z.B. ab dem Kommentar „Niels· 04.01.12 · 15:38 Uhr“.
    Vielleicht liest du das und die darauf folgenden Beiträge erst einmal?

    Dort ging es auch schon kurz um
    Matteo Carrera, Domenico Giulini; Aug 2009: „Influence of global cosmological expansion on local dynamics and kinematics“
    Das ist ein neueres Paper von den beiden Autoren, die auch das von dir verlinkte „Auswirkung der Expansion auf das Sonnensystem“ geschrieben haben.

    Wenn Expansion einen Effekt auf z.B. die Physik des Sonnensystems hat, dann werden dort Störungsterme auftauchen, die da/dt als dominanten Term enthalten.

    Weil andere Ableitungen des Skalenfaktors nichts mit der Expansion zu tun haben?

    Und woher weißt du eigentlich so genau, dass garantiert keine da/dt-Terme auftreten?
    Weil das in einer groben, fragwürdigen newtonschen Näherung so rauskommt?
    Wie das Problem korrekt allgemein relativistisch zu lösen ist, ist aktueller Forschungsgegenstand und ungeklärt.
    Deswegen weiß man auch nicht, wie die korrekten Störungsbeiträge bis zur letzten Ordnung tatsächlich aussehen.
    Darauf weisen Carrera und Giulini übrigens auch extra noch einmal in der Diskussion hin.

    Wenn dem so ist, hast du Recht und die Professoren Peacock, Bunn & Hogg sind Spinner.

    Ich hab Bunn & Hogg nicht als Spinner bezeichnet.
    Und wo kommt Peacock her? Der hat hiermit doch überhaupt nichts zu tun?

  108. @Niels
    Vielen Dank für die reichhaltige Antwort. Ich kann leider aus Zeitgründen nicht auf alles eingehen. Erst mal soviel:

    Du hast nur drei Paper von Autoren gefunden, die der Meinung sind, eine Interpretation der Gleichungen mit Hilfe des Dopplereffekt wäre sinnvoller oder pädagogisch hilfreicher.

    Womit sie eine andere Meinung vertreten als alle Lehrbücher, die so eine Gleichsetzung ausdrücklich als schädlich für das Verständnis ablehnen.

    Weshalb sollte man eine Minderheit nicht ernst nehmen? Peacock ist der Verfasser von „Cosmological Physics“, also nicht Irgendwer. Am Ende des von dir zitierten Absatzes „3 The nature of the the redshift“ Finally, …. schreibt Peacock:

    „To second order it is exactly correct to think of the cosmological redshift as a combination of doppler and gravitational redshift.“

    Es geht um die Gleichwertigkeit zweier Modelle, welcher Autor streitet das ab?
    Das (ohne kosmologische Konstante) expandierende leere FRW-Modell und das Milne-Modell (Explosion in flacher Minkowski Raumzeit) sind gleichwertig. Bei Letzterem ist die Rotverschiebung ausschließlich Dopplereffekt. Gibt man Masse hinzu, dann nicht mehr ausschließlich, wie Peacock vermerkt. Eine gravitative Komponente kommt hinzu.

    Auch diese Kosmologen sind sich also alle einig, dass das Universum expandiert.
    Es geht ihnen nur darum, dass sie die Beschreibung der kosmologischen Rotverschiebung über den Dopplereffekt für „natürlicher“, „passender“ oder „klarer“ halten.

    Sicher, Expansion so verstanden, daß sich die Dinge voneinander enrfernen. Diese Kosmologen heben auf die Gleichwertigkeit von Raumausdehnung und Kinematik ab.

    Ich hab also keine Ahnung, warum Güntim ausgerechnet auf diese Arbeit hingewiesen hat.

    Weil Davis im Spektrum 11/10 schreibt:

    Der leere Raum hat keine physikalische Realität. Wenn Galaxien sich voneinander entfernen, steht es uns darum frei, diese Relativbewegung wahlweise als „Expansion des Raums“ oder als „Bewegung durch den Raum“ zu betrachten.

    Kannst Du ein Gedankenexperiment vorschlagen, das zwischen beiden Modellen unterscheidet?

    Ich hätte eher Einwände erwartet, die mit Details aus dem Leistungsspektrum des CMB argumentieren.

  109. @Güntim

    Wie wär’s damit, dass die SR der Bewegung durch den Raum eine obere Schranke c verpasst? Ohne Raumexpansion gäbe es ein lokalisierbares Zentrum des Universums, von dem der Urknall ausgegangen wäre. Ausgehend von diesem Punkt könnten sich dann die schnellsten Galaxien höchstens mit Lichtgeschwindigkeit entfernen.

    Nun sind die fernsten Galaxien in jeder Richtung völlig symmetrisch verteilt. Wären wir z.B. zufällig an vorderster Front der Ausdehnung, dann hätten wir in Richtung der Expansion nur leeren Raum, keine Galaxien, und auch keine Hintergrundstrahlung. Bei der beobachten völligen Symmetrie müssten wir hingegen das Zentrum des Universums sein. Ziemlich unwahrscheinlich.

  110. @Alderamin

    Wie wär’s damit, dass die SR der Bewegung durch den Raum eine obere Schranke c verpasst? Ohne Raumexpansion gäbe es ein lokalisierbares Zentrum des Universums, von dem der Urknall ausgegangen wäre.

    Der erste Satz ist richtig, der zweite naheliegend, aber falsch. Beide Modelle bewahren das kosmologische Prinzip, Homogenität, kein Rand kein Zentrum. Wobei der Urknall nicht von einem Punkt, sondern aus einer beliebig kleinen Region hervorgeht (beide Modelle). Nachzulesen bei Milne Modell (leicht zu googeln) und bei „The empty universe“, Diss. Davis. S. 59ff. Andernfalls wären sie in der Tat nicht gleichwertig. Auch das Hinzufügen einer Materiedichte und einer kosmologischen Konstante ändert daran prinzipiell nichts.

    Chodorowski in „The kinematik component of the cosmological redshift“, link oben:

    Summing up, the expansion of the universe is never superluminal. A common misconception that the expansion is superluminal is based on the wrong identification of recession velocity with the ‚proper‘ recession velocity.

  111. @Güntim

    Güntim schrieb:
    Weshalb sollte man eine Minderheit nicht ernst nehmen?

    Weil es die Minderheit ist? Wenn man kein ausgewiesener Experte auf einem speziellen Feld ist, fährt man immer besser, wenn man der Mehrheitsmeinung glaubt.

    Außerdem ist die Minderheit hier wirklich nicht besonders groß.
    Wenn man sich mit jeder Minderheitenmeinung beschäftigt, der vier Leute anhängen, kommt man zu gar nichts Anderem mehr.
    Und wenn man nicht mal selbst auf diesem Gebiet forscht, sollte man sich erst mal auf den Mehrheitskonsens und die Lehrbuchmeinung verlassen.
    Bis man die Lehrbuchmeinung richtig verstanden hat ist man auch schon sehr gut beschäftigt.

    Darüber hinaus vertreten diese drei Arbeiten noch nicht einmal die selbe Ansicht.

    Peacock meint, dass man die kosmologsiche Rotverschiebung bis zur zweiten Ordnung als Kombination aus Doppler- und graviativer Rotverschiebung zusammen setzen kann.
    (Wohlgemerkt nur bis zur zweiten Ordnung, also für kleine Rotverschiebungen.
    Für größere Rotverschiebungen werden auch die folgenden Ordnungen bedeutsam.
    Ich sehe also nicht ganz, was uns diese Aufteilung sagen soll.)

    Bunn & Hogg sind der Ansicht, dass man jede Rotverschiebung in jeder beliebigen Raumzeit entweder als reine Doppler-Verschiebung oder als reine gravitative Verschiebung interpretieren kann.

    Chodorowski ist ebenfalls der Ansicht, dass man in eine Kombination aus Doppler- und graviativer Rotverschiebung aufteilen kann, allerdings auf andere Weise als Peacock. Er erwähnt übrigens sowohl Peacock als auch Bunn & Hogg namentlich und widerspricht ihnen.

    It turns out that for small redshifts, the cosmological redshift can be decomposed into a Doppler shift and a Newtonian gravitational one. Can the cosmological redshift be decomposed into a Doppler shift and a gravitational shift (not necessarily Newtonian) for an arbitrary value of the redshift? This is the question which we want to deal with in this Paper. Formally, the answer is no.

    However, Bunn & Hogg have recently pointed out that to settle properly this problem, one has to transport parallely the velocity four-vector of a distant galaxy to the observer’s position. Performing such a transport along the null geodesic of photons arriving from the galaxy, they found that the cosmological redshift is purely kinematic. Here we argue that one should rather transport the velocity four-vector along
    the geodesic connecting the points of intersection of the world-lines of the galaxy and the observer with the hypersurface of constant cosmic time. We find that the resulting relation between the transported velocity and the redshift of arriving photons is not given by a relativistic Doppler formula.

    Man betrachtet hier also nicht nur eine einzige Minderheitenmeinung, sondern gleich drei verschiedene, die sich auch noch gegenseitig widersprechen!
    Da kann man sich jetzt wochenlang den Kopf drüber zerbrechen. Die Chance, dass eine der Meinungen sich als richtig herausstellt ist allerdings so extrem gering, dass sich der Zeitaufwand dafür eigentlich nicht lohnt.
    Wie gesagt, da versucht man lieber die Argumente hinter der Mehrheitsmeinung richtig zu verstehen. Dann erkennt man nämlich höchstwahrscheinlich selbst, warum die meisten Experten dieser Ansicht sind.

    Güntim schrieb:

    Das (ohne kosmologische Konstante) expandierende leere FRW-Modell und das Milne-Modell (Explosion in flacher Minkowski Raumzeit) sind gleichwertig.

    Na ja.
    Beide Modelle liefern unterschiedliche Ergebnisse, die man aber ineinander umrechnen kann.
    Aber warum ist das wichtig? Leben wir in einem leerem Universum? 😉

    Güntim schrieb:

    Bei Letzterem ist die Rotverschiebung ausschließlich Dopplereffekt.

    Dazu sagt Davis, dass das falsch ist:

    Peacock (1999) claims that using the special relativistic Doppler formula to calculate recession velocity from large cosmological redshifts, although generally incorrect, is appropriate in the case of an empty universe.
    We maintain it is no appropriate, even in the empty FRW universe.

    Hatte ich übrigens schon einmal drauf hingewiesen und zitiert.
    Mir erscheint die Argumentation von Davis übrigens logischer, ich traue mir da aber kein abschließendes Urteil zu.
    Das kannst du jedenfalls nicht einfach als zweifelsfreie Tatsache hinstellen.

    Güntim schrieb:
    Gibt man Masse hinzu, dann nicht mehr ausschließlich, wie Peacock vermerkt. Eine gravitative Komponente kommt hinzu.

    Peacock bemerknt, dass eine gravitative Komponente in zweiter Ordnung hinzu kommt.
    Das ist dann eine Näherungsabschätzung.
    In weiteren Ordnungen kommen weitere Terme dazu. Bei hohen Rotverschiebungen wird deutlich, dass man diese weiteren Ordnungen nicht mehr ignorieren darf, weil man sonst völlig falsche Ergebnisse bekommt.
    Aus einer Näherungsabschätzung für kleine Rotverschiebungen ziehst du jetzt ziemlich haarsträubende Schlussfolgerungen.

    Güntim schrieb:
    Diese Kosmologen heben auf die Gleichwertigkeit von Raumausdehnung und Kinematik ab.

    Nein, das tun sie nicht.
    Sie sind der Meinung, dass man die kosmologische Rotverschiebung auch ganz (Bunn & Hogg) oder teilweise (Peacock, Chodorowski) mathematisch als Doppler-Verschiebung beschreiben kann.
    Aus verschiedenen Gründen halten sie diese Sichtweise für lehrreich.

    Güntim schrieb:
    Kannst Du ein Gedankenexperiment vorschlagen, das zwischen beiden Modellen unterscheidet?

    Da braucht es kein Gedankenexperiment. Dazu gibt es schon lange sehr viele echte Beobachtungsdaten.
    Wie Alderamin schon erwähnt hat, beruht die ganze moderne Kosmologie auf der Expansion des Raumes.
    Auf dieser Grundlage beruht das Kosmologische Prinzip und das Kopernikanische Prinzip aus, die beide sehr gut zu allen Beobachtungen passen.
    https://en.wikipedia.org/wiki/Metric_expansion_of_space#Observational_evidence

    Wo kommt den der Raum her, in den deiner Meinung nach die Galaxien hineinfliegen?
    Warum entfernen sich alle Galaxien ausgerechnet von uns weg? Befinden wir uns zufällig im Zentrum des Universums?
    Warum kommt die Hintergrundstrahlung derart isotrop aus allen Richtungen?
    Wie gesagt, googel mal nach Belegen für das Urknallmodell.

  112. @Güntim
    Oh, deinen Kommentar von 18:24 Uhr hatte ich übersehen.

    Beide Modelle bewahren das kosmologische Prinzip, Homogenität, kein Rand kein Zentrum.
    […] Auch das Hinzufügen einer Materiedichte und einer kosmologischen Konstante ändert daran prinzipiell nichts

    Doch, durch das Hinzufügen einer Materiedichte und einer kosmologischen Konstante ändert sich prinzipiell alles!
    Wobei das Hinzufügen eines einzelnen dieser Parameter schon völlig ausreicht.

    Nachzulesen bei Milne Modell (leicht zu googeln)

    Und wenn man das tut, findet man bei Wikipedia:

    Unless the universe modeled has zero density, Milne’s proposal does not follow the predictions of general relativity for the curvature of space caused by global matter distribution, as seen in, for example statistics associated with large-scale structure.

    und

    Even though the Milne model as a special case of a Friedmann-Robertson-Walker universe is a solution to General relativity, the assumption of zero energy content limits its use as a realistic description of the universe. Besides lacking the capability of describing matter Milne’s universe is also incompatible with certain cosmological observations. In particular it makes no prediction of the cosmic microwave background radiation nor the abundance of light elements which are hallmark pieces of evidence that cosmologists agree support Big Bang cosmology over alternatives.

    sowie

    Milne’s proposal directly contradicts the Einstein equations for cosmological scales. Special relativity becomes a global property of Milne’s universe while general relativity is confined to a local property. The reverse is true for standard cosmological models, and most scientists and mathematicians agree that the latter is self-consistent while the former is mathematically impossible.

    Passt nicht ganz zu deinen Behauptungen, oder?

    und bei „The empty universe“, Diss. Davis. S. 59ff.

    Nein, auch hier steht vielmehr das genaue Gegenteil.

    The Milne universe is unable to explain the acceleration and deceleration of
    expansion
    that occurs in a non-empty universe.

    Zu diesen Beobachtungsdaten passt es also ebenfalls nicht.

    This is one of the main differences between Minkowski and FRW coordinates. The Milne universe, as measured in any observer’s inertial frame, is not homogeneous.

    That is why SR is a good approximation to FRW in an infinitesimal region
    surrounding any point.
    However, far from that point the deviation of SR from the GR description becomes dramatic. This is why recession velocities in FRW space do not obey the special relativistic Doppler shift equation, even in the empty universe.

    We have shown in Sect. 3-3 that the SR velocity-redshift relation does not hold even in the limit of an empty FRW universe. Yet the empty universe is the one case in which our SR concepts should apply. Moreover, we have seen in Chapter 2 that many misconceptions about the expansion of the Universe arise from the misapplication of special relativistic concepts. If we are to understand the expansion of the Universe, and in particular the concept of “stretching space”, it is important to understand the relationship between the FRW concept of recession velocities and velocities as they appear in special relativity.

    In ihrer Dokrotrarbeit widerspricht dir Davis eigentlich praktisch in jedem Satz.

    Siehe z.B. auch hier:

    Chodorowski in „The kinematik component of the cosmological redshift“, link oben:
    Summing up, the expansion of the universe is never superluminal. A common misconception that the expansion is superluminal is based on the wrong identification of recession velocity with the ‚proper‘ recession velocity.

    Davis:

    2-1.1 Misconception #1: Recession velocities cannot exceed the speed of light

    A common misconception is that the expansion of the Universe cannot be faster than the speed of light.
    Galaxies that are receding from us superluminally are at rest locally (when their peculiar velocity, vpec = 0) and motion in their local inertial frames remains well described by special relativity. They are in no sense catching up with photons (vpec = c). Rather, the galaxies and the photons (that are directed away from us) are both receding from us at recession velocities greater than the speed of light.

    The general relativistic interpretation of the expansion interprets cosmological redshifts as an indication of velocity since the proper distance between comoving objects increases. However, the velocity is due to the rate of expansion of space, not movement through space, and therefore cannot be calculated with the special relativistic Doppler shift formula.

    Present day recession velocities exceed the speed of light in all viable cosmological models for objects with redshifts greater than z ∼ 1.5. At higher redshifts special relativistic “corrections” can be more incorrect than the simple linear approximation.

  113. Hi Niels,

    lass mich zusammenfassen:

    Du zitierst Bunn&Hogg mit ihrer Aussage (ich kürze aufs Wesentliche:
    stretching-of-space description of the redshift … concluding, incorrectly, that hydrogen atoms … must all constantly “resist the temptation” to expand along with the universe.
    it is commonly believed that the Solar System has a very slight tendency to expand due to the Hubble expansion (although this tendency is generally thought to be negligible in practice). Again, explicit calculation shows this belief not to be correct.

    und machst dich darüber (und den nächsten von dir zitierten Abschnitt) lustig, behauptest, das sei falsch.

    Expanding „along with the Universe“ bedeutet zweifelsfrei, dass bei Expansion des Universums eine Tendenz zur Expansion da sein muss und umgekehrt, bei Kontraktion dann eben eine Tendenz zur Kontraktion. Das kann man nicht missverstehen, hier wird von einer Abhängigkeit von \dot a gesprochen.
    Ebenso bedeutet „Hubble expansion“ unbestritten \dot a größer 0. Weil \dot a kleiner 0 eine „Hubble contraction“ wäre. Die Unterscheidung ist vollkommen unabhängig von \ddot a.

    Dann habe ich dir gezeigt, dass keine solche Abhängigkeit von \dot a existiert, genau wie Bunn und Hogg sagen. Und dass alleine die Tatsache, wie schnell du dich ohne nachzudenken über diese vollkommen richtige Behauptung zweier gestandener Professoren im Fachgebiet lustig machst, ein prächtiger Beweis für genau diese falschen Vorstellungen ist, die sie versuchen zu überwinden.

    Und du meinst dann, dass es egal sei, ob der Effekt nun von \dot a oder \ddot a abhängt. Dass die Berechnung in dem Review Paper falsch sein muss. Und dass es irgendwie relevant sei ob bei einem Effekt, der selber eine 10^-17 oder so Störung ist, noch weitere Ordnungen (die nächste ungerade wäre dann 10^-51!) nicht doch ein \dot a vorkommt?

    Einfach zuzugeben „Stimmt, die haben Recht, da war ich vorailig“ fällt dir nicht ein?

  114. @Niels und Ich:
    Eine naive Rechnung, bei der der Abstand Erde-Sonne mit der kosmologischen Expansionrate da/dt wächst, würde auf eine mittlere Zunahme des Abstands Erde-Sonne um größenordnungsmäßig 11m/Jahr hinauslaufen. Eine derartig große Expansion kann aber nach den Beobachtungen ausgeschlossen werden, siehe z.B.
    „Estimations of changes of the Sun’s mass and the gravitation constant from the modern observations of planets and spacecraft“
    E.V. Pitjeva, N.P. Pitjev
    https://arxiv.org/abs/1108.0246

  115. @Ich
    Schade, dass du dir offenbar nicht die Mühe gemacht hast, oben nachzulesen, was darüber schon geschrieben wurde.

    Expanding „along with the Universe“ bedeutet zweifelsfrei, dass bei Expansion des Universums eine Tendenz zur Expansion da sein muss

    Wenn man das Problem mit einem newtonschen Näherungsansatz untersucht, gibt es laut dem von dir selbst verlinkten Paper doch genau so eine Tendenz:

    As an example, the deviation in the radius for an hypothetical spacecraft orbiting around the Sun at 100 AU would be just of the order of 1 mm.

    Das kann man nicht missverstehen, hier wird von einer Abhängigkeit von \dot a gesprochen.

    Mit \dot ist die erste Ableitung des Skalenfaktors gemeint?

    Warum wird unmissverständlicherweise nur von einer solchen Abhängigkeit gesprochen?

    Ebenso bedeutet „Hubble expansion“ unbestritten \dot a größer 0.

    Expansion bedeutet selbstverständlich, dass das Universum größer wird.
    Damit kann ich Geschwindigkeit der Expansion nicht Null oder kleiner Null sein.
    Und?

    Dann habe ich dir gezeigt, dass keine solche Abhängigkeit von \dot a existiert, genau wie Bunn und Hogg sagen.

    Wie hast du mir das denn gezeigt? Indem du geschrieben hast

    Wenn Expansion einen Effekt auf z.B. die Physik des Sonnensystems hat, dann werden dort Störungsterme auftauchen, die da/dt als dominanten Term enthalten.
    oder wie?

    Wie zitiert vergrößert sich ins unserem Universum das Sonnensystem durch die Expansion, wenn man einem newtonschen Näherungsansatz folgt.
    Obwohl es bei diesem Ansatz keine da/dt-Abhängigkeit „als dominanten Term“ gibt.
    Seltsam, dabei hast du doch angeblich „gezeigt“, dass das nicht sein kann?

    Hier geht es aber wie gesagt immer nur um den newtonschen Ansatz, weil es noch niemand geschafft hat, das Problem im Rahmen der ART zu lösen.
    Wie die Abhängigkeit von den Ableitungen des Skalenfaktors bei korrekter allgemein relativistischer Rechnung aussehen, kann deswegen momentan niemand wissen.

    Und du meinst dann, dass es egal sei, ob der Effekt nun von \dot a oder \ddot a abhängt.

    Nein. Ich meine, dass es im newtonschen Näherungsansatz einen Effekt gibt, obwohl das laut Bunn und Hogg „incorrect“ ist.
    Außerdem meine ich, dass das im Allgemeinen einfach noch nicht geklärt ist.

    Siehe auch das Paper:

    Matteo Carrera and Domenico Giulin:
    However, the general problem of gaining a qualitative and quantitative understanding of how the cosmological dynamics influences local systems remains challenging, with only partial clues being so far provided by exact solutions to the field equations of General Relativity.

    Dass die Berechnung in dem Review Paper falsch sein muss.

    Nicht falsch, sondern eine newtonsche Näherung.
    Wie es auch drübersteht und wie das ganze Kapitel heißt.
    Während das „Review Paper“ selbst klar stellt, dass es für die allgemein relativistische Lösung des Problems nur unvollständige Indizien gibt und damit sowohl das qualitative als auch das quantitative Verständnis herausfordernd bleibt.

    Und dass es irgendwie relevant sei ob bei einem Effekt, der selber eine […] Störung ist, noch weitere Ordnungen […] nicht doch ein \dot a vorkommt?

    Natürlich ist das relevant. Es geht dir schließlich darum, ob es prinzipiell eine Abhängigkeit von der ersten Ableitung des Skalenfaktors gibt oder nicht.
    Ob diese Abhängigkeit groß oder klein ist spielt bei der Frage nach der grundsätzlichen Existenz einer solchen Abhängigkeit doch überhaupt keine Rolle.

    Dass jede Abhängigkeit von der Expansion nur extrem klein sein kann ist doch von Anfang an klar.
    Was tut das zur Sache und warum soll das ein Argument für irgend etwas sein?

    @UMa
    Schon klar. Würde ein „naiver“ Zusammenhang vorliegen, gäbe es schließlich weder die Erde noch das Sonnensystem noch die Milchstraße.

    Es geht Ich aber doch darum, ob es so einen Zusammenhang überhaupt gibt oder eben nicht.

    Das ganze übrigens nur, weil er meint, dass Bunn & Hogg mit der Bemerkung

    Similarly, it is commonly believed that the Solar System has a very slight tendency to expand due to the Hubble expansion (although this tendency is generally thought to be negligible in practice). Again, explicit calculation shows this belief not to be correct.
    The tendency to expand due to the stretching of space is nonexistent, not merely negligible.

    zwangsläufig nur über eine Anhängigkeit von da/dt sprechen, weswegen jede andere tatsächlich vorliegende Abhängigkeit überhaupt keine Rolle spielt.

    Siehst du das eigentlich auch so?
    Meinen Bunn & Hogg das zweifelsfrei so, wie es Ich versteht?

  116. Hi Niels,

    lass mich mal eins voranstellen: Ich bin ziemlich irritiert von deinem Beitrag.

    Wie zitiert vergrößert sich ins unserem Universum das Sonnensystem durch die Expansion, wenn man einem newtonschen Näherungsansatz folgt.
    Obwohl es bei diesem Ansatz keine da/dt-Abhängigkeit „als dominanten Term“ gibt.
    Seltsam, dabei hast du doch angeblich „gezeigt“, dass das nicht sein kann?

    Ich will mal übersetzen:
    Jemand behauptet, das Roulette folge dem Mond. Bei Vollmond käme Rot, bei Neumond Schwarz.
    Ich zeige anhand von Studien, dass es keine solche Abhängigkeit gibt.
    Dann sagst du: Guckstu, jetzt ist Rot gekommen, und es ist Vollmond. Seltsam, dabei hast du doch angeblich „gezeigt“, dass das nicht sein kann?

    Was soll das? Schreibst du für Publikum? Oder hältst du das für ein wissenschaftliches Argument?
    Gut, es ist immer möglich, dass ich etwas missverstehe oder einfach nicht auf die gutgemeinten Intentionen komme, die einem solchen „Argument“ zugrundeliegen mögen. Dann wäre es äußerst hilfreich (=erforderlich), wenn du mich in deine Motivation für diese Aussage einweihen würdest. Ich kann das ohne diese Info nur als Diskussionsverweigerung auslegen, mir fehlen da hoffentlich ein paar Teile im Puzzle.

    Zur Ergänzung:
    Wie du richtig sagst, bedeutet „Expansion“, dass sich das Universum vergrößert. Das ist eineindeutig, d.h. aus „Expansion“ folgt „Universum vergrößert sich“ und umgekehrt. Was wiederum eineindeutig mit \dot a zusammenhängt. Was wiederum genau das ist, was ich die ganze Zeit sage.
    Die von mir extra markierten Fragmente
    „temptation to expand along with the universe“
    „tendency to expand due to the Hubble expansion“
    behaupten also einen direkten Zusammenhang
    Expansion = \dot a größer 0 = temptation, tendency to expand.
    Vielleicht liegt da auch ein Problem mit der Sprachbarriere vor, für den Fall sei gesagt, dass „along with“ eine starke Korrelation meint und „due to“ sogar eine kausale Abhängigkeit.
    Beides ist offensichtlich nicht der Fall, womit bewiesen ist, dass die Herren da Recht haben.

    Das war mein letzter Versuch, das zu erklären. Mir fällt sonst nichts mehr ein, was ich nicht schon gesagt hätte.

    Würde mich freuen, wenn wir doch noch auf einen grünen Zweig kämen. Du bist aber am Zug. Jetzt.

  117. @Ich
    ???

    Ich zeige anhand von Studien, dass es keine solche Abhängigkeit gibt.

    Du hast ein einziges Paper verlinkt, in dem es um „Auswirkung der Expansion auf das Sonnensystem“ geht.
    Dieses Paper kommt zum Ergebnis, dass es bei Verwendung eines Newtonschen Ansatzes einen solchen Einfluss gibt, der aber unglaublich winzig ist.
    Als Beispiel berechnen die Autoren, dass die Expansion den Radius eines in einer Entfernung von 100 AU um die Sonne kreisenden Raumschiffes um einen Millimeter vergrößert.

    Du behauptest jetzt, dass du mit der Verlinkung dieses Papers bewiesen hast, dass die Expansion keinen Einfluss hat, also insbesondere Radien nicht vergrößert oder verkleinert werden?

    Obwohl die Autoren diese angeblich nicht vorhandene Vergrößerung sogar berechnet habe und als Zahlenwert angeben?

    Zu dem Schluss, dass die Expansion absolut keinen Einfluss hast kommst du, weil beim Newtonschen Ansatz kein da/dt -Störungsterm zu den klassischen, aus der newtonsche Gravitationstheorie hergeleiteten Bewegungsgleichungen dazukommt?

    Ich verstehe überhaupt nicht, was das für eine Überlegung ist.
    Das seltsame Zeug über Roulette bei Vollmond hilft auch überhaupt nicht weiter.

    Wie du richtig sagst, bedeutet „Expansion“, dass sich das Universum vergrößert. Das ist eineindeutig, d.h. aus „Expansion“ folgt „Universum vergrößert sich“ und umgekehrt. Was wiederum eineindeutig mit \dot a zusammenhängt. Was wiederum genau das ist, was ich die ganze Zeit sage.

    Damit hast du etwas völlig triviales extrem kompliziert beschrieben:

    Wenn das Universum expandiert, dehnt es sich aus.
    Das ist eigentlich nicht mal eine Folgerung, sondern nur eine Übersetzung des Fremdwortes Expansion.

    Das hat aber niemand bestritten?

    Die von mir extra markierten Fragmente […]
    behaupten also einen direkten Zusammenhang Expansion = \dot a größer 0 = temptation, tendency to expand.

    Nein, das sagen diese Fragmente nicht.

    Sie sagen „Expansion = temptation, tendency to expand“.

    Die dritte Gleichheitsbeziehung mit „\dot“ hast du einfach grundlos reininterpretiert.

    Wenn man behauptet, dass in unserem Universum die Expansion des Universums nicht zu einer Expansion des Sonnensystem führt, bezieht man sich doch nicht nur auf eine da/dt-Abhängigkeit, sondern auf die Expansion als Ganzes.

    Zur Expansion gehören aber natürlich alle Ableitungen des Skalenfaktors, nicht nur die Erste.

    Schnallst du dich beim Autofahren auch nicht an, weil bei der Bewegung des Autos nur die erste Ableitung, die Fahrtgeschwindigkeit, eine Rolle spielt?

    Vielleicht liegt da auch ein Problem mit der Sprachbarriere vor, für den Fall sei gesagt, dass „along with“ eine starke Korrelation meint und „due to“ sogar eine kausale Abhängigkeit.
    Beides ist offensichtlich nicht der Fall, womit bewiesen ist, dass die Herren da Recht haben.

    Es gibt selbstverständlich einen eindeutigen kausalen Zusammenhang zwischen der Expansion und der Vergrößerung des Sonnensystem.

    In newtonscher Näherung ist das Sonnensystem nämlich deswegen vergrößert, weil es eine Expansion gibt, die nicht konstant ist.

    Die Expansion ist also die Ursache der Vergrößerung.

    Warum soll es also offensichtlich keine „starke Korrelation“ und keine kausale Abhängigkeit geben?
    Verlangst du eine stärke Korrelation als eine direkte Kausalbeziehung?

    Aber selbst wenn man dir recht gibt und die Bunn & Hogg tatsächlich mit dem zur Diskussion stehenden Absatz

    „Eine unbeschleunigte Expansion führt nicht zu einer Vergrößerung des Sonnensystems.
    Das zeigen eindeutige/explizite Rechnungen.“

    meinen sollten, wäre das trotzdem bestenfalls sehr unklar ausgedrückt und streng genommen falsch.
    Weil es solche eindeutigen Rechnungen eben überhaupt nicht gibt, sondern momentan nur eine newtonsche Näherungsabschätzung.

    Die kannst ja mal bei ein paar anderen Problemen der allgemeinen Relativitätstheorie Newtonsche Abschätzungen durchführen.
    Dann siehst du, dass das praktisch durchgängig absoluten Quatsch liefert.

    Auf solche Probleme weisen die Autoren der von dir verlinkten Studie auch selbst hin:
    Note that, in the sense of General Relativity, a body that is co-moving with the cosmological expansion is moving on an inertial trajectory, i.e. it moves force free.
    Forces in the Newtonian sense are now the cause for deviations form the co-moving acceleration

    In der newtonschen Betrachtung gibt es also Kräfte, die es in der richtigen ART-Beschreibung nicht gibt.
    Außerdem ist die ART nichtlinear, der ganze Ansatz einer Aufspaltung in einen äußeren Störungsterm ist also eigentlich komplett illegal.
    So etwas geht selbstverständlich nur bei linearen DGLs wie etwa bei den Maxwell-Gleichungen.

    Vielleicht hat man in diesem Fall aber trotzdem Glück. Aber das kann momentan niemand wissen.

    Einfach zuzugeben „Stimmt, die haben Recht, da war ich vorailig“ fällt dir nicht ein?

    […]

    Was soll das? Schreibst du für Publikum? Oder hältst du das für ein wissenschaftliches Argument?

    […]
    Ich kann das ohne diese Info nur als Diskussionsverweigerung auslegen
    […]

    Das war mein letzter Versuch, das zu erklären. Mir fällt sonst nichts mehr ein, was ich nicht schon gesagt hätte.
    Würde mich freuen, wenn wir doch noch auf einen grünen Zweig kämen. Du bist aber am Zug. Jetzt.

    Wenn man diese Bemerkungen in deinem letzten Post, das Zeug über den Vollmond sowie die Spekulationen über meine Motivation streichst, bleibt höchstens noch ein Drittel deines Beitrags übrig.
    Was soll das bringen?

    Es geht doch hoffentlich auch ein bisschen freundlicher?
    Etwas weniger aggressiv, gönnerhaft und arrogant wäre auch noch sehr schön.

    Danke.

  118. „Es geht doch hoffentlich auch ein bisschen freundlicher?
    Etwas weniger aggressiv, gönnerhaft und arrogant wäre auch noch sehr schön. “

    Wage ich zu bezweifeln….

    Aber wenn ich als 08/15 Laie jemanden folgen würde in seinen ausführungen ist das Niels

  119. Hi Niels,

    Ich:
    Wie du richtig sagst, bedeutet „Expansion“, dass sich das Universum vergrößert. Das ist eineindeutig, d.h. aus „Expansion“ folgt „Universum vergrößert sich“ und umgekehrt. Was wiederum eineindeutig mit \dot a zusammenhängt. Was wiederum genau das ist, was ich die ganze Zeit sage.
    Du:
    Damit hast du etwas völlig triviales extrem kompliziert beschrieben
    Und ein paar Zeilen später:
    Sie sagen „Expansion = temptation, tendency to expand“.
    Die dritte Gleichheitsbeziehung mit „\dot“ hast du einfach grundlos reininterpretiert.

    Erst ist die Gleichsetzung \dot a mit Expansion trivial, ja sogar nur eine Übersetzung des Wortes. Einen Absatz später ist sie „grundlos reininterpretiert“. Und das lustigerweise auch noch in einm Zusammenhang, wo die vollkommen offensichtliche aber einfach grundlose Interpretation einem anerkannten, korrekten und veröffentlichten Paper zweier Professoren einen Sinn verleiht. Wie soll das weitergehen, wenn’s mal weniger eindeutig ist?

    Was soll das bringen?

    Ganz einfach: ich sehe nach nur drei Beiträgen, ob eine weitere Diskussion sinnlos ist, nicht erst nach 30 Seiten fruchtlosen Austauschs. Spart beiden eine Menge Mühe.

    Du musst das auch gar nicht als aggressiv, gönnerhaft und arrogant auslegen. Ich will mich eben nicht mit dir streiten, deswegen bin ich nach diesem kurzen Test auch wieder weg.

    Noch ein Lesetipps für dich: https://de.wikipedia.org/wiki/St%C3%B6rungstheorie_(Allgemeine_Relativit%C3%A4tstheorie)
    Wenn’s dich etwas nachdenklich macht, wie weit man auch bei sicher geglaubtem Wissen danebenliegen kann, dann wär’s gut. Das wäre sowas wie ein positives Ergbnis dieser Diskussion.

    Gruß,
    Ich

  120. Niels:

    Güntim schrieb: Kannst Du ein Gedankenexperiment vorschlagen, das zwischen beiden Modellen unterscheidet?

    Niels: Da braucht es kein Gedankenexperiment. Dazu gibt es schon lange sehr viele echte Beobachtungsdaten.

    Es gibt ein solches Gedankenexperiment nicht. Wie sollte man auch ‚mitbewegt‘ von ‚durch den Raum bewegt‘ unterscheiden können, wie Davis in dem besagten Spektrum Artikel völlig richtig anmerkt.

    Niels: Das ist komplett falsch. Sobald man das Urknall-Modell vertritt, glaubt man daran, dass der Raum expandiert.

    Es spricht ja nichts dagegen, das dem mainstream folgend zu glauben. Man kann diese oder eine kinematische Interpretation bevorzugen. Es geht um die Nichtunterscheidbarkeit.

    Davis S.59: Importantly, the Milne universe obeys the cosmological principle: it looks the same for every observer.
    Und S.65: In summary we have shown that Minkowski coordinates (Milne universe) and the Robertson-Walker coordinates (FRW universe) are interchangeable descriptions for the empty universe.

    Diese Gleichwertigkeit der Beschreibung ist nicht auf das leere Universum beschränkt. Das zeigen die nun schon vielfach erwähnten Autoren, auch wenn sie sich im Detail unterscheiden. Um mehr geht es nicht.

    Ich muß mich ausklinken, bin knapp an Zeit. Danke für die Mühe.

  121. @Güntim

    Diese Gleichwertigkeit der Beschreibung ist nicht auf das leere Universum beschränkt.

    Davis ist wie wie schon mehrfach zitiert der Meinung, dass diese Gleichwertigkeit sogar im Fall des leeren Universums fragwürdig ist. Sie erwähnt Peacock namentlich, um ihm zu widersprechen. Diese Stelle habe ich aber schon mindestens drei mal zitiert.
    (Wobei ich sowieso nicht weiß, was Fluchtgeschwindigkeiten in einem leeren Universum überhaupt darstellen sollen.
    Wenn es in diesem im Universum absolut nichts gibt, was hat dann eine Geschwindigkeit und in Bezug worauf bewegt es sich?)

    Bei einem nicht-leeren Universum gibt es diese Gleichwertigkeit aber auf gar keinen Fall!
    Der Wikipedia-Eintrag zum „Milne model“ widerspricht deiner Behauptung in praktisch jedem Satz.
    Ich hab das zwar oben schon alles zitiert, aber lies doch einfach mal direkt bei der Wiki nach.
    https://en.wikipedia.org/wiki/Milne_model

    Das zeigen die nun schon vielfach erwähnten Autoren, auch wenn sie sich im Detail unterscheiden.

    Wie schon geschrieben, meiner Meinung nach geht es den Autoren der drei Paper um etwas ganz anderes.
    Die Unterschiede sind auch ziemlich groß, das sind nicht nur Details.

    Ob sie überhaupt irgend etwas gezeigt haben ist ziemlich fragwürdig.
    Schließlich hat sich die absolute Mehrheit der Experten nicht überzeugen lassen. Von keiner der drei verschiedenen Positionen.
    Für all diese Menschen zeigen diese drei Paper also gar nichts.
    Auch die seit Veröffentlichung der Paper geschriebenen Lehrbücher vertreten weiterhin die bisherige Lehrmeinung.

    Es gibt ein solches Gedankenexperiment nicht. Wie sollte man auch ‚mitbewegt‘ von ‚durch den Raum bewegt‘ unterscheiden können[…]

    In den Zitaten in meinem letzten Post an dich sind ganz viele aus Beobachtungen gewonnene Erkenntnisse über das Universum genannt, die klar zeigen, dass man beides durchaus unterschieden kann.

    Im Ergebnis kann das Urknall-Modell mit „Mitbewegung“ alles korrekt beschreiben, das Milne-Modell mit „Bewegung durch den Raum“ scheitert dagegen sehr häufig und bei grundlegenden Dingen..
    Nochmal ganz kurz:

    • Im Milne Modell gibt es die Beschleunigung und die Abbremsung der Expansion des Universums nicht. Das Milne Modell kann deren Auftreten also nicht erklären.
    • Das Milne Modell kann die Häufigkeitsverteilung der leichten chemischen Elemente nicht erklären. Bei diesem Modell ist absolut unklar, warum das Massenverhältnis von Helium zu Wasserstoff gerade 1:4 war. Aus dem Urknallmodell dagegen errechnet man genau dieses Verhältnis. Auch die relativen Häufigkeiten von Deuterium, Helium3 und Lithium für das frühe Universum werden von der Urknall-Theorie sehr gut erklärt, während das Milne Modell dazu überhaupt keine Aussagen macht.
    • Das Milne Modell kann nicht erklären, wie es möglich sein kann, dass man aus der Rotverschiebung Fluchtgeschwindigkeiten von Galaxien größer als die Lichtgeschwindigkeit errechnet.
      Das ist in der globalen SRT-Welt des Milne Modells nämlich eigentlich unmöglich.
    • Das Milne Modell sagt die großskaligen Strukturen der Materie (large-scale structure) falsch voraus.
    • Im Milne Modell gibt es keine kosmische Hintergrundstrahlung!

    Das sind jetzt nur die Punkte, die ich vorgestern nach fünf-minütigem Googeln gefunden habe und die im Post von vorgestern auch schon alle erwähnt wurden.

    Außerdem gibt es noch sehr bedeutende theoretische Widersprüche:

    • Wenn sich nur die Galaxien bewegen, die Raumzeit aber nicht expandiert, bekommt man eine Menge Schwierigkeiten. Wo kommt beispielsweise der Raum her, in dem sich die Galaxien im Milne-Modell bewegen? Ist dieser Raum begrenzt? usw.
    • Beim Urknallmodell beschreibt man die Raumzeit global über die ART als vierdimensionale Mannigfaltigkeit. In frei fallenden Bezugssystemen gilt die SRT, weil so eine Mannigfaltigkeiten lokal auf den euklidischen Raum abgebildet werden kann.
      Wie das funktioniert, beschreibt das mathematische Teilgebiet der Differentialgeometrie. Dort wird auch bewiesen, dass dieses Vorgehen mathematisch möglich ist.
      Beim Milne-Modell beschreibt man die Raumzeit global mit der SRT und lokal mit der ART.
      Es ist völlig unklar, wie das mathematisch funktionieren soll.
      Die meisten Mathematiker und Physiker sind sich sogar einig, dass das mathematisch unmöglich ist.

    Nach all diesen schon vorgestern aufgeführten Punkten behauptest du immer noch, dass beide Theorien gleichwertig sind?
    Dass es keine Möglichkeit gibt, zwischen ihnen zu unterscheiden?
    Dazu fällt mir nichts mehr ein. Deswegen habe ich hier auch nur den Inhalt meine alten Posts wiederholt. 😉

    Wie sollte man auch ‚mitbewegt‘ von ‚durch den Raum bewegt‘ unterscheiden können, wie Davis in dem besagten Spektrum Artikel völlig richtig anmerkt.

    Den Spektrum-Artikel kenne ich nicht, aber in ihrer Doktorarbeit schreibt Davis mehrmals, dass das sehr wohl möglich ist.
    Sie ist ebenfalls davon überzeugt, dass Mitbewegung der richtige Standpunkt ist.
    Da hab ich ebenfalls schon einmal alle wichtigen Stellen zitiert.

    Aber hier nochmal ganz kurz:
    However, the velocity is due to the rate of expansion of space, not movement through space

    Sie erwähnt außerdem einen der Punkte, wie man „‚mitbewegt‘ von ‚durch den Raum bewegt‘ unterscheidet“:
    The Milne universe is unable to explain the acceleration and deceleration of
    expansion that occurs in a non-empty universe.

    Auch diese Zitate habe schon einmal in älteren Antworten gebracht.
    Wie man nach dem Lesen der vielen, vielen Zitate von David immer noch der Meinung sein kann, dass für sie herkömmliche Expansion und Bewegung im Raum gleichwertig sind, ist mir echt schleierhaft.

    Zusammenfassend hätte ich eigentlich auch sagen einfach können:
    Lies bitte nochmal meine alten Antworten.
    In diesem Beitrag steht absolut nichts eigentlich Neues. Das wurde alles schon mindestens einmal gesagt, sogar jeweils mit ausführlicheren Zitaten.

    Sorry, aber ich glaube wir stecken fest.

  122. @Ich

    ich sehe nach nur drei Beiträgen, ob eine weitere Diskussion sinnlos ist, nicht erst nach 30 Seiten fruchtlosen Austauschs. Spart beiden eine Menge Mühe.

    Bist du sicher, dass wir eine Diskussion geführt haben? Aufgrund deiner bisherigen Beiträge kann man das nämlich bezweifeln.
    Darauf, mir 30 Seiten Beleidigungen von dir durchzulesen, hätte ich mich übrigens sowieso nicht eingelassen. Du musst du dir keine Sorgen machen.

    In deinem ersten Beitrag ging es darum,
    dass ich es deiner Meinung nach wage, mich ohne nachzudenken über diese vollkommen richtige Behauptung zweier gestandener Professoren im Fachgebiet lustig zu machen.
    Wenn ich sofort wie gefordert meinen „Fehler“ bereut hätte, wäre die „Diskussion“ vorbei gewesen.
    Na ja, vielleicht hättest du dann noch eine Entschuldigung und die offizielle Anerkennung deiner Überlegenheit gefordert?
    (Warum ist dir eigentlich dieser völlig unwichtige, kurze Nebensatz aus der Einführung des Papers der gestandenen Professoren so unfassbar wichtig?
    Die Hauptthematik ist dir nämlich offenbar vollkommen egal. Seltsam.)

    In deinem zweiten Beitrag ging es um Folgendes:

    Einfach zuzugeben „Stimmt, die haben Recht, da war ich vorailig“ fällt dir nicht ein?

    In deinem dritten Beitrag um

    Jemand behauptet, das Roulette folge dem Mond. Bei Vollmond käme Rot, bei Neumond Schwarz.
    Ich zeige anhand von Studien, dass es keine solche Abhängigkeit gibt.
    Dann sagst du: Guckstu, jetzt ist Rot gekommen, und es ist Vollmond. Seltsam, dabei hast du doch angeblich „gezeigt“, dass das nicht sein kann?
    […]
    Was soll das? Schreibst du für Publikum? Oder hältst du das für ein wissenschaftliches Argument?
    […]
    Ich kann das ohne diese Info nur als Diskussionsverweigerung auslegen
    […]
    Das war mein letzter Versuch, das zu erklären. Mir fällt sonst nichts mehr ein, was ich nicht schon gesagt hätte.
    Würde mich freuen, wenn wir doch noch auf einen grünen Zweig kämen. Du bist aber am Zug. Jetzt.

    In deinem vierten Beitrag um

    Wie soll das weitergehen, wenn’s mal weniger eindeutig ist?
    […]
    Ganz einfach: ich sehe nach nur drei Beiträgen, ob eine weitere Diskussion sinnlos ist, nicht erst nach 30 Seiten fruchtlosen Austauschs. Spart beiden eine Menge Mühe.
    Du musst das auch gar nicht als aggressiv, gönnerhaft und arrogant auslegen. Ich will mich eben nicht mit dir streiten, deswegen bin ich nach diesem kurzen Test auch wieder weg.
    […]
    Wenn’s dich etwas nachdenklich macht, wie weit man auch bei sicher geglaubtem Wissen danebenliegen kann, dann wär’s gut. Das wäre sowas wie ein positives Ergbnis dieser Diskussion.

    Auf meine Antworten bist du nie eingegangen.
    Du hast exakt ein Argument geliefert, das steht schon im ersten Beitrag und ist denkbar kurz:

    Wenn Expansion einen Effekt auf z.B. die Physik des Sonnensystems hat, dann werden dort Störungsterme auftauchen, die da/dt als dominanten Term enthalten.

    Als Erklärung, warum es ausschließlich um da/dt gehen soll und nicht auch um weitere Ableitungen, schreibst du:

    Das kann man nicht missverstehen, hier wird von einer Abhängigkeit von \dot a gesprochen.

    Diese Erklärung wiederholst du dann noch einmal, ohne dass etwas Neues dazukommt.
    (Außer das aus der Expansion des Universums eine Vergrößerung folgt. Klasse.)
    Auf meine Beiträge gehst du überhaupt nicht ein.
    Das war es dann auch schon an Inhalt.
    Der ganze Rest deiner Äußerungen besteht aus aggressiven, gönnerhaften und arroganten Bemerkungen. An deinem gesamten hier veröffentlichten Textvolumen macht das wahrscheinlich 80% aus.

    Und das war dann deiner Meinung nach eine Diskussion? Sogar eine, die „sowas wie ein positives Ergbnis“ für mich haben soll?
    Für dich war es ja vielleicht ein positives Ergebnis, weil du dir jetzt möglicherweise einreden kannst, dass du mit deinem Ausstieg „gewonnen“ hast, weil dein Gesprächspartner dir unfassbar unterlegen war.
    Mir hat es absolut überhaupt nichts gebracht.

    Ich kann mich zum Thema eigentlich nur nochmal wiederholen:

    Im newtonschen Näherungsansatz führt die beschleunigte Expansion zu einer Vergrößerung des Sonnensystems.
    Du kannst dir im Paper von Carrera und Giulini den entsprechenden Zahlenwert und die Rechnung anschauen.
    Und das, obwohl da/dt nicht als dominanten Term enthalten ist.

    Davon abgesehen ist das wie gesagt eigentlich nicht so furchtbar bedeutsam, weil für die komplette, allgemein relativistische Betrachtung gilt:

    the general problem of gaining a qualitative and quantitative understanding of how the cosmological dynamics influences local systems remains challenging, with only partial clues being so far provided by exact solutions to the field equations of General Relativity.

    Es gibt nur „partial clues“ aufgrund von „exact solutions“ der Feldgleichungen für Spezialmetriken, wie die McVittie-Metrik.
    Dort steht nirgends, dass der newtonschen Näherungsansatz das Problem schon gelöst hat. Vielmehr kann man aus diesem Zitat schließend, dass dieser Ansatz nicht einmal „partial clues“ für das allgemeine Problem liefert.

  123. Niels, es ist ja völlig richtig, Das Milne Modell kann nicht alles, insbesondere nicht das, wofür es nicht geschaffen ist. Dies als pars pro toto:

    Niels: ■Das Milne Modell kann die Häufigkeitsverteilung der leichten chemischen Elemente nicht erklären.

    Welcher Kosmologe oder Diskussionsteilnehmer soll damit widerlegt werden? Wer behauptet, daß ein masseloses Modell (wie leicht nach zu lesen, enthält es masselose Testpartikel, um deren Rotverschiebung es u.a. geht) Aussagen über Materie und im Speziellen über die primordiale Nucleosynthese macht? Diese Argumentation ist unverständlich.
    Sie steht darüber hinaus in keinerlei Zusammenhang mit der Frage, ob oder ob nicht die kosmologische Rotverschiebung eine kinematische Komponente hat.

    Der Wert dieses Modells liegt, wie schon erläutert woanders, beispielhaft hierzu Chodorowski in „Cosmology Under Milne’s Shadow“:
    https://www.publish.csiro.au/?act=view_file&file_id=AS05016.pdf

    Though not a viable alternative to the currently favoured model, the Milne model has great pedagogical value, elucidating the kinematic aspect of the universe’s expansion.

    Der Wert dieses Modells läßt sich u.a. auch aus der nicht perfekt aber erstaunlich genau übereinstimmenden Vorhersage der SN Ia Rotverschiebungsdaten ermessen, s. Fig 1 in diesem Artikel. Schließlich, auf die Äquivalenz (Koordinatentrafo) Milne – FRW leer wurde bereits hingewiesen.

    However, the velocity is due to the rate of expansion of space, not movement through space

    Wo genau schreibt Davis das?

    Den Spektrum-Artikel kenne ich nicht

    Dann empfehle ich ihn zu lesen. Denn dort schreibt Davis, wie schon zitiert von der Äquvivalenz „Expansion des Raumes“ mit „Bewegung durch den Raum“ in unserem Universum. In einem netten Bildchen zeigt sie den Paralleltransport der 4-Geschwindigkeit entlang einer Photon Geodäte.

    Es ist die einzige mir bekannte populärwissenschafliche Publikation, die dieses heiße Eisen aufgreift. Ich war zunächt ablehnend, habe mich aber dann im wissenschaftlichen Bereich umgesehen. Aus alledem schließe ich, daß man eine kinematische Komponente der kosmologischen Rotverschiebung nicht in Bausch und Bogen ablehnen sollte. Niemand muß deshalb auf das Bild der Raumexpansion verzichten.

    Sorry, aber ich glaube wir stecken fest.

    Na, hier haben wir volle Übereinstimmung.

  124. @Güntim

    Den Spektrum-Artikel kenne ich nicht

    Dann empfehle ich ihn zu lesen. Denn dort schreibt Davis, wie schon zitiert von der Äquvivalenz „Expansion des Raumes“ mit „Bewegung durch den Raum“ in unserem Universum.

    Der ist wie schon viele Kommentatoren erzählt und mit Link belegt haben hinter einer Bezahlschranke.
    Ich bin mir ziemlich sicher, dass dieser eine Satz aus dem Zusammenhang gerissen ist. Geld möchte ich aber nicht ausgeben, um mich davon zu überzeugen.
    Kannst du den Artikel hochladen oder mir schicken?

    Wer behauptet, daß ein masseloses Modell (wie leicht nach zu lesen, enthält es masselose Testpartikel, um deren Rotverschiebung es u.a. geht) Aussagen über Materie und im Speziellen über die primordiale Nucleosynthese macht?

    Du. Das steht zum Beispiel hier:

    Diese Gleichwertigkeit der Beschreibung ist nicht auf das leere Universum beschränkt.

    Es ist es zudem nun mal Materie, die eine Fluchtgeschwindigkeit hat.
    Masselose Testpartikel sind doch ziemlich offensichtlich eine ungeeignete Wahl, wenn es um Folgerungen aus Gravitationstheorien geht.

    Sie steht darüber hinaus in keinerlei Zusammenhang mit der Frage, ob oder ob nicht die kosmologische Rotverschiebung eine kinematische Komponente hat.

    Doch.
    Das Milne-Modell kann man entweder ganz für unser Universum verwenden oder gar nicht.
    Einen einzelnen Punkt wie die Rotverschiebung herauszugreifen ist doch sinnlos, wenn alles andere überhaupt nicht passt.

    Außerdem hast du jetzt ein Argument von den sieben aufgezählten Argumenten aufgegriffen.
    Bei fünf davon spielt Materie überhaupt keine Rolle.

    Das Milne Modell kann nicht alles, insbesondere nicht das, wofür es nicht geschaffen ist.

    Ganz genau. Es kann vor allem kein nicht-leeres Universum beschreiben.
    Sobald Materie vorhanden ist, macht es völlig falsche Vorhersagen.

    Wo genau schreibt Davis das?

    Wie schon mehrmals gesagt, übrigens auch fett markiert in meinem letzten Post, in ihrer von dir verlinkten Doktorarbeit.

    Der Wert dieses Modells läßt sich u.a. auch aus der nicht perfekt aber erstaunlich genau übereinstimmenden Vorhersage der SN Ia Rotverschiebungsdaten ermessen

    Die Nutzlosigkeit dieses Modells lässt sich daran ermessen, dass praktisch alle anderen Vorhersagen nicht mit den Beobachtungsdaten über unser Universum übereinstimmen.
    Mit den Vorhersagen des Urknall-Modells, in dem der Raum expandiert, dagegen schon.

    the Milne model has great pedagogical value

    Das sehen die meisten anderen Forscher und Lehrbücher allerdings komplett anders und warnen ausdrücklich davor, sich die Entfernung der Galaxien als Bewegung durch den Raum vorzustellen.
    Wie erwähnt gibt es dafür auch gute Argumente.

  125. @Niels

    Ich bin mir ziemlich sicher, dass dieser eine Satz aus dem Zusammenhang gerissen ist.

    Definitiv nicht, auch nicht der Paralleltransport der 4-Geschwindigkeit. Ich habe das Heft 11/2010, leider kann ich den Artikel auch nicht hochladen.

    Ich habe bei misconceptions in Davis’s thesis 2003 nachgelesen und teile deinen Eindruck, daß sie hier die Doppler Interpretation ablehnt. Dabei beruft sie sich auf die – nach ihrer Rechnung – 23 sigma Abweichung der SRT Vorhersage von der Beobachtung. Nachdem das von Chodorowski 2005 in „Cosmology under Milne’s shadow“ auf nur 2 sigma korrigiert wurde (s. Fig 1), scheint Davis ihre Interpretation geändert zu haben. Soweit zum Spektrum Artikel.

    Du. Das steht zum Beispiel hier:.

    Nein, weder ich noch sonst jemand hehauptet, daß das Milne Modell die primordiale Nucleosynthese vorhersagt. Weshalb dies widersinnig ist, habe ich dargelegt.

    Ganz genau. Es kann vor allem kein nicht-leeres Universum beschreiben. Sobald Materie vorhanden ist, macht es völlig falsche Vorhersagen.

    Nein, die Vorhersage der SN Ia Daten ist außerordentlich nahe an der Beobachtung, wie Chodorowski 2005 gezeigt hat.

    Suum cuique, ich klinke mich jetzt aus.

    P.S. Was ich weniger gut fand: Weshalb auf einige pointierte Bemerkungen, wie die von ‚Ich‘, gleich Geschütze wie „Arroganz …“ auffahren. Geht es nicht etwas gelassener? Indem man (ev. vermeintliche) Spitzen ignoriert, erreicht man mehr. Aber das ist mein persönlicher Eindruck, den man nicht teilen muß.

  126. @Güntim

    Nein, weder ich noch sonst jemand hehauptet, daß das Milne Modell die primordiale Nucleosynthese vorhersagt. Weshalb dies widersinnig ist, habe ich dargelegt.

    Auf die anderen Argumente gehst du immer noch nicht ein.

    Vielleicht ist die „Vorhersage der SN Ia Daten“ tatsächlich in recht guter Übereinstimmung mit der Beobachtung und der Vorhersage des Urknallmodells zu den SN Ia Daten. (Obwohl das erst mal nur Chodorowski behauptet.)
    Daraus darauf zu schließen, Expansion des Raumes und Bewegung durch den Raum (bzw. Urknallmodell und Millne-Modell) wären gleichwertig, obwohl alle anderen Beobachtungen außer den SN Ia Daten dem widersprichen, ist doch exakt das selbe wie folgende Argumentationslinie:

    Das Bohrsche Atommodell sagt den Durchmesser des Wasserstoffatoms in guter Näherung richtig voraus und man kann mit dem Bohrschen Modell die Rydberg-Formel herleiten, also auch die Spektrallinien von Wasserstoff in guter Näherung vorausagen.

    Deswegen ist das Bohrsche Atommodell gleichwertig mit der quantenmechanischen Beschreibung eines Atoms.
    Schließlich geht es nur um die Spektrallinien und den Durchmesser von Wasserstoff.

    Man folgert völlig korrekt:
    Die Vorstellung, dass Elektronen auf bestimmten Bahnen klassisch um den Atomkern kreisen ist deswegen völlig gleichwertig zum quantenmechanischen Orbitalmodell des Atoms.

    Die unzähligen Argumente und Messdaten, die für das quantenmechanischen Orbitalmodell und gegen das Bohrsche Atommodell sprechen, kann man ignorieren, weil ja schließlich die Vorhersagen für die Spektrallinien des Wasserstoffs bei beiden Modellen in Näherung übereinstimmen.

    Bohrmodell und Milne-Modell wurden für unser Universum widerlegt. Da kann man nicht einfach jeweils eine einzige, eigentlich zufällige Übereinstimmung mit der richtigen Theorie und den Beobachtungsdaten suchen und dann behaupten, dass diese Modelle deswegen mit den korrekten Modellen gleichwertig sind.

    Was ich weniger gut fand: Weshalb auf einige pointierte Bemerkungen, wie die von ‚Ich‘, gleich Geschütze wie „Arroganz …“ auffahren. Geht es nicht etwas gelassener? Indem man (ev. vermeintliche) Spitzen ignoriert, erreicht man mehr.

    Ich sehe nicht, was ich bei ‚Ich‘ überhaupt hätte erreichen können. Er ist weder auf meine Argumente eingegangen noch hat er neue Argumente geliefert, nachdem seine erste Behauptung nicht akzeptiert wurde.

    Mit einer bloßen Aneinanderreihung von „pointierte Bemerkungen“ als Antwort muss man sich nicht zufrieden geben, oder?
    Darauf darf man doch mal ganz gelassen hinweisen, nicht?
    Vor allem, wenn man vorher schon mal höflich erwähnt hat, dass man auf so etwas nicht so besonders steht.
    Wenn ich ähnlich „pointiert“ geantwortet hätte, könntest du mir tatsächlich fehlende Gelassenheit vorwerfen.

  127. Edwin Hubble hat sich später gegen die Interpretation als „Ausdehnung des Raums“ ausgesprochen, da er sie als zu problematisch ansah. Wird gern unterschlagen. Wenn dazu die Rotverschiebung nicht als Entfernugnsmesser taugt, wie soll sie da noch als „Beweis“ für die angebliche Ausdehnung taugen?

    1. @Stefan: „Wenn dazu die Rotverschiebung nicht als Entfernugnsmesser taugt, wie soll sie da noch als “Beweis” für die angebliche Ausdehnung taugen?“

      Wer sagt denn, dass die Rotverschiebung sich nicht zur Entfernungsmessung eignet? Ich jedenfalls nicht…

  128. @Stefan

    Mit seiner Distanzierung irrte Hubble halt. Das ist ohnehin lange her. Spätestens seit das nach ihm benannte Weltraumteleskop in den 1990ern Helligkeitsmessungen von Cepheiden-Veränderlichen in anderen Galaxien vorgenommen hat, ist die Beziehung zwischen Entfernung und kosmologischer Rotverschiebung sehr gut belegt und kalibriert. Weitere Messungen an Supernovae vom Typ Ia haben diese Beziehung bis an den Rand des sichtbaren Universums bestätigt. An denen im übrigen auch die zu erwartende Zeitdilatation (die auch eine Variante der kosmologischen Rotverschiebung ist) gezeigt werden konnte.

    1. @Gerd: „Theoretisch könnte es doch auch sein, daß wir uns von der Galaxie fortbewegen….“

      Nein. Weil die Sonne Teil der Milchstraße ist und sich mit ihr gemeinsam bewegt.

  129. Vielen Dank für Ihre sehr schnelle Antwort, Herr Freistetter.

    Mit der Rotation unserer Heimatgalaxis habe ich kein Problem. Andererseits bewegt sich unser Sonnensystem innerhalb der Milchstraße. Da gibt es dann aber auch, meine Meinung, Situationen, in denen sich unser System – eben durch die Rotation bedingt, von dem zu beochtenden fernen System wegbewegt.. Das alleine müßte schon eine Rotverschiebung bedingen, bzw. eine vorhandene verstärken.

    Oder irre ich mich grundsätzlich?

    1. @Kintzel: „Oder irre ich mich grundsätzlich?“

      Nein. Aber die Sonne braucht 220 Millionen Jahre um die Sonne einmal zu umrunden. Es dauert also entsprechend lange, bis sich so ein Effekt auswirkt. Für die Beobachtungen der Astronomen spielt das keine Rolle.

  130. @Gerd Kintzel

    Zum einen wird die Bewegung des Sonnensystems in der Milchstraße wie auch der Milchstraße selbst bereits bei den Rotverschiebungen mit berücksichtigt (d.h. es werden die ermittelten Geschwindigkeiten entsprechend korrigiert). Zum anderen werden die Geschwindigkeiten ferner Galaxien mit zunehmender Entfernung sehr viel größer (bis um ca. das 500-fache) als die Komponenten der Bewegung der Sonne um den lokalen Ruhepunkt, d.h. die Eigenbewegung der Erde spielt bei kosmologischen Entfernungen keine Rolle mehr.

    Diese Eigenbewegung wiederum lässt sich ermitteln anhand der Rotverschiebung der kosmischen Hintergrundstrahlung, die überall von gleich weit weg herkommt und daher eigentlich überall die gleiche Rotverschiebung haben sollte. Sie ist jedoch bipolar mit einem Geschwindigkeitsmaximum von 600 km/s. Dieses Bild zeigt die Situation sehr schön.

  131. Dass Elektromagnetismus der um 10^36 mal stärker als Gravitation ist, keinen Einfluss auf die Kosmologie haben soll, ist offensichtlich falsch. Erkenntnisse der Plasmaphysik zeigen das klar. So wären erfundene Faktoren wie Dunkle Materie, um Bewegung in Galaxien zu erklären, überflüssig. Kein Wunder dass sie nicht gefunden wird, es kann nicht alles mit Gravitation der schwächsten aller Kräfte, erklärt werden.
    Wenn Raum unendlich ist, wie soll er sich dann „ausdehnen“?
    Dass ist ein Beispiel dafür, wie eine mathematische Beschreibung physikalisch keinen Sinn ergibt. Warum sollte Raum endlich sein? Ich bin fast 100% sicher, dass die Gravitationskosmologie sich nicht mehr lange halten kann, die Widersprüche sind zu groß.

  132. Wieviele Strohmänner? Da ist ja fast jeder Satz ein Strohmann!

    @Stefan:

    Dass Elektromagnetismus der um 10^36 mal stärker als Gravitation ist, keinen Einfluss auf die Kosmologie haben soll, ist offensichtlich falsch. Erkenntnisse der Plasmaphysik zeigen das klar.

    Warum soll das offensichtlich sein?

    Wenn Raum unendlich ist, wie soll er sich dann “ausdehnen”?

    Das kann er! Stell dir vor unser Raum wäre einfach nur die reelle Zahlengerade. Nun multipliziere jede Zahl mit 2 dann haben sich alle „Distanzen“ verdoppelt also hat sich unsere 1-Dimensionale unendliche Zahlengerade ausgedehnt.

    Warum sollte Raum endlich sein?

    Warum sollte er unendlich sein. Hier sei nur das Altbekannte Beispiel mit der Ameise auf dem Luftballon erwähnt. Der Ameise würde der Raum (auf den ersten Blick) unendlich erscheinen aber wir, würden die Endlichkeit des Raumes=Luftballon (sofort) sehen.

    Ich bin fast 100% sicher, dass die Gravitationskosmologie sich nicht mehr lange halten kann, die Widersprüche sind zu groß.

    Ich glaube das nicht.

  133. @Stefan

    Dass Elektromagnetismus der um 10^36 mal stärker als Gravitation ist, keinen Einfluss auf die Kosmologie haben soll, ist offensichtlich falsch.

    Da offensichtlich sämtliche Materie, die wir sehen können, elektrisch neutral ist (oder siehst Du irgendwo Zeeman-Effekte im interstellaren Gas?), kann der Elektromagnetismus offenbar nur lokale Auswirkungen haben (z.B. im Plasma der Sterne oder in Akkretionsscheiben um junge Sterne und Schwarze Löcher) .

    Erkenntnisse der Plasmaphysik zeigen das klar. So wären erfundene Faktoren wie Dunkle Materie, um Bewegung in Galaxien zu erklären, überflüssig. Kein Wunder dass sie nicht gefunden wird, es kann nicht alles mit Gravitation der schwächsten aller Kräfte, erklärt werden.

    Dann erläutere mal, wie diese Theorie die Rotation von Galaxien, das Ausbilden von Voids und Filamenten, so wie den Bullet-Cluster erklären.

    Wenn Raum unendlich ist, wie soll er sich dann “ausdehnen”?

    Indem sich jedes lokale Element vergrößert.

    Warum sollte Raum endlich sein?

    Wer sagt das denn? Mit Sicherheit endlich ist nur der Bereich, aus dem uns Licht im endlichen Weltalter erreichen kann. Ob es dahinter endlich weiter in Form einer geschlossenen Topologie, oder unendlich weiter in einer offenen Topologie geht, das weiß bis jetzt noch kein Mensch.

    Ich bin fast 100% sicher, dass die Gravitationskosmologie sich nicht mehr lange halten kann,

    Freu‘ Dich mal nicht zu früh, das Modell ist (auf Hubble zurück gehend) schon bald 85 Jahre alt und durch zahlreiche Beobachtungen bestätigt. Das Weltalter ist bis auf ca. 1% genau bekannt und passt gut zum Alter der ältesten Sterne. Außerdem kann man den Werdegang der Galaxien im Teleskop bis zu den frühesten Vertretern während der Re-Ionisationsphase live mitverfolgen. Kosmische Hintergrundstrahlung, Baryonische Oszillationen, das Verhältnis der Elemente im primordialen Gas, ULRIGs (ultra-luminous infra-red galaxies), kleine Galaxien in der Frühzeit des Universums, die später miteinander kollidieren und verschmelzen, die Metallhäufigkeiten in den Sternenpopulationen I-III, die Galaxienflucht, Zeitdilatation ferner Supernovae und Gamma-Bursts, Quasare, das alles wird durch die Urknalltheorie erklärt. Kann das Eure Plasmatheorie auch? Lass‘ hören!

    die Widersprüche sind zu groß.

    Ich wüsste jetzt keinen. Dass es noch nicht entdeckte Teilchen gibt, die offenbar nur per Gravitation und im günstigsten Fall schwacher Wechselwirkung interagieren, ist ja kein Widerspruch zu irgendetwas. Klär‘ uns mal auf.

  134. Ob man mit der ganzen Theorie recht viel schlauer wird, dass wage ich zu bezweifeln. Ein paar Zitate aus Stephen Hawkings „der Große Entwurf“, auf Seite 81 steht wörtlich:

    Die Quantenphysik sagt uns, dass, egal wie gründlich unsere Beobachtungen der Gegenwart sein mögen,
    die (unbeobachtete) Vergangenheit so unbestimmt wie die Zukunft ist und nur als
    Spektrum von Möglichkeiten existiert. Das Universum hat laut Quantenphysik nicht nur eine einzige Vergangenheit,
    nicht nur eine einzige Geschichte.
    Der Umstand, dass die Vergangenheit nicht eindeutig bestimmt ist, bringt es mit sich, dass sich Beobachtungen, die man an der
    Gegenwart an einem System vornimmt, auf seine Vergangenheit auswirken können.

    Seite 119:
    Daher lassen die Gesetze der M-Theorie verschiedene Universen zu, je nachdem wie die Extradimensionen aufgewickelt sind.

    Was bedeutet das für uns? Wenn die M-Theorie 10 hoch 500 Sätze scheinbarer Gesetze zulässt, wie sind wir dann zu diesem
    Universum und den uns sich darbietenden Gesetzen gekommen? Und was ist mit den anderen möglichen Welten?

    Seite 177
    Aus diesern Gründen ist die M-Theorie der einzige Kandidat für eine vollständige Theorie des Universums. Wenn sie endlich ist –
    und das gilt es noch zu beweisen – dann ist sie das Modell eines Universums, das sich selbst erschafft.
    Wir müssen ein Teil dieses Universums sein, weil es kein anderes konsistentes Modell gibt.

  135. Einstein hat gesagt, die Naturgesetze gelten überall im Universum, die „Urknall-These“ verstößt aber prinzipiell gegen dieses Postulat. 1000fache Lichtgeschwindigkeit ist für einen Steven Hawking kein Mathematisches Problem – ich kaufe schon lange kein Buch mehr von ihm, weil er sich sowieso immer wieder widerspricht. .

    1. @Josef

      Einstein hat gesagt, die Naturgesetze gelten überall im Universum

      Das Relativitätsprinzip war schon Galileo bekannt, Einstein hat das dann bedeutend erweitern können zur SRT.

      die “Urknall-These” verstößt aber prinzipiell gegen dieses Postulat

      Ja und warum genau?

      1000fache Lichtgeschwindigkeit ist für einen Steven Hawking kein Mathematisches Problem

      Ich gehe davon aus du spielst auf die Inflationäre Phase des Universums an. Dabei solltest du beachten das Raumausdehnung nicht gleich Geschwindigkeit ist. Das ist mathematisch auch ganz klar sobald man die Mathematik hinter der allgemeinen Relativitätstheorie versteht (die ist aber nicht ganz leicht). Eine etwas Laienhafte Erklärung liefert Wikipedia.

      ich kaufe schon lange kein Buch mehr von ihm, weil er sich sowieso immer wieder widerspricht.

      Wenn du die Bücher von Hawking nicht magst, dann kann man auch die Bücher von Brian Green empfehlen, oder falls du mathematisch relativ fit bist kann man auch das Buch von Penrose „The Road to Reality“ empfehlen.

      Wer es aber ganz genau wissen will (mit der ganzen Mathematik und der Physik dahinter), dem empfehle ich das Buch von Zee „Einstein Gravity in a nutshell“ (Achtung das Buch ist für Experten!).

    2. @Josef Schwarzinger: Die Begrenzung mit der Lichtgeschwindigkeit gilt für Bewegung IM Raum, nicht für die Expansion des Raums selbst.

  136. Mathematisch ist vieles möglich. Und Geschwindigkeit ist relativ. So wissen wir ja, das es im Universum Sterne gibt, die sich mit x-fach LG von UNS entfernen. Warum soll da mathematisch eine Grenze sein ?

    Das Mathematik und Physik sich manchmal beißen zeigt der ‚running gag‘ der Mathematik :
    In einem Raum sind drei Menschen. Wenn nun Fünf rausgehen, müssen Zwei wieder reinkommen, damit der Raum leer ist.
    Mathematisch legal, siehe Kontoauszüge, aber physikalisch ? Eher nicht.

    @ Josef in #156 :
    Gibt es einen Buchtitel zu dem Hawking Zitat ?

  137. Eine Frage zur beschleunigten Expansion:

    Galaxie A ist 1 Mrd. Lj entfernt und Galaxie B ist 2 Mrd. Lj
    entfernt. Wenn sich Galaxie B schneller entfernt als Galaxie A, heißt das dann, dass das Universum vor 2 Mrd. Jahren
    schneller expandiert ist als vor 1 Mrd. Jahren?

    VG
    Andi

  138. @Andreas

    Wenn sich Galaxie B schneller entfernt als Galaxie A, heißt das dann, dass das Universum vor 2 Mrd. Jahren
    schneller expandiert ist als vor 1 Mrd. Jahren?

    Nein, die Expansionsgeschwindigkeit nimmt in erster Näherung linear mit der Entfernung zu. Der Hubble Parameter beträgt ca. 70 km/s/MPc (1 MPc = 3,26 Mio Lichtjahre). Bei 1 Mrd. Lj Entfernung erwartet man also 21500 km/s Radialgeschwindigkeit, bei 2 Mrd. Lj 43000 km/s.

    Was Florian meint ist, dass es sich gezeigt hat, dass über größere Entfernungen der Hubble Parameter seinen Wert ändert. Es zeigte sich, dass sich sehr ferne Galaxien langsamer als bei konstantem Hubble-Parameter von uns entfernen. Heute läuft die Expansion schneller ab. Daraus ergibt sich auch ein höheres Weltalter (der alte Wert in den frühen 90ern lag bei 11-12 Milliarden Jahren, heute sind es 13,8 Milliarden).

  139. Gegen These! Das All dehnt sich zwar aus, verlangsamt aber seine Ausdehnung. Grund, weit entfernte Galaxien, scheinen schneller von uns weg zu „fliegen“ als nähere, da das Licht weit entfernter Galaxien aber viel viel älter ist, als das von nahen Galaxien schließe ich. Die Ausdehnung verlangsamt sich.

  140. @Lea

    Das All dehnt sich zwar aus, verlangsamt aber seine Ausdehnung.

    Das hat man bis Mitte der 19990er für selbstverständlich gehalten, aber die Messungen der Rotverschiebung bei verschiedenen Entfernungen hat das eindeutig widerlegt (und nun hat man ein Erklärungsproblem). Außerdem käme man dann mit dem Alter der ältesten Sterne nicht hin, das war damals ein Problem. Wenn die Expansion sich verlangsamt, also früher schneller verlief als in der heutigen Umgebung der Milchstraße, sind die fernen Galaxien in kürzerer Zeit an ihre heutigen Orte gekommen, d.h. der Urknall wäre weniger langer her. Man kam damals auf 11 Milliarden Jahre, aber die ältesten Sterne in Kugelsternhaufen waren fast 13 Milliarden Jahre alt. Heute weiß man, dass die Expansion sich beschleunigt, das All ist älter (13,8 Milliarden Jahre, wenn ich die zuletzt aktuelle Zahl richtig erinnere), und mit den ältesten Sternen passt es wieder.

  141. Hallo Florian,
    zunächst einmal vielen Dank für deine Erklärungen!
    Könntest du mir evt mit der mitbewegten Entfernung helfen? Sie ist doch keine Observabel, inwiefern lässt sich etwas mit ihr anfangen? Ich dachte auch , dass sie konstant bleibt, weil die Expansion herausfaktorisiert wurde?
    Ich dachte die proper distance würde eher angeben, wie weit es entfernt wäre, könnte man es „heute“ simultan messen?
    Verzeih, wenn die Frage doof ist, mein Hirn ist vor lauter Entfernungmaße ganz „balla-balla“
    Liebe Grüße <3

Schreibe einen Kommentar zu volki Antworten abbrechen

Deine E-Mail-Adresse wird nicht veröffentlicht. Erforderliche Felder sind mit * markiert

Diese Website verwendet Akismet, um Spam zu reduzieren. Erfahre mehr darüber, wie deine Kommentardaten verarbeitet werden.